NUR104 Medical Surgical Nursing Final Reveiw Questions

¡Supera tus tareas y exámenes ahora con Quizwiz!

Select all the signs and symptoms below that can present in myasthenia gravis:* A. Respiratory failure B. Increased salivation C. Speech impairment D. Ptosis E. Slurred speech F. Restlessness G. Mask-like appearance of looking sleepy H. Difficulty swallowing

The answers are A, C, D, E, F (restlessness from hypoxia, which is experienced with respiratory failure), G, and H.

Which nursing intervention is most appropriate for the nurse to implement for a client with​ delirium? (Select all that​ apply.) Restricting visiting hours Providing adequate pain management Using calendars and clocks to reorient the client Assigning the same nurse to care for the client each day Keeping lights on at all​ times, so the client can see her surroundings

Providing adequate pain management Using calendars and clocks to reorient the client Assigning the same nurse to care for the client each day

A nurse is assigned to care for a client with nephrotic syndrome. The nurse assesses which important parameter on a daily basis? a) weight b) albumin levels c) activity tolerance d) blood urea nitrogen (BUN) level

a) weight

A patient with Parkinson's disease who takes levodopa/carbidopa [Sinemet] comes to the clinic for a semiannual physical examination. Which question is the most important for the nurse to ask? a) "Have you noticed any swelling in your feet?" b) "Are you having vivid dreams or hallucinations?" c) "Have you noticed any changes in your stool?" d) "Have you had your flu vaccine?"

b) "Are you having vivid dreams or hallucinations?" Patients taking levodopa/carbidopa [Sinemet] are at increased risk for the psychiatric side effects of levodopa, including visual hallucinations, vivid dreams, nightmares, and paranoid ideation. The other questions are not directly related to problems that are likely to occur with this drug.

Which statement should the nurse include in the teaching plan for a patient being started on levodopa/carbidopa [Sinemet] for newly diagnosed Parkinson's disease? a) Take the medication on a full stomach. b) Change positions slowly. c) The drug may cause the urine to be very dilute. d) Carbidopa has many adverse effects.

b) Change positions slowly. Postural hypotension is common early in treatment, so the patient should be instructed to change positions slowly. Administration with meals should be avoided, if possible, because food delays the absorption of the levodopa component. If the patient is experiencing side effects of nausea and vomiting, administration with food may need to be considered. The levodopa component in Sinemet may darken the color of the urine. Carbidopa has no adverse effects of its own.

Which nursing assessment finding indicates that the client who has undergone renal transplant has not met expected outcomes? a) Weight loss b) Fever c) Absence of pain d) Diuresis

b) Fever Fever is an indicator of infection or transplant rejection.

A 14 year-old female has sickle cell anemia. Which factors below can increase the patient's risk for developing sickle cell crisis? Select all that apply: a) Shellfish b) Infection c) Dehydration d) Hypoxia e) Low altitudes f) Hemorrhage g) Strenuous Exercise

b) Infection c) dehydration d) Hypoxia f) Hemorrhage g) strenuous exercise - The answers are B, C, D, F and G. Sickle cell crisis can occur when the body experiences low amounts of oxygen in the body (so think about something that increases the body's need for oxygen or affects how oxygen is being transported). Therefore, infection (especially respiratory infections), dehydration, hypoxia, HIGH (not low) altitudes, hemorrhage (blood loss), or strenuous exercise can lead to a sickle cell crisis.

A patient with a history of numbness, weakness, and blurred vision recently was diagnosed with multiple sclerosis (MS). What does the nurse understand to be the underlying pathophysiology for these symptoms? a) An imbalance of dopamine and acetylcholine in the central nervous system b) Inflammation and myelin destruction in the central nervous system c) An inability of serotonin to bind to its receptors in the chemoreceptor trigger zone d) High-frequency discharge of neurons from a specific focus area of the brain

b) Inflammation and myelin destruction in the central nervous system The underlying pathophysiology of MS is related to myelin destruction and slowing of axonal conduction related to inflammation within the central nervous system. The demyelination leads to the characteristic neurologic symptoms associated with MS.

Three year old Carlo has been admitted to the pediatric unit with a tentative diagnosis of nephrotic syndrome Prednisone is prescribed for Carlo. The nurse evaluate its effectiveness by a) checking his BP every 4 hours b) checking his urine for protein c) weighing him each morning before breakfast d) observing him for behavioral changes

b) checking his urine for protein Monitor side effect of prolonged steroid therapy Hyperglycemia - test urine monitor growth of child by checking height because steroid has growth suppressing effect by preventing calcium deposition in the bones Gastric Irritation - give milk or meals, test for occult blood, administer with antacids Avoid exposure to infection because child is immunosuppressed

The client with acute kidney injury has a serum potassium of 6.0 mEq/L. The nurse would plan which of the following as a priority action? a) check the sodium level b) place the client on a cardiac monitor c) encourage increased vegetables in the diet d) allow an extra 500 ml of fluid intake to dilute the electrolyte concentration

b) place the client on a cardiac monitor

The nurse is caring for a patient with superficial partial-thickness burns of the face sustained within the last 12 hours. Upon assessment the nurse would expect to find which manifestation? a. blisters b. reddening of the skin c. destruction of all skin layers d. damage to sebaceous glands

b. The clinical appearance of superficial partial-thickness burns includes erythema, blanching with pressure, and pain and minimal swelling with no vesicles or blistering during the first 24 hours.

Which disease is characterized by increased body metabolism, tachycardia, increased body temperature, and anxiety. a.) Hashimoto's thyroiditis b.) Graves' disease c.) Addison's disease d.) Cushing's syndrome

b.) Graves' disease

The nurse correctly identifies which of the following patients is at the greatest risk for developing sepsis. a. A patient 2 days post-op with a clean, dry incision site without any drainage b. A patient in the ICU with a qSOFA score of 1 c. A patient that has a pressure ulcer on their heel with a fever and a diagnosis of type 2 diabetes with poorly controlled blood sugar d. A patient about to be discharged to a Long Term Acute Care unit 5 days post chest tube removal without complications

c. A patient that has a pressure ulcer on their heel with a fever and a diagnosis of type 2 diabetes with poorly controlled blood sugar

Which characteristic will the nurse associate with a focal seizure? a. The patient lost consciousness during the seizure. b. The seizure involved both sides of the patient's brain. c. The seizure involved lip smacking and repetitive movements. d. The patient fell to the ground and became stiff for 20 seconds.

c. The seizure involved lip smacking and repetitive movements. Complex focal seizure is characterized commonly by lip smacking and automatisms (repetitive movements that may not be appropriate). Loss of consciousness, bilateral brain involvement, and a tonic phase are associated with generalized seizure activity.

Which of the following anti-hypertensive medications is contraindicated for clients with renal insufficiency? a) beta-adrenergic blockers b) calcium-channel blockers c) direct-acting vasodilators d) angiotensin-converting enzyme inhibitors

d) angiotensin-converting enzyme inhibitors

In the oliguric phase of renal failure, what is the most appropriate nursing diagnosis? a) fluid volume deficit b) activity intolerance c) ineffective breathing pattern d) fluid volume excess

d) fluid volume excess

The client is diagnosed with diabetes insipidus. Which laboratory value should bemonitored by the nurse? 1. Serum sodium. 2. Serum calcium 3. Urine glucose. 4. Urine white blood cells.

1 The client will have an elevated sodium level as a result of low circulating blood volume. The fluid is being lost through the urine. Diabetes means "to pass through" inGreek, indicating polyuria, a symptom shared with diabetes mellitus. Diabetes insipidus is a totally separate disease process.

Which of the following can be used as treatments for hyperkalemia? Select all that apply 1) Insulin 2) Kayexalate 3) IV dextrose 4) Calcium 5) Inotropium

1, 2, 3 Other: albuterol

The client diagnosed with sickle cell anemia comes to the ED c/o joint pain throughout the body. Oral temp 102.4, SpO2 91%. Which action should the nurse implement first: 1. Request ABGs 2. Administer oxygen 3. Start IV 4. Administer analgesic

2. Administer oxygen Sickling episodes are triggered by low oxygenation.

A pediatric nurse clinician is discussing the pathophysiology related to childhood leukemia with a class of nursing students. Which statement made by a nursing student indicates a lack of understanding of the pathophysiology of this disease? " 1. Normal bone marrow is replaced by blast cells 2. Red blood cell production is affected 3. the platelet count is decreased 4. the presence of a reed-sternberg cell is found on biopsy

4. Reed-sternberg Cell is found in Hodgkins

The family of a client who is diagnosed with delirium asks the​ nurse, "Can you give us any pointers on how we can help our father with his​ delirium?" Which statement should the nurse include in the​ response? (Select all that​ apply.) ​"Promote consistency in your​ father's schedule." ​"Make sure that your father has appropriate​ nutrition." ​"Encourage your father to partake in new activities​ daily." ​"Reassure your father that delirium is often​ temporary." ​"If your father appears to be confused as to who he​ is, do not correct his​ misconceptions."

"Promote consistency in your​ father's schedule." ​"Make sure that your father has appropriate​ nutrition." ​"Reassure your father that delirium is often​ temporary."

The patient with leukemia has acute disseminated intravascular coagulation (DIC) and is bleeding. What diagnostic findings should the nurse expect to find? 1. Elevated D-dimers 2. Elevated fibrinogen 3. Reduced prothrombin time (PT) 4. Reduced fibrin degradation products (FDPs)

14. Elevated D-dimers a. The D-dimer is a specific marker for the degree of fibrinolysis and is elevated with DIC. FDP is elevated as the breakdown products from fibrinogen and fibrin are formed. Fibrinogen and platelets are reduced. PT, PTT, aPTT, and thrombin time are all prolonged.

The client diagnosed with a pituitary tumor has developed syndrome of inappropriateantidiuretic hormone (SIADH). Which interventions would the nurse implement? 1. Assess for dehydration and monitor blood glucose levels. 2. Assess for nausea and vomiting and weigh daily. 3. Monitor potassium levels and encourage fluid intake. 4. Administer vasopressin IV and conduct a fluid deprivation test.

2 Early signs and symptoms are nausea and vomiting. The client has a syndrome of the inappropriate secretion of the antidiuresis(against allowing the body to urinate) hormone. In other words, the client is producing a hormone that will not allow the client to urinate.

What are signs of Sepsis/Septic shock? a. tachycardia b. hypotension c. fever d. bradycardia e. polyuria f. malaise g. tachypnea

A, B, C, F, G

You're caring for a patient with an indwelling catheter. The patient complains of spasm like pain at the catheter insertion site. Which of the following options below are other signs and symptoms the patient could experience or the nurse could observe if a urinary tract infection was present? SELECT-ALL-THAT-APPLY:* A. Increased WBC B. Crystalluria C. Positive McBurney's Sign D. Feeling the need to void even though a catheter is present E. Dark and cloudy urine F. Cramping

A, D, E, F

A nurse in the ICU is caring for a group of patients at risk for sepsis. Which of the following would put the patient at the highest risk? A. An 85-year-old with an undiagnosed urinary tract infection B. A 55-year-old with pericarditis C. A 60 year old with iron deficiency anemia D. A 25 year old with an upper respiratory infection

A. An 85 year old with an undiagnosed urinary tract infection

Four days following a stroke, a patient is to start oral fluids and feedings. Before feeding the patient, the nurse should first a. check the patient's gag reflex b. order a soft diet for the patient c. raise the head of the bed to sitting position d. evaluate the patient's ability to swallow small sips of ice water

A: check the patient's gag reflex- the first step in providing oral feedings for a patient with a stroke is ensuring that the patient has an intact gag reflex because oral feedings will not be provided if gag reflex is impaired. The nurse should then evaluate the patient's ability to swallow ice chips or ice water after placing the patient in an upright position

I AM SO IMPRESSED YOU ARE STILL AT IT! KEEP UP THE GREAT WORK. YOUR HARD WORK WILL PAY OFF! AGAIN... STAND UP, WIGGLE, SING, AND REMEMBER WHY YOU ARE DOING THIS. HUGS!

AGAIN... STAND UP, WIGGLE, SING, AND REMEMBER WHY YOU ARE DOING THIS. HUGS!

The patient with type 1 diabetes mellitus is having a seizure. Which medication should the nurse anticipate will be administered first? a. IV dextrose solution b. IV diazepam (Valium) c. IV phenytoin (Dilantin) d. Oral carbamazepine (Tegretol)

ANS A This patient's seizure could be caused by low blood glucose, so IV dextrose solution would be given first to stop the seizure. IV diazepam, IV phenytoin, and oral carbamazepine would be used first to treat seizures from other causes such as head trauma, drugs, and infections. These drugs will be tried if the IV dextrose is ineffective.

Which information will the nurse include in teaching a female patient who has peripheral arterial disease, type 2 diabetes, and sensory neuropathy of the feet and legs? a. Choose flat-soled leather shoes. b. Set heating pads on a low temperature. c. Use callus remover for corns or calluses. d. Soak feet in warm water for an hour each day.

ANS: A The patient is taught to avoid high heels and that leather shoes are preferred. The feet should be washed, but not soaked, in warm water daily. Heating pad use should be avoided. Commercial callus and corn removers should be avoided. The patient should see a specialist to treat these problems.

A patient experiences thrombocytopenia. The nurse should monitor the patient for which major complication? 1 Fatigue 2 Weakness 3 Hemorrhage 4 Abdominal pain

3 Hemorrhage

The nurse is reviewing the lab reports of several patients. Which report is consistent with a diagnosis of thrombocytopenia? 1 Hemoglobin 13 2 Hemoglobin 16 3 Platelets 20,000/µL 4 Platelets 1,000,000/ µL

3 Platelets 20,000/µL

A 70-year-old female patient with left-sided hemiparesis arrives by ambulance to the emergency department. Which action should the nurse take first? a. Monitor the blood pressure. b. Send the patient for computed tomography (CT) scan. c. Check the respiratory rate and effort. d. Assess the Glasgow Coma Scale score.

ANS: C The initial nursing action should be to assess the airway and take any needed actions to ensure a patent airway. The other activities should take place quickly after the ABCs (airway, breathing, and circulation) are completed. DIF: Cognitive Level: Apply (application) REF: 1397-1398 OBJ: Special Questions: Prioritization TOP: Nursing Process: Implementation MSC: NCLEX: Physiological Integrity

An unresponsive patient with type 2 diabetes is brought to the emergency department and diagnosed with hyperosmolar hyperglycemic syndrome (HHS). The nurse will anticipate the need to a. give 50% dextrose. b. insert an IV catheter. c. initiate O2 by nasal cannula. d. administer glargine (Lantus) insulin.

ANS: B HHS is initially treated with large volumes of IV fluids to correct hypovolemia. Regular insulin is administered, not a long-acting insulin. There is no indication that the patient requires O2. Dextrose solutions will increase the patient's blood glucose and would be contraindicated.

Which of these patients being seen at the human immunodeficiency virus (HIV) clinic should the nurse assess first? a. Patient whose latest CD4+ count is 250/µL b. Patient whose rapid HIV-antibody test is positive c. Patient who has had 10 liquid stools in the last 24 hours d. Patient who has nausea from prescribed antiretroviral drugs

ANS: C The nurse should assess the patient for dehydration and hypovolemia. The other patients also will require assessment and possible interventions, but do not require immediate action to prevent complications such as hypovolemia and shock

Which laboratory value reported to the nurse by the unlicensed assistive personnel (UAP) indicates an urgent need for the nurse's assessment of the patient? a. Bedtime glucose of 140 mg/dL b. Noon blood glucose of 52 mg/dL c. Fasting blood glucose of 130 mg/dL d. 2-hr postprandial glucose of 220 mg/dL

ANS: B The nurse should assess the patient with a blood glucose level of 52 mg/dL for symptoms of hypoglycemia and give the patient a carbohydrate- containing beverage such as orange juice. The other values are within an acceptable range or not immediately dangerous for a patient with diabetes.

Which collaborative treatment would the nurse anticipate in the client diagnosed with DIC? 1. Administer oral anticoagulants 2. Prepare for plasmapheresis 3. Administer fresh frozen plasma 4. Calculate the intake and output

3. Fresh frozen plasma and platelet concentrates are administered to restore clotting factors and platelets

Which is most important to respond to in a patient presenting with a T3 spinal injury? A. Blood pressure of 88/60 mm Hg, pulse of 56 beats/minute B. Deep tendon reflexes of 1+, muscle strength of 1+ C. Pain rated at 9 D. Warm, dry skin

A. Blood pressure of 88/60 mm Hg, pulse of 56 beats/minute Neurogenic shock is a loss of vasomotor tone caused by injury, and it is characterized by hypotension and bradycardia. The loss of sympathetic nervous system innervations causes peripheral vasodilation, venous pooling, and a decreased cardiac output. The other options can be expected findings and are not as significant. Patients in neurogenic shock have pink and dry skin, instead of cold and clammy, but this sign is not as important as the vital signs.

The nurse is discharging a patient admitted with a transient ischemic attack (TIA). For which medications might the nurse expect to provide discharge instructions (select all that apply)? A. Clopidogrel (Plavix) B. Enoxaparin (Lovenox) C. Dipyridamole (Persantine) D. Enteric-coated aspirin (Ecotrin) E. Tissue plasminogen activator (tPA)

A. Clopidogrel (Plavix) C. Dipyridamole (Persantine) D. Enteric-coated aspirin (Ecotrin) Aspirin is the most frequently used antiplatelet agent. Other drugs to prevent clot formation include clopidogrel (Plavix), dipyridamole (Persantine), ticlopidine (Ticlid), combined dipyridamole and aspirin (Aggrenox), and anticoagulant drugs, such as oral warfarin (Coumadin). Tissue plasminogen activator is a fibrinolytic medication used to treat ischemic stroke not prevent TIAs or strokes.

A 76 year old female is admitted due to a recent fall. The patient is confused and agitated. The family members report that this is not normal behavior for the patient. They explain that the patient is very active in the community and cares for herself. Based on the information you have gathered about the patient, which physician's order takes priority?* A. "Collect a urinalysis" B. "Collect a T3 and T4 level" C. "Insert a Foley Catheter" D. "Keep patient NPO"

A. Collect a urinalysis

Which modifiable risk factors for stroke would be most important for the nurse to include when planning a community education program? A. Hypertension B. Hyerlipidemia C. Alcohol consumption D. Oral contraceptive use

A. Hypertension Hypertension is the single most important modifiable risk factor, but it is still often undetected and inadequately treated. The public is often more aware of hyperlipidemia and oral contraceptive use as risk factors for stroke. Alcohol is also a modifiable risk factor.

Clinical manifestations associated with a diagnosis of type 1 DM include all of the following except: A. Hypoglycemia B. Hyponatremia C. Ketonuria D. Polyphagia

A. Hypoglycemia

When assessing for potential adverse effects of fludrocortisone (Florinef), the nurse monitors for signs and symptoms of which condition? A. Hypokalemia B. Hypovolemia C. Hyponatremia D. Hypercalcemia

A. Hypokalemia

A nurse suspects a patient to be septic. What manifestations would the nurse expect to see? (select all that apply) A. Hypotension B. Confusion C. Tachypnea D. Hypertension E. Bradycardia

A. Hypotension B. Confusion C. Tachypnea

Computed tomography (CT) of a 68-year-old male patient's head reveals that he has experienced a hemorrhagic stroke. What is the priority nursing intervention in the emergency department? A. Maintenance of patient's airway. B. Positioning to promote cerebral perfusion. C. Control of fluid and electrolyte imbalances. D. Administration of tissue plasminogen activator (tPA)

A. Maintenance of patient's airway. Maintenance of a patent airway is the priority in the acute care of a patient with a hemorrhagic stroke. It supersedes the importance of fluid and electrolyte imbalance and positioning. tPA is contraindicated in hemorrhagic stroke.

Which of the following is not a symptom of hyperkalemia? A. Positive Chvostek's sign B. Decreased blood pressure C. Muscle twitches/cramps D. Weak and slow heart rate

A. Positive Chvostek's sign

A patient has a potassium level of 9.0. Which nursing intervention is a priority? A. Prepare the patient for dialysis and place the patient on a cardiac monitor B. Administer Spironolactone C. Place patient on a potassium restrictive diet D. Administer a laxative

A. Prepare the patient for dialysis and place the patient on a cardiac monitor

This nerve controls the PNS which stimulates contraction and relaxation of the smooth muscle in the GI tract? A. X B. V C. VI D. VII

A. X - The vagus nerve

A nurse develops a teaching plan for a patient diagnosed with basal cell carcinoma (BCC). Which information should the nurse include in the teaching plan? a. Treatment plans include watchful waiting. b. Screening for metastasis will be important. c. Low dose systemic chemotherapy is used to treat BCC. d. Minimizing sun exposure will reduce risk for future BCC.

ANS: D BCC is frequently associated with sun exposure and preventive measures should be taken for future sun exposure. BCC spreads locally, and does not metastasize to distant tissues. Since BCC can cause local tissue destruction, treatment is indicated. Local (not systemic) chemotherapy may be used to treat BCC

Which assessment information is most important for the nurse to obtain when evaluating whether treatment of a patient with anaphylactic shock has been effective? a. Heart rate b. Orientation c. Blood pressure d. Oxygen saturation

ANS: D Because the airway edema that is associated with anaphylaxis can affect airway and breathing, the O2 saturation is the most critical assessment. Improvements in the other assessments will also be expected with effective treatment of anaphylactic shock.

Client teaching for a client taking a thiazide diuretic includes which instruction? a. Instruct the client to add salt liberally to his food. b. Instruct the client to decrease intake of potassium-rich foods. c. Instruct the client to check pulse rate if digoxin is taken with hydrochlorothiazide. d. Advise the client to rise slowly from a sitting to a standing position.

ANS: D Diuretics can lead to orthostatic hypotension. The client should limit Na intake and ingest K-containing foods.

A patient who is human immunodeficiency virus (HIV)-infected has a CD4+ cell count of 400/µL. Which factor is most important for the nurse to determine before the initiation of antiretroviral therapy (ART) for this patient? a. HIV genotype and phenotype b. Patient's social support system c. Potential medication side effects d. Patient's ability to comply with ART schedule

ANS: D Drug resistance develops quickly unless the patient takes ART medications on a strict, regular schedule. In addition, drug resistance endangers both the patient and the community. The other information is also important to consider, but patients who are unable to manage and follow a complex drug treatment regimen should not be considered for ART.

A nurse is teaching a client about lifestyle changes when taking a potassium-sparing diuretic. Which statement indicates a need for more teaching? a. "I need to have my blood drawn frequently." b. "I need to call the clinic if I am urinating less than every 2 hours." c. "I need to be careful when out in the sun." d. "I need to eat foods like bananas frequently."

ANS: D Eating high-potassium foods with potassium-sparing diuretics can lead to hyperkalemia.

A patient with left-sided weakness that started 60 minutes earlier is admitted to the emergency department and diagnostic tests are ordered. Which test should be done first? a. Complete blood count (CBC) b. Chest radiograph (Chest x-ray) c. 12-Lead electrocardiogram (ECG) d. Noncontrast computed tomography (CT) scan

ANS: D Rapid screening with a noncontrast CT scan is needed before administration of tissue plasminogen activator (tPA), which must be given within 4.5 hours of the onset of clinical manifestations of the stroke. The sooner the tPA is given, the less brain injury. The other diagnostic tests give information about possible causes of the stroke and do not need to be completed as urgently as the CT scan. DIF: Cognitive Level: Apply (application) REF: 1395 OBJ: Special Questions: Prioritization TOP: Nursing Process: Implementation MSC: NCLEX: Physiological Integrity

A 73-year-old patient with a stroke experiences facial drooping on the right side and right-sided arm and leg paralysis. When admitting the patient, which clinical manifestation will the nurse expect to find? a. Impulsive behavior b. Right-sided neglect c. Hyperactive left-sided tendon reflexes d. Difficulty comprehending instructions

ANS: D Right-sided paralysis indicates a left-brain stroke, which will lead to difficulty with comprehension and use of language. The left-side reflexes are likely to be intact. Impulsive behavior and neglect are more likely with a right-side stroke. DIF: Cognitive Level: Apply (application) REF: 1407 TOP: Nursing Process: Assessment MSC: NCLEX: Physiological Integrity

A 56-year-old patient arrives in the emergency department with hemiparesis and dysarthria that started 2 hours previously, and health records show a history of several transient ischemic attacks (TIAs). The nurse anticipates preparing the patient for a. surgical endarterectomy. b. transluminal angioplasty. c. intravenous heparin administration. d. tissue plasminogen activator (tPA) infusion.

ANS: D The patient's history and clinical manifestations suggest an acute ischemic stroke and a patient who is seen within 4.5 hours of stroke onset is likely to receive tPA (after screening with a CT scan). Heparin administration in the emergency phase is not indicated. Emergent carotid transluminal angioplasty or endarterectomy is not indicated for the patient who is having an acute ischemic stroke. DIF: Cognitive Level: Apply (application) REF: 1391 | 1398 TOP: Nursing Process: Planning MSC: NCLEX: Physiological Integrity

The client is being treated with a thiazide diuretic. The nurse should expect to see an increased serum _____ as a result of treatment with this drug. a. potassium b. sodium c. magnesium d. calcium

ANS: D Treatment with a thiazide diuretic will produce an increase in the levels of serum calcium, glucose, and uric acid, and a decrease in the levels of serum potassium, sodium, and magnesium.

Which prescribed intervention will the nurse implement first for a patient in the emergency department who is experiencing continuous tonic-clonic seizures? a. Give phenytoin (Dilantin) 100 mg IV. b. Monitor level of consciousness (LOC). c. Administer lorazepam (Ativan) 4 mg IV. d. Obtain computed tomography (CT) scan.

ANS: C To prevent ongoing seizures, the nurse should administer rapidly acting antiseizure medications such as the benzodiazepines. A CT scan is appropriate, but prevention of any seizure activity during the CT scan is necessary. Phenytoin will also be administered, but it is not rapidly acting. Patients who are experiencing tonic-clonic seizures are nonresponsive, although the nurse should assess LOC after the seizure.

A patient who was admitted with diabetic ketoacidosis secondary to a urinary tract infection has been weaned off an insulin drip 30 minutes ago. The patient reports feeling lightheaded and sweaty. Which action should the nurse take first? a. Infuse dextrose 50% by slow IV push. b. Administer 1 mg glucagon subcutaneously. c. Obtain a glucose reading using a finger stick. d. Have the patient drink 4 ounces of orange juice.

ANS: C The patient's clinical manifestations are consistent with hypoglycemia, and the initial action should be to check the patient's glucose with a finger stick or order a stat blood glucose. If the glucose is low, the patient should ingest a rapid-acting carbohydrate, such as orange juice. Glucagon or dextrose 50% might be given if the patient's symptoms become worse or if the patient is unconscious.

The nurse is preparing to administer doses of hydrochlorothiazide and digoxin (Lanoxin) to a patient who has heart failure. The patient reports having blurred vision. The nurse notes a heart rate of 60 beats per minute and a blood pressure of 140/78 mm Hg. Which action will the nurse take? a. Administer the medications and request an order for serum electrolytes. b. Give both medications and evaluate serum blood glucose frequently. c. Hold the digoxin and notify the provider. d. Hold the hydrochlorothiazide and notify the provider.

ANS: C When thiazide diuretics are taken with digoxin, patients are at risk of digoxin toxicity because thiazides can cause hypokalemia. The patient has bradycardia and blurred vision, which are both signs of digoxin toxicity. The nurse should hold the digoxin and notify the provider. Serum electrolytes may be ordered, but the digoxin should not be given.

A 71-year-old patient with Alzheimer's disease (AD) who is being admitted to a long-term care facility has had several episodes of wandering away from home. Which action will the nurse include in the plan of care? a. Reorient the patient several times daily. b. Have the family bring in familiar items. c. Place the patient in a room close to the nurses' station. d. Ask the patient why the wandering episodes have occurred.

ANS: C-Place the patient in a room close to the nurses' station. Patients at risk for problems with safety require close supervision. Placing the patient near the nurse's station will allow nursing staff to observe the patient more closely

Which data obtained when assessing a patient who had a kidney transplant 8 years ago and who is receiving the immunosuppressants tacrolimus (Prograf), cyclosporine (Sandimmune), and prednisone (Deltasone) will be of most concern to the nurse? a. The blood glucose is 144 mg/dl. b. The patient has a round, moonlike face. c. There is a nontender lump in the axilla. d. The patient's blood pressure is 150/92.

Answer: C Rationale: A nontender lump suggests a malignancy such as a lymphoma, which could occur as a result of chronic immunosuppressive therapy. The elevated glucose, moon face, and hypertension are possible side effects of the prednisone and should be addressed, but they are not as great a concern as the possibility of a malignancy.

A patient begins hemodialysis after having had conservative management of chronic kidney disease. The nurse explains that one dietary regulation that will be changed when hemodialysis is started is that a. unlimited fluids are allowed since retained fluid is removed during dialysis. b. increased calories are needed because glucose is lost during hemodialysis. c. more protein will be allowed because of the removal of urea and creatinine by dialysis. d. dietary sodium and potassium are unrestricted because these levels are normalized by dialysis.

Answer: C Rationale: Once the patient is started on dialysis and nitrogenous wastes are removed, more protein in the diet is allowed. Fluids are still restricted to avoid excessive weight gain and complications such as shortness of breath. Glucose is not lost during hemodialysis. Sodium and potassium intake continues to be restricted to avoid the complications associated with high levels of these electrolytes.

Before administering sodium polystyrene sulfonate (Kayexalate) to a patient with hyperkalemia, the nurse should assess a. the BUN and creatinine. b. the blood glucose level. c. the patient's bowel sounds. d. the level of consciousness (LOC).

Answer: C Rationale: Sodium polystyrene sulfonate (Kayexalate) should not be given to a patient with a paralytic ileus (as indicated by absent bowel sounds) because bowel necrosis can occur. The BUN and creatinine, blood glucose, and LOC would not impact on the nurse's decision to give the medication.

Two hours after a kidney transplant, the nurse obtains all these data when assessing the patient. Which information is most important to communicate to the health care provider? a. The BUN and creatinine levels are elevated. b. The urine output is 900 to 1100 ml/hr. c. The patient's central venous pressure (CVP) is decreased. d. The patient has level 8 (on a 10-point scale) incision pain when coughing.

Answer: C Rationale: The decrease in CVP suggests hypovolemia, which must be rapidly corrected to prevent renal hypoperfusion and acute tubular necrosis. The other information is not unusual in a patient after a transplant.

A patient in the oliguric phase of acute renal failure has a 24-hour fluid output of 150 ml emesis and 250 ml urine. The nurse plans a fluid replacement for the following day of ___ ml. a. 400 b. 800 c. 1000 d. 1400

Answer: C Rationale: Usually fluid replacement should be based on the patient's measured output plus 600 ml/day for insensible losses.

Dr. Jones prescribes corticosteroids for a child with nephritic syndrome. What is the primary purpose of administering corticosteroids to this child? A. To increase blood pressure. B. To reduce inflammation. C. To decrease proteinuria. D. To prevent infection.

Answer: C. To decrease proteinuria. The primary purpose of administering corticosteroids to a child with nephritic syndrome is to decrease proteinuria.

A patient who is in hypovolemic shock has the following clinical signs: Heart rate 120 beats/minute, blood pressure 80/55 mmHg and urine output 20ml/hr. After administering an IV fluid bolus, which of these signs if noted by the healthcare provider is the best indication of improved perfusion? A. Heart rate drops to 100 beats/minute. B. Right atrial pressure increases. C. Urine output increases to 30mL/hour. D. Systolic blood pressure increases to 85 mmHg.

Answer: C. Urine output increases to 30mL/hour. C: An increase in the urine output indicates that perfusion is improving. A, B, and D: These options are not indicative of an improved perfusion.

A school nurse has sent home four children who show evidence of pediculosis capitis. What is an important instruction the nurse should include in the note being sent home to parents? A) The child's scalp should be monitored for 48 to 72 hours before starting treatment. B) Nits may have to be manually removed from the child's hair shafts. C) The disease is self-limiting and symptoms will abate within 1 week. D) Efforts should be made to improve the child's level of hygiene.

B) Nits may have to be manually removed from the child's hair shafts.

You're providing an in-service on skin cancer prevention to a local group of parents and grandparents. One topic you discuss is the identification of melanoma. Select all the characteristics of this type of skin cancer you will educate the participants about:* A. Diameter of 8 mm or higher B. Red, purple, and black color C. Dissimilar borders D. Uniform appearance E. Increase in size, shape, color

B, C, and E. When educating about how to identify melanoma using the ABCDE acronym. A: asymmetrical, B: borders uneven (dissimilar), C: color dark or multiple colors, D: diameter greater than 6 mm (NOT 8 mm). E: evolution (lesion increases in size, shape, color)

The nurse provides information to the caregiver of a 68-year-old man with epilepsy who has tonic-clonic seizures. Which statement, if made by the caregiver, requires further teaching? A. "It is normal for a person to be sleepy after a seizure." B. "I should call 911 if breathing stops during the seizure." C. "The jerking movements may last for 30 to 40 seconds." D. "Objects should not be placed in the mouth during a seizure."

B. "I should call 911 if breathing stops during the seizure."Caregivers do not need to call an ambulance or send a person to the hospital after a single seizure unless the seizure is prolonged, another seizure immediately follows, or extensive injury has occurred. Altered breathing is a clinical manifestation of a tonic-clonic seizure. Contact emergency medical services (or call 911) if breathing stops for more than 30 seconds. No objects (e.g., oral airway, padded tongue blade) should be placed in the mouth. Lethargy is common in the postictal phase of a seizure. Jerking of the extremities occurs during the clonic phase of a tonic-clonic seizure. The clonic phase may last 30 to 40 seconds.

You're educating a patient about treatment options for Guillain-Barré Syndrome. Which statement by the patient requires you to re-educate the patient about treatment?* A. "Treatments available for this syndrome do not cure the condition but helps speed up recovery time." B. "Plasmapheresis or immunoglobin therapies are treatment options available for this syndrome but are most effective when given within 4 weeks of the onset of symptoms." C. "When I start plasmapheresis treatment a machine will filter my blood to remove the antibodies from my plasma that are attacking the myelin sheath." D. "Immunoglobulin therapy is where IV immunoglobulin from a donor is given to a patient to stop the antibodies that are damaging the nerves.

B. "Plasmapheresis or immunoglobin therapies are treatment options available for this syndrome but are most effective when given within 4 weeks of the onset of symptoms."

80 year old Mr. Rogers, together with his daughter, arrived at the med-surg unit for diagnostic confirmation and management of probable delirium. Which statement by the client's daughter best supports the diagnosis? A. "Maybe it's just caused by aging. This usually happens at his age." B. "The changes in his behavior came on so quickly! I wasn't sure what was happening." C. "Dad just didn't seem to know what he was doing. He has been forgetful for years." D. "Dad has always been so independent. He's lived alone for years since mom died."

B. "The changes in his behavior came on so quickly! I wasn't sure what was happening."

You're providing discharge teaching to a patient with peripheral arterial disease. Which statement by the patient requires you to re-educate the patient?* A. "It is important I quit smoking." B. "To prevent my feet and legs from getting too cold at night, I will use a heating pad. C. "A walking program would be beneficial in the treatment of my PAD." C. "I will avoid wearing tight socks or shoes."

B. "To prevent my feet and legs from getting too cold at night, I will use a heating pad.

The lowest fasting plasma glucose level suggestive of a diagnosis of DM is: A. 90mg/dl B. 115mg/dl C. 126mg/dl D. 180mg/dl

B. 126mg/dl

Which is a classic presentation of Guillain-Barré syndrome? A. Acute change in level of consciousness B. Ascending, symmetric paralysis C. Acute onset of paralysis in lower extremities D. Paresthesias in legs starting with feet and radiating to groin area

B. Ascending, symmetric paralysis Guillain-Barré syndrome is an acute, rapidly progressing polyneuritis with ascending, symmetric paralysis. The other options are not related to Guillain-Barré syndrome.

Your patient has returned from a peripheral artery bypass for the treatment of peripheral arterial disease. The nurse will make it PRIORITY to?* A. Assess the surgical site for excessive drainage B. Assess and grade lower extremity pulses bilaterally C. Apply compression stockings D. Elevate the lower extremity above heart level

B. Assess and grade lower extremity pulses bilaterally

A patient's potassium level is 3.0. Which foods would you encourage the patient to consume? A. Cheese, collard greens, and fish B. Avocados, strawberries, and potatoes C. Tofu, oatmeal, and peas D. Peanuts, bread, and corn

B. Avocados, strawberries, and potatoes

The nurse recognizes the presence of Cushing's triad in the patient with a. Increased pulse, irregular respiration, increased BP b. decreased pulse, irregular respiration, increased pulse pressure c. increased pulse, decreased respiration, increased pulse pressure d. decreased pulse, increased respiration, decreased systolic BP

B. Cushing's triad consists of three vital sign measures that reflect ICP and its effect on the medulla, the hypothalamus, the pons, and the thalamus. Because these structures are very deep, Cushing's triad is usually a late sign of ICP. The signs include an increasing systolic BP with a widening pulse pressure, a bradycardia with a full and bounding pulse, and irregular respirations.

The patient is diagnosed with Guillain-Barré syndrome and admitted to the inpatient unit from the emergency department. What is the most important nursing observation? A. Urine output B. Depth of respiration C. Bowel sounds D. Lower extremity strength

B. Depth of respiration The most serious complication is paralysis progressing to the nerves that innervate the thoracic area and causing respiratory failure. You must constantly monitor the respiratory system by checking respiratory rate and depth, forced vital capacity, and negative inspiratory force. The other options may be affected, but respiratory function is most important.

A 25-year-old male is admitted in sickle cell crisis. Which of the following interventions would be of highest priority for this client? A. Taking hourly blood pressures with mechanical cuff B. Encouraging fluid intake of at least 200mL per hour C. Position in high Fowler's with knee gatch raised D. Administering Tylenol as ordered

B. Encouraging fluid intake of at least 200mL per hour

A patient presents to the emergency department diaphoretic and confused with a mottled appearance to the skin. Upon further assessment, the nurse finds their BP is 90/60, HR is 120, and RR is 36. Based on these signs and symptoms, what is the priority nursing intervention? A. Administer a vasopressor B. Fluid resuscitation C. Obtain a CBC with differential D. Prepare to intubate the patient

B. Fluid resuscitation

The nurse would expect to find what clinical manifestation in a patient admitted with a left-sided stroke? A. Impulsivity B. Impaired speech C. Left-side neglect D. Short attention span

B. Impaired speech Clinical manifestations of left-sided brain damage include right hemiplegia, impaired speech/language, impaired right/left discrimination, and slow and cautious performance. Impulsivity, left-sided neglect, and short attention span are all manifestations of right-sided brain damage.

The blood bank notifies the nurse that the two units of blood ordered for an anemic patient are ready for pick up. Which action should the nurse take to prevent an adverse effect during this procedure? A. Immediately pick up both units of blood from the blood bank. B. Infuse the blood slowly for the first 15 minutes of the transfusion. C. Regulate the flow rate so that each unit takes at least 4 hours to transfuse. D. Set up the Y-tubing of the blood set with dextrose in water as the flush solution.

B. Infuse the blood slowly for the first 15 minutes of the transfusion. Because a transfusion reaction is more likely to occur at the beginning of a transfusion, the nurse should initially infuse the blood at a rate no faster than 2 mL/min and remain with the patient for the first 15 minutes after hanging a unit of blood. Only one unit of blood can be picked up at a time, must be infused within 4 hours, and cannot be hung with dextrose.

A patient has a potassium level of 2.0. What would you expect to be ordered for this patient? A. Potassium 30 meq IV push B. Infusion of Potassium intravenously C. An oral supplement of potassium D. Intramuscular injection of Potassium

B. Infusion of Potassium intravenously NEVER PUSH POTASSIUM!!!!

A patient who has sickle cell disease has developed cellulitis above the left ankle. What is the nurse's priority for this patient? A. Start IV fluids. B. Maintain oxygenation. C. Maintain distal warmth. D. Check peripheral pulses.

B. Maintain oxygenation. Maintaining oxygenation is a priority as sickling episodes are frequently triggered by low oxygen tension in the blood which is commonly caused by an infection. Antibiotics to treat cellulitis, pain control, and fluids to reduce blood viscosity will also be used, but oxygenation is the priority.

Which of the below patients are at the highest risk for neutropenia? A. Patient with cataract B. Patient who has leukemia undergoing chemotherapy. C. Patient who has been vomiting for 24 hours D. Patient who has prenicious anemia.

B. Patient who has leukemia undergoing chemotherapy.

Which characteristic of a patient's recent seizure is consistent with a focal impaired seizure? A. The patient lost consciousness during the seizure. B. The seizure involved lip smacking and repetitive movements. C. The patient fell to the ground and became stiff for 20 seconds. D. The etiology of the seizure involved both sides of the patient's brain.

B. The seizure involved lip smacking and repetitive movements. The most common complex focal seizure involves lip smacking and automatisms (repetitive movements that may not be appropriate). Loss of consciousness, bilateral brain involvement, and a tonic phase are associated with generalized seizure activity.

A nurse cares for a client who is prescribed vasopressin (DDAVP) for diabetes insipidus. Which assessment findings indicate a therapeutic response to this therapy? (SATA) a. Urine output is increased. b. Urine output is decreased. c. Specific gravity is increased. d. Specific gravity is decreased. e. Urine osmolality is increased. f. Urine osmolality is decreased.

BCE Diabetes insipidus causes urine output to be greatly increased, with a low urine osmolality, as evidenced by a low specific gravity. Effective treatment results in decreased urine output that is more concentrated, as evidenced by an increased specific gravity

The nurse has a prescription for a patient to receive prednisone (Deltasone) to treat contact dermatitis. The nurse would question this prescription for this patient with what condition? A. Asthma B. Multiple sclerosis C. Acquired immune deficiency syndrome (AIDS) D. Chronic obstructive pulmonary disease

C. Acquired immune deficiency syndrome (AIDS)

The MOST common cause of the peripheral arterial disease is?* A. Diabetes B. Deep vein thrombosis C. Atherosclerosis D. Pregnancy

C. Atherosclerosis

A patient is admitted with complaints of palpations, excessive sweating, and unable to tolerate heat. In addition, the patient voices concern about how her appearance has changed over the past year. The patient presents with protruding eyeballs and pretibial myxedema on the legs and feet. Which of the following is the likely cause of the patient's signs and symptoms? A. Thyroiditis B. Deficiency of iodine consumption C. Grave's Disease D. Hypothyroidism

C. Grave's Disease

In promoting health maintenance for prevention of strokes, the nurse understands that the highest risk for the most common type of stroke is present in a. African Americans b. women who smoke c.individuals with hypertension and diabetes d. those who are obese with high dietary fat intake

C: Individuals with hypertension and diabetes- The highest risk factors for thrombotic stroke are hypertension and diabetes. African Americans have a higher risk for stroke than do white persons but probably because they have a greater incidence of hypertension. Factors such as obesity, diet high in saturated fats and cholesterol, cigarette smoking, and excessive alcohol use are also risk factors but carry less risk than hypertension.

Risk factors for type 2 diabetes include all of the following except: a. Advanced age b. Obesity c. Smoking d. Physical inactivity

C: smoking. Additional risk factors for type 2 diabetes are a family history of diabetes, impaired glucose metabolism, history of gestational diabetes, and race/ethnicity. African-Americans, Hispanics/Latinos, Asian Americans, Native Hawaiians, Pacific Islanders, and Native Americans are at greater risk of developing diabetes than whites.

Indication for Osmotic Diuretics

Cerebral Edema ( increased ICP) Intraocular pressure (fluid pressure inside the eye) - Glaucoma Anuria/oliguria (unable to pass urine/passes less urine) AKI Severe Burns

A nurse provides instructions to a client regarding the administration of the prednisone and instructs the client that the best time to take the medication is during? A. Before breakfast B. After breakfast C. Evening D. Before bedtime

Correct Answer: B. After breakfast Prednisone is a corticosteroid (glucocorticoids) and It should be administered early in the morning because it helps in decreasing the risk for adrenal insufficiency and stimulates the burst of glucocorticoids released naturally by the adrenal glands each morning. Prednisone can cause gastric upset and should always be taken with a meal.

A male client with type 1 diabetes mellitus asks the nurse about taking an oral antidiabetic agent. Nurse Jack explains that these medications are only effective if the client: A. Prefers to take insulin orally. B. Has type 2 diabetes. C. Has type 1 diabetes. D. Is pregnant and has type 2 diabetes.

Correct Answer: B. Has type 2 diabetes. Oral antidiabetic agents are only effective in adult clients with type 2 diabetes. Antidiabetic drugs (with the exception of insulin are all pharmacological agents that have been approved for hyperglycemic treatment in type 2 diabetes mellitus (DM). If lifestyle modifications (weight loss, dietary modification, and exercise) do not sufficiently reduce A1C levels (target level: ? 7%), pharmacological treatment with antidiabetic drugs should be initiated.

Shortly after admission to an acute care facility, a male client with a seizure disorder develops status epilepticus. The physician orders diazepam (Valium) 10 mg I.V. stat. How soon can the nurse administer the second dose of diazepam, if needed and prescribed? A. In 30 to 45 seconds B. In 10 to 15 minutes C. In 30 to 45 minutes D. In 1 to 2 hours

Correct Answer: B. In 10 to 15 minutes When used to treat status epilepticus, diazepam may be given every 10 to 15 minutes, as needed, to a maximum dose of 30 mg. The nurse can repeat the regimen in 2 to 4 hours, if necessary, but the total dose shouldn't exceed 100 mg in 24 hours. It is crucial to monitor respiratory and cardiovascular status, blood pressure, heart rate, and symptoms of anxiety in patients taking diazepam.

The nurse is caring for the client with increased intracranial pressure. The nurse would note which of the following trends in vital signs if the ICP is rising? A. Increasing temperature, increasing pulse, increasing respirations, decreasing blood pressure. B. Increasing temperature, decreasing pulse, decreasing respirations, increasing blood pressure. C. Decreasing temperature, decreasing pulse, increasing respirations, decreasing blood pressure. D. Decreasing temperature, increasing pulse, decreasing respirations, increasing blood pressure.

Correct Answer: B. Increasing temperature, decreasing pulse, decreasing respirations, increasing blood pressure. A change in vital signs may be a late sign of increased intracranial pressure. Trends include increasing temperature and blood pressure and decreasing pulse and respirations. Respiratory irregularities also may arise. Cushing triad is a clinical syndrome consisting of hypertension, bradycardia and irregular respiration and is a sign of impending brain herniation. This occurs when the ICP is too high the elevation of blood pressure is a reflex mechanism to maintain CPP.

During preoperative teaching for a female client who will undergo subtotal thyroidectomy, the nurse should include which statement? A. "The head of your bed must remain flat for 24 hours after surgery." B. "You should avoid deep breathing and coughing after surgery." C. "You won't be able to swallow for the first day or two." D. "You must avoid hyperextending your neck after surgery.

Correct Answer: D. "You must avoid hyperextending your neck after surgery." To prevent undue pressure on the surgical incision after subtotal thyroidectomy, the nurse should advise the client to avoid hyperextending the neck. Caution patient to avoid bending neck; support head with pillows. Reduces the likelihood of tension on the surgical wound.

Nurse Kate is providing dietary instructions to a male client with hypoglycemia. To control hypoglycemic episodes, the nurse should recommend: A. Increasing saturated fat intake and fasting in the afternoon. B. Increasing intake of vitamins B and D and taking iron supplements. C. Eating a candy bar if lightheadedness occurs. D. Consuming a low-carbohydrate, high protein diet and avoiding fasting.

Correct Answer: D. Consuming a low-carbohydrate, high protein diet and avoiding fasting. To control hypoglycemic episodes, the nurse should instruct the client to consume a low-carbohydrate, high protein diet, avoid fasting and avoid simple sugars. Increasing saturated fat intake and increasing vitamin supplementation wouldn't help control hypoglycemia. A person experiencing a minor case of low blood sugar can consume 15-20 grams (g) of fast-acting carbohydrates, such as a small glass of fruit juice or a few crackers

Which nursing diagnosis takes highest priority for a female client with hyperthyroidism? A. Risk for imbalanced nutrition: More than body requirements related to thyroid hormone excess. B. Risk for impaired skin integrity related to edema, skin fragility, and poor wound healing. C. Body image disturbance related to weight gain and edema. D. Imbalanced nutrition: Less than body requirements related to thyroid hormone excess.

Correct Answer: D. Imbalanced nutrition: Less than body requirements related to thyroid hormone excess In the client with hyperthyroidism, excessive thyroid hormone production leads to hypermetabolism and increased nutrient metabolism. These conditions may result in a negative nitrogen balance, increased protein synthesis and breakdown, decreased glucose tolerance, and fat mobilization and depletion. This puts the client at risk for marked nutrient and calorie deficiency, making Imbalanced nutrition: Less than body requirements the most important nursing diagnosis. Options B and C may be appropriate for a client with hypothyroidism, which slows the metabolic rate.

A male client has an impairment of cranial nerve II. Specific to this impairment, the nurse would plan to do which of the following to ensure the client to ensure client safety? A. Speak loudly to the client. B. Test the temperature of the shower water. C. Check the temperature of the food on the delivery tray. D. Provide a clear path for ambulation without obstacles.

Correct Answer: D. Provide a clear path for ambulation without obstacles. Cranial nerve II is the optic nerve, which governs vision. The nurse can provide safety for the visually impaired client by clearing the path of obstacles when ambulating. Compromise of the optic nerve results in visual field defects and/or visual loss. The type of visual field defect depends on which region of the optic pathway is disrupted.

A male client with a tentative diagnosis of hyperosmolar hyperglycemic nonketotic syndrome (HHNS) has a history of type 2 diabetes that is being controlled with an oral diabetic agent, tolazamide (Tolinase). Which of the following is the most important laboratory test for confirming this disorder? A. Serum potassium level B. Serum sodium level C. Arterial blood gas (ABG) values D. Serum osmolarity

Correct Answer: D. Serum osmolarity Serum osmolarity is the most important test for confirming HHNS; it's also used to guide treatment strategies and determine evaluation criteria. A client with HHNS typically has a serum osmolarity of more than 350 mOsm/L. The serum osmolality is determined by the formula 2Na + Glucose /18 + BUN / 2.8. The resultant hyperglycemia increases the serum osmolarity to a significant degree. The glucose level in HHS is usually above 600 mg/dL. Hyperglycemia also creates an increase in the osmotic gradient with free water drawn out from the extravascular space from the increased osmotic gradient

Which statement by the client supports the diagnosis of myasthenia gravis (MG)? A "I have weakness and fatigue in my feet and legs." B "My eyelids droop, and I see double everything." C "I get chest pain and faint after I walk in the hall." D "I gained 3 pounds this week, and I am spitting up pink frothy sputum."

Correct answer B: These are ocular signs/symptoms of MG. Ptosis is drooping of the eyelid, and diplopia is unilateral or bilateral double vision. Weakness and fatigue of upper body muscle occur with MG.

To maintain a positive nitrogen balance in a major burn, the patient must: a. eat a high-protein, low-fat, high-carbohydrate diet b. increase normal caloric intake by about three times c. eat at least 1500 calories/day in small, frequent meals d. eat rice and whole wheat for the chemical effect on nitrogen balance.

Correct answer: a Rationale: The patient should be encouraged to eat high-protein, high-carbohydrate foods to meet increased caloric needs. Massive catabolism can occur and is characterized by protein breakdown and increased gluconeogenesis. Failure to supply adequate calories and protein leads to malnutrition and delays in healing.

The nurse is caring for a client who is diagnosed with delirium. The​ client's spouse is upset and tells the​ nurse, "I​ don't know how I am going to be able to care for my​ spouse, who can no longer think​ clearly." How should the nurse​ respond? ​A. "I'm sure that you will be​ fine." B. ​"Your spouse's condition will continue to​ deteriorate; maybe you should look into nursing​ homes." ​C. "Delirium is usually more prevalent in​ younger, rather than​ older, clients." D. ​"It is important for you to know​ that, generally, delirium is​ reversible."

D

Which option indicates a sign of Cushing's triad, an indication of increased intracranial pressure (ICP)? A. Heart rate increases from 90 to 110 beats/minute B. Kussmaul respirations C. Temperature over 100.4° F (38° C) D. Heart rate decreases from 75 to 55 beats/minute

D Cushing's triad is systolic hypertension with a widening pulse pressure, bradycardia with a full and bounding pulse, and slowed respirations. The rise in blood pressure is an attempt to maintain cerebral perfusion, and it is a neurologic emergency because decompensation is imminent. The other options are not part of Cushing's triad.

A nurse is caring for an alert client with diabetes who is receiving an 1800-calorie diabetic diet. The client's blood glucose level is 30 mg/dL (3 mmol/L). The primary healthcare provider's protocol calls for treatment of hypoglycemia with 15 g of a simple carbohydrate. What should the nurse do first? A. Provide 12 ounces (360 mL) of nondiet soda B. Give 25 mL dextrose 50% by slow intravenous (IV) push C. Have the client drink 8 ounces (240 mL) of fruit juice D. Ask the client to ingest one tube of glucose gel

D - Ask the client to ingest one tube of glucose gelOne tube of glucose gel contains 15 g of carbohydrate and is the most appropriate intervention in this situation. Providing 12 ounces (360 mL) of nondiet soda is too much carbohydrate; 4 to 6 ounces (120 to 180 mL) is adequate. Administering dextrose by IV push is not appropriate for an alert client who is able to eat and drink. Having the client drink 8 ounces (240 mL) of fruit juice is too much carbohydrate; 4 to 6 ounces (120 to 180 mL) is adequate.

A client who is receiving chemotherapy for leukemia has developed neutropenia. Which of these foods should the client avoid? A Stewed potatoes B Wheat bread C Steak D Fresh celery

D Fresh Celery This answer is correct because the neutropenic client needs to avoid raw fruits & vegetables. The reason for taking this precaution is to minimize the client's risk for infection, due to his neutropenia.

A patient with type 1 diabetes has received diet instruction as part of the treatment plan. The nurse determines a need for additional instruction when the patient says, a. "I may have an occasional alcoholic drink if I include it in my meal plan." b. "I will need a bedtime snack because I take an evening dose of NPH insulin." c. "I will eat meals as scheduled, even if I am not hungry, to prevent hypoglycemia." d. "I may eat whatever I want, as long as I use enough insulin to cover the calories."

D Rationale: Most patients with type 1 diabetes need to plan diet choices very carefully. Patients who are using intensified insulin therapy have considerable flexibility in diet choices but still should restrict dietary intake of items such as fat, protein, and alcohol. The other patient statements are correct and indicate good understanding of the diet instruction.

The nurse is caring for a client recovering from disseminated intravascular coagulation (DIC). Which safety measure should the nurse implement? A avoid using an electric razor B educate patient to avoid freshly cracked pepper C demonstrates the use of a cane D educate the patient to use a soft-bristled toothbrush

D soft-bristled toothbrush This answer is correct because using a soft, manual toothbrush is a safety measure to help reduce bleeding in the gums and mouth. Other hygiene considerations for this client include using electric razors rather than razor blades and soft washcloths.This answer is correct because using a soft, manual toothbrush is a safety measure to help reduce bleeding in the gums and mouth. Other hygiene considerations for this client include using electric razors rather than razor blades and soft washcloths.

A patient comes to the clinic complaining of a red rash of small, fluid-filled blisters and is suspected of having herpes zoster. What presentation is most consistent with herpes zoster? A) Grouped vesicles occurring on lips and oral mucous membranes B) Grouped vesicles occurring on the genitalia C) Rough, fresh, or gray skin protrusions D) Grouped vesicles in linear patches along a dermatome

D) Grouped vesicles in linear patches along a dermatome

hile performing an initial assessment of a patient admitted with appendicitis, the nurse observes an elevated blue-black lesion on the patient's ear. The nurse knows that this lesion is consistent with what type of skin cancer? A) Basal cell carcinoma B) Squamous cell carcinoma C) Dermatofibroma D) Malignant melanoma

D) Malignant melanoma

A patient with ICP monitoring has pressure of 12 mm Hg. The nurse understand that this pressure reflects a. a severe decrease in cerebral perfusion pressure b. an alteration in the production of CSF c. the loss of autoregulatory control of ICP d. a normal balance between brain tissue, blood, and CSF

D. A normal balance between brain tissue, blood, and CSF- normal is 10- 15 mm Hg

Metabolic and nutritional needs of the patient with increased ICP are best met with a. enteral feedings that are low in sodium b. the simple glucose available in D5W IV solutions c. a fluid restriction that promotes a moderate dehydration d. balanced, essential nutrition in a form that the patient can tolerate

D. Balanced, essential nutrition in a form that the patient can tolerate= A patient with increased ICP is in a hypermetabolic and hypercatabolic state and needs adequate glucose to maintain fuel for the brain and other nutrients to meet metabolic needs. Malnutrition promotes cerebral edema, and if a patient cannot take oral nutrition, other means of providing nutrition should be used, such as tube feedings or parenteral nutrition. Glucose alone is not adequate to meet nutritional requirements, and 5% dextrose solutions may increase cerebral edema by lowering serum osmolarity. Patients should remain in a normovolemic fluid state with close monitoring of clinical factors such as urine output, fluid intake, serum and urine osmolality, serum electrolytes, and insensible losses.

The male client with sickle cell anemia comes to the emergency room with a temperature of 101.4 F and tells the nurse that he is having a sickle cell crisis. Which diagnostic test should the nurse anticipate the emergency room doctor ordering for the client? -- A. Spinal tap. -- B. Hemoglobin electrophoresis. -- C. Sickle-turbidity test (Sickledex). -- D. Blood cultures

D. Blood cultures Fever indicates infection. Treating cause is a priority in Sickle cell anemia.

When assessing the body function of a patient with increased ICP, the nurse should initially assess a. corneal reflex testing b. extremity strength testing c. pupillary reaction to light d. circulatory and respiratory status

D. Circulatory and respiratory status- Of the body functions that should be assessed in an unconscious patient, cardiopulmonary status is the most vital function and gives priorities to the ABCs (airway, breathing, and circulation)

Cranial nerve __________ moves the eyeballs/eyelids and adjusts the pupils and lens of the eye. A. VII B. X c. I D. III

D. III - Oculomotor nerve

A patient with pernicious anemia is ordered to receive supplementary Vitamin B12. What is the best route to administer this medication for patients with this disorder? A. Intravenous B. Orally C. Through a central line D. Intramuscular

D. Intramuscular

The priority intervention in the emergency department for the patient with a stroke is a. intravenous fluid replacement b. administration of osmotic diuretics to reduce cerebral edema c. initiation of hypothermia to decrease the oxygen needs of the brain d. maintenance of respiratory function with a patent airway and oxygen administration

D: Maintenance of respiratory function with a patent airway and oxygen administration- the first priority in acute management of the patient with a stroke is preservation of life. Because the patient with a stroke may be unconscious or have a reduced gag reflex, it is most important to maintain a patent airway for the patient and provide oxygen if respiratory effort is impaired. IV fluid replacement, treatment with osmotic diuretics, and perhaps hypothermia may be used for further treatment.

The nurse should assess a client diagnosed with delirium for which​ symptom? (Select all that​ apply.) Disorganized thoughts Altered sleep patterns Limited attention span Ability to answer questions Ability to​ read, write, and understand speech

Disorganized thoughts Altered sleep patterns Limited attention span

The most common signs and symptoms of leukemia related to bone marrow involvement are which of the following? A. Petechiae, fever, fatigue B. Headache, papilledema, irritability C. Muscle wasting, weight loss, fatigue D. Decreased intracranial pressure, psychosis, confusion

Answer A is Correct. Signs of infiltration of the bone marrow are petechiae from lowered platelet count, fever related to infection from the depressed number of effective leukocytes, and fatigue from the anemia

You're providing free education to a local community group about the signs and symptoms of Parkinson's Disease. Select all the signs and symptoms a patient could experience with this disease:* A. Increased Salivation B. Loss of smell C. Constipation D. Tremors with purposeful movement E. Shuffling of gait F. Freezing of extremities G. Euphoria H. Coordination issues

The answers are B, C, E, F, H. These are all signs and symptoms experienced with PD (they vary among patients). There is NOT increased salivation (although drooling occurs...this is due to the decreased ability to swallow. There are tremors at REST (not movement) along with depression rather the euphoria.

Peripheral arterial disease leads to a decrease in rich oxygenated blood being delivered to the lower extremities, which leads to ischemia and necrosis of skin tissue.* True False

True

The vestibulocochlear nerve (cranial nerve VIII) is responsible for auditory? True False

True

Cranial nerve II's function is eyesight. True False

True - Cranial nerve II is the optic nerve.

During history taking of a client admitted with newly diagnosed Hodgkin's disease, which of the following would the nurse expect the client to report? "a) weight gain B) night sweats C) Severe lymph node pain D) Headache with minor visual changes

During history taking of a client admitted with newly diagnosed Hodgkin's disease, which of the following would the nurse expect the client to report?"a) weight gainB) night sweatsC) Severe lymph node painD) Headache with minor visual changes

What is the indication for loop diuretic?

Edema often due to congestive heart failure or chronic kidney disease. Hypertension (high blood pressure) Hyperkalemia (increased potassium level) Liver cirrhosis. SIADH

True or False: Guillain-Barré Syndrome occurs when the body's immune system attacks the myelin sheath on the nerves in the central nervous system.* True False

FALSE: Guillain-Barré Syndrome is an autoimmune neuro condition where the immune system attacks the nerves (myelin sheath) in the PERIPHERAL NERVOUS SYSTEM and cranial nerves. This condition does NOT occur in the central nervous system (CNS).

The optic nerve is known as cranial nerve X (ten). True False

False Optic nerve is cranial nerve II Vagus nerve is cranial nerve X

The parasympathetic nervous system loses the ability to stimulate nerve impulses in patients who are experiencing neurogenic shock. This leads to hemodynamic changes. True False

False - The SNS loses the ability to stimulate the nerve impulses in patients who are experiencing neurogentic shock. This leads to hemodynamic changes such as peripheral pooling, massive vasoconstriction, decreased preload, decreased afterload, decreased cardiac output.

A patient with nasogastric suctioning is experiencing diarrhea. The patient is ordered a morning dose of Lasix 20mg IV. Patient's potassium level is 3.0. What is your next nursing intervention? A. Hold the dose of Lasix and notify the doctor for further orders B. Administered the Lasix and notify the doctor for further orders C. Turn off the nasogastric suctioning and administered a laxative D. No intervention is need the potassium level is within normal range

Hold the dose of Lasix and notify the doctor for further orders

What is the indication for thiazide duretic?

Hypertension Chronic Heart Failure

With peripheral arterial insufficiency, leg pain during rest can be reduced by: 1. Elevating the limb above the heart level 2. Lowering the limb so that it is independent 3. Massaging the limb after application of cold compress 4. Placing the limb in a plane horizontal to the body

Lowing the limb so it is dependent

OK... I THINK I NEED TO GET UP AND WIGGLE.. AND SING... AND GET A GOOD STIFF DRINK. BECAUSE YOU KNOW ALMOST EVERY CONDITION WE HAVE STUDIED HAS SAID IT IS OK TO HAVE ONE DRINK ... LOL.

OH YEAH... YOU CAN HAVE ONE TOO!

Saying hi to my fellow classmates that I know are using these cards! Hi Amanda, Amanda, Chrissy, Lisa, Shanta! Keep up the good work everyone! Baby Nurses semester 2 (or 3) almost done.

OK ... get up and wiggle. Get a drink and sing. Good to stimulate your vagus nerve. That is which nerve now.... that is right... Cranial nerve X.

SAY WHAT! 300 QUESTIONS AND STILL GOING.... YOU ARE AMAZING! LOOK OUT WORLD THIS BABY NURSE MEANS BUSINESS! KEEP UP THE GOOD WORK!

OOOHHH YOU FLIPPED THE CARD TO SEE WHAT I WAS REALLY THINKING... WELL FLIP THE CARD BACK OVER BECAUSE THAT IS WHAT I AM REALLY THINKING YOU ROCK!

https://www.youtube.com/watch?v=1k8craCGpgs

Sing baby sing! This will stimulate which cranial nerve?

A young woman makes an appointment to see a physician at the clinic. She complains of tiredness, weight gain, muscle aches and pains, and constipation. The physician will likely order: 1. T3 and T4 serum level laboratory tests. 2. glucose tolerance test. 3. cerebral computed tomography (CT) scan. 4. adrenocortical stimulating test.

T3 and T4 serum level laboratory tests. These complaints are strongly suggestive of thyroid disorder; T3 and T4 laboratory tests are the most useful diagnostic tests.

The nurse is ordered to administer Lorazepam to a patient experiencing status epilepticus. As a precautionary measure, the nurse will also have what reversal agent on standby? A. Narcan B. Flumazenil C. Calcium Chloride D. Idarucizumab

The answer is B. Flumazenil is the reversal agent for Lorazepam, which is a benzodiazepine.

You're providing care to a patient in anaphylactic shock. What is NOT a typical medical treatment for this condition, and if ordered the nurse should ask for an order clarification? A. IV Diphenhydramine B. Epinephrine C. Corticosteroids D. Isotonic intravenous fluids E. IV Furosemide

The answer is E. Furosemide is a loop-diuretic. This medication removes extra fluid from the blood volume. This is NOT used as treatment in anaphylactic shock. Patients with this condition actually need fluids because of the shift of fluid from the intravascular space to the interstitial space. All the other medications may be ordered for this condition depending on the patient's condition.

Before a blood transfusion you educate the patient to immediately report which of the following signs and symptoms during the blood transfusion that could represent a transfusion reaction:* A. Sweating B. Chills C. Hives D. Poikilothermia E. Tinnitus F. Headache G. Back pain H. Pruritus I. Paresthesia J. Shortness of Breath K. Nausea

The answers are A, B, C, F, G, H, J, and K. As the nurse you want to educate the patient to report signs and symptoms associated with blood transfusion reactions, which would include: sweating, chills, hives, headache, back pain, pruritus (itching), shortness of breath, and nausea.

A patient is receiving treatment for a complete spinal cord injury at T4. As the nurse you know to educate the patient on the signs and symptoms of autonomic dysreflexia What signs and symptoms will you educate the patient about? Select all that apply:* A. Headache B. Low blood glucose C. Sweating D. Flushed below site of injury E. Pale and cool above site of injury F. Hypertension G. Slow heart rate H. Stuffy nose

The answers are A, C, F, G and H. All of these are signs and symptoms of autonomic dysreflexia. The patient will have flushing above site of injury due to vasodilation from parasympathetic activity, BUT will be pale and cool below site of injury due to vasoconstriction occurring below the site of injury for the sympathetic response reflex.

You have a patient who has a brain tumor and is at risk for seizures. In the patient's plan of care you incorporate seizure precautions. Select below all the proper steps to take in initiating seizure precautions: A. Oxygen and suction at bedside B. Bed in highest position C. Remove all pillows from the patient's head D. Have restraints on stand-by E. Padded bed rails F. Remove restrictive objects or clothing from patient's body G. IV access

The answers are A, E, F, and G. The bed needs to be in the LOWEST position possible, a pillow should be underneath the patient's head to protect it from injury, AVOID using restraints (this can cause musculoskeletal damage).

You're providing education to a group of nursing students about nephrotic syndrome. A student describes the signs and symptoms of this condition. Which signs and symptoms verbalized by the student require you to re-educate the student about this topic? Select-all-that-apply: A. Slight proteinuria B. Hypoalbuminemia C. Edema D. Hyperlipidemia E. Tea-colored urine F. Hypertension

The answers are A, E, and F. The patient with nephrotic syndrome will experience massive proteinuria (not slight) along with low albumin in the blood (hypoalbuminemia), edema, and high cholesterol and triglyceride levels. It is not common for the patient to experience tea-colored urine or hypertension (rare) this is very common with acute glomerulonephritis.

You're working on a neuro unit. Which of your patients below are at risk for developing neurogenic shock? Select all that apply:* A. A 36-year-old with a spinal cord injury at L4. B. A 42-year-old who has spinal anesthesia. C. A 25-year-old with a spinal cord injury above T6. D. A 55-year-old patient who is reporting seeing green halos while taking Digoxin.

The answers are B and C. Any patient who has had a cervical or upper thoracic (above T6) spinal cord injury, receiving spinal anesthesia, or taking drugs that affect the autonomic or sympathetic nervous system is at risk for developing neurogenic shock.

The mother of a child, who was recently diagnosed with nephrotic syndrome, asks how she can identify early signs that her child is experiencing a relapse with the condition. You would tell her to monitor the child for the following: Select-all-that-apply: A. Weight loss B. Protein in the urine using an over-the-counter kit C. Tea-colored urine D. Swelling in the legs, hands, face, or abdomen

The answers are B and D. The patient will NOT experience weight loss but weight GAIN as a sign of relapse with this condition. In addition, the urine will appear dark and foamy. Tea-colored urine indicates there is blood in the urine, which is NOT common with nephrotic syndrome.

You're providing an in-service to a group of new nurse graduates on the causes of autonomic dysreflexia. Select all the most common causes you will discuss during the in-service:* A. Hypoglycemia B. Distended bladder C. Sacral pressure injury D. Fecal impaction E. Urinary tract infection

The answers are B, C, D, and E. Anything that can cause an irritating stimulus below the site of the spinal injury (T6 or higher) can lead to autonomic dysreflexia, which causes an exaggerated sympathetic reflex response and the parasympathetic system is unable to oppose it. This will lead to severe hypertension. The most common cause of AD is a bladder issue (full/distended bladder, urinary tract infection etc). Other common causes are due to a bowel issue like fecal impaction or skin break down (pressure injury/ulcer, cut, infection etc.).

Which nursing diagnosis is a priority in the care of a patient with myasthenia gravis (MG)? a. Acute confusion b. Bowel incontinence c. Activity intolerance d. Disturbed sleep pattern

c. Activity intolerance The primary feature of MG is fluctuating weakness of skeletal muscle. Bowel incontinence and confusion are unlikely signs of MG. Although sleep disturbance is likely, activity intolerance is of primary concern.

While obtaining subjective assessment date from a patient with hypertension, the nurse recognizes that a modifiable risk factor for the development of hypertension is: a. a low-calcium diet b. excessive alcohol consumption c. a family history of hypertension d. consumption of a high-protein diet

b. excessive alcohol consumption

When hydrocortisone use is discontinued, the nurse must recognize the possibility of what side effect, if this drug is stopped abruptly? a.) Development of myxedema b.) Circulatory collapse c.) Development of Cushing's syndrome d.) Development of diabetes insipidus

b.) Circulatory collapse

A client has been diagnosed to have chronic renal failure. Sodium polystyrene sulfonate (exchange resin kayexalate) is prescribed. The action of the medication is that it releases a) bicarbonate in exchange for primarily sodium ions b) sodium ions in exchange for primarily bicarbonate ions c) sodium ions in exchange for primarily potassium ions d) potassium ions in exchange for primarily sodium ions

c) sodium ions in exchange for primarily potassium ions

In providing care for patients with chronic, progressive neurologic disease, what is the major goal of treatment that the nurse works toward? a. Meet the patient's personal care needs. b. Return the patient to normal neurologic function. c. Maximize neurologic functioning for as long as possible. d. Prevent the development of additional chronic diseases.

c. Many chronic neurologic diseases involve progressive deterioration in physical or mental capabilities and have no cure, with devastating results for patients and families. Health care providers can only attempt to alleviate physical symptoms, prevent complications, and assist patients in maximizing function and self-care abilities for as long as possible.

A male patient with a diagnosis of Parkinson's disease (PD) is admitted to a long-term care facility. Which action should the health care team take to promote adequate nutrition for this patient? a. Provide multivitamins with each meal. b. Provide a diet that is low in complex carbohydrates and high in protein. c. Provide small, frequent meals throughout the day that are easy to chew and swallow. d. Provide the patient with a minced or pureed diet that is high in potassium and low in sodium.

c. Provide small, frequent meals throughout the day that are easy to chew and swallow. Nutritional support is a priority in the care of individuals with PD. Patients may benefit from smaller, more frequent meals that are easy to chew and swallow. Multivitamins are not necessary at each meal. Vitamin and protein intake must be monitored to prevent interactions with medications. Introducing a minced or pureed diet is likely premature, and a low carbohydrate diet is not indicated.

Which observation of the patient made by the nurse is most indicative of Parkinson's disease? a. Large, embellished handwriting b. Weakness of one leg resulting in a limping walk c. Difficulty rising from a chair and beginning to walk d. Onset of muscle spasms occurring with voluntary movement

c. The bradykinesia of PD prevents automatic movements and activities such as beginning to walk, rising from a chair, or even swallowing saliva cannot be executed unless they are consciously willed. Handwriting is affected by the tremor and results in the writing trailing off at the end of words. Specific limb weakness and muscle spasms are not characteristic of PD.

After the diagnosis of disseminated intravascular coagulation (DIC), what is the first priority of collaborative care? a. Administer heparin. b. Administer whole blood. c. Treat the causative problem. d. Administer fresh frozen plasma.

c. Treating the underlying cause of DIC will interrupt the abnormal response of the clotting cascade and reverse the DIC. Blood product administration occurs based on the specific component deficiencies and is reserved for patients with life-threatening hemorrhage. Heparin will be administered if the manifestations of thrombosis are present and the benefit of reducing clotting outweighs the risk of further bleeding.

The nurse is caring for a child with systemic lupus erythematosus that has a platelet count of 47,000. Which assessment findings may be consistent with this laboratory value? a. spoon shaped nails b. fever and cloudy urine c. petechia and epitaxis d. loss of balance and difficulty sleeping

c. petechia and epitaxis

Antidote for warfarin overdose a. protamine zinc insulin b. protamine sulfate c. vitamin K d. warfarin

c. vitamin K

Information provided by the patient that would help differentiate a hemorrhagic stroke from a thrombotic stroke includes a. sensory disturbance. b. a history of hypertension. c. presence of motor weakness. d. sudden onset of severe headache.

d Rationale: A hemorrhagic stroke usually causes a sudden onset of symptoms, which include neurologic deficits, headache, nausea, vomiting, decreased level of consciousness, and hypertension. Ischemic stroke symptoms may progress in the first 72 hours as infarction and cerebral edema increase.

Which of the following may be included in the diet of the client with chronic renal failure? a) orange slices b) watermelon slices c) cantaloupe slices d) apple slices

d) apple slices the client with renal failure should be given low potassium diet because of hyperkalemia. Apple contains very little potassium. So, it can be given to the client.

The nurse is caring for a patient with partial- and full-thickness burns to 65% of the body. When planning nutritional interventions for this patient, what dietary choices should the nurse implement? a. full liquids only b. whatever the patient requests c. high-protein and low sodium foods d. high calorie and high protein foods

d. A hypermetabolic state occurs proportional to the size of the burn area. Massive catabolism can occur and is characterized by protein breakdown and increased gluconeogenesis. Caloric needs are often in the 5000-kcal range. Failure to supply adequate calories and protein leads to malnutrition and delayed healing.

The client is due to receive Kayexalate for complaints of nausea, vomiting, abdominal cramps, short QT interval, weakness, and oliguria. The nurse is aware that this drug is used to treat which imbalance? a. Hypocalcemia b. Severe Hypercalcemia c. Hypokalemia The client is due to receive Kayexalate for complaints of nausea, vomiting, abdominal cramps, short QT interval, weakness, and oliguria. The nurse is aware that this drug is used to treat which imbalance? a. Hypocalcemia b. Severe Hypercalcemia c. Hypokalemia d. Severe Hyperkalemia

d. Severe Hyperkalemia

What is the most reliable method of detecting renal transplant rejection? 1) Ultrasonography 2) KUB X-ray 3) BUN and creatinine evaluation 4) Renal biopsy

4) Renal biopsy

The patient with diabetes mellitus has had a right-sided stroke. Which nursing intervention should the nurse plan to provide for this patient related to expected manifestations of this stroke? A. Safety measures B. Patience with communication C. Mobility assistance on the right side D. Place food in the left side of patient's mouth

A. Safety measures A patient with a right-sided stroke has spatial-perceptual deficits, tends to minimize problems, has a short attention span, is impulsive, and may have impaired judgment. Safety is the biggest concern for this patient. Hemiplegia occurs on the left side of this patient's body. The patient with a left-sided stroke has hemiplegia on the right, is more likely to have communication problems, and needs mobility assistance on the right side with food placed on the left side if the patient needs to be fed after a swallow evaluation has taken place.

The nurse suspects that a patient who has severe sepsis now has disseminated intravascular coagulation​ (DIC). Which​ finding, if​ observed, helps confirm this​ suspicion? A. Petechiae B. Polyuria C. Clear breath sounds D. Bradycardia

A

A client has been newly diagnosed with hypothyroidism and will take levothyroxine (Synthroid) 50 mcg/day by mouth. As part of the teaching plan, the nurse emphasizes that this medication: A) not to skip a dose, or change brands B) May decrease the client's energy level C) Must be stored in a dark container D) Will decrease the client's heart rate

A not to skip a dose, or change brands

The nurse is monitoring a patient for increased ICP following a head injury. Which of the following manifestations indicate an increased ICP (select all that apply) a. fever b. oriented to name only c. narrowing pulse pressure d. dilated right pupil > left pupil e. decorticate posturing to painful stimulus

A, B, D, E- The first sign of increased ICP is a change in LOC. Other manifestations are dilated ipsilateral pupil, changes in motor response such as posturing, and fever, which may indicate pressure on the hypothalamus. Changes in vital signs would be an increased systolic BP with widened pulse pressure and bradycardia

You receive a patient in the ER who has sustained a cervical spinal cord injury. You know this patient is at risk for neurogenic shock. What hallmark signs and symptoms, if experienced by this patient, would indicate the patient is experiencing neurogenic shock? Select all that apply:* A. Blood pressure 69/38 B. Heart rate 170 bpm C. Blood pressure 250/120 D. Heart rate 29 E. Warm and dry extremities F. Cool and clammy extremities G. Temperature 104.9 'F H. Temperature 95 'F

A, D, E, and H. Hallmark signs and symptoms of neurogenic shock are: hypotension, bradycardia, hypothermia, warm/dry extremities (this is due to the vasodilation and blood pooling and will be found in the extremities).

The nurse is admitting a client who is suspected of having delirium. The nurse should assess for which potential precipitating​ factor? (Select all that​ apply.) A. Infection B. Excessive sleep C. Change in environment D. Moderate alcohol use E. History of hypoglycemia

A,C,E

Blood sugar is well controlled when Hemoglobin A1C is: a. Below 7% b. Between 12%-15% c. Less than 180 mg/dL d. Between 90 and 130 mg/dL

A- below 7%A1c measures the percentage of hemoglobin that is glycated and determines average blood glucose during the 2 to 3 months prior to testing. Used as a diagnostic tool, A1C levels of 6.5% or higher on two tests indicate diabetes. A1C of 6% to 6.5% is considered

You're providing discharge teaching to a patient about pernicious anemia. Which statement by the patient indicates they did NOT understand the discharge teaching? A. "Pernicious anemia is caused by not consuming enough Vitamin B12." B. "Pernicious anemia causes the red blood cells to appear very large and oval." C. "Treatment for pernicious anemia includes a series of intramuscular injections of Vitamin B12." D. "A red, smooth tongue can be a sign of pernicious anemia."

A. "Pernicious anemia is caused by not consuming enough Vitamin B12." Pernicious anemia is caused by a lack of instrict factor which helps your body absorb B12.

The nurse is teaching a patient who will begin taking furosemide. The nurse learns that the patient has just begun a 2-week course of a steroid medication. What will the nurse recommend? a. Consume licorice to prevent excess potassium loss. b. Report a urine output greater than 600 mL/24 hours. c. Obtain an order for a potassium supplement. d. Take the furosemide at bedtime.

ANS: C The interaction of furosemide and a steroid drug can result in an increased loss of potassium. Patients should take a potassium supplement. Patients should avoid licorice while taking furosemide, partially due to the hypokalemic effects of both substances. Urine output greater than 600 mL/24 hours is normal. Patients should take furosemide in the morning to avoid nocturia.

A patient with diabetes is starting on intensive insulin therapy. Which type of insulin will the nurse discuss using for mealtime coverage? a. Lispro (Humalog) b. Glargine (Lantus) c. Detemir (Levemir) d. NPH (Humulin N)

ANS: A Rapid- or short-acting insulin is used for mealtime coverage for patients receiving intensive insulin therapy. NPH, glargine, or detemir will be used as the basal insulin.

The nurse has administered 4 oz of orange juice to an alert patient whose blood glucose was 62 mg/dL. Fifteen minutes later, the blood glucose is 67 mg/dL. Which action should the nurse take next? a. Give the patient 4 to 6 oz more orange juice. b. Administer the PRN glucagon (Glucagon) 1 mg IM. c. Have the patient eat some peanut butter with crackers. d. Notify the health care provider about the hypoglycemia.

ANS: A The "rule of 15" indicates that administration of quickly acting carbohydrates should be done two or three times for a conscious patient whose glucose remains less than 70 mg/dL before notifying the health care provider. More complex carbohydrates and fats may be used after the glucose has stabilized. Glucagon should be used if the patient's level of consciousness decreases so that oral carbohydrates can no longer be given.

Which topic will the nurse teach after a patient has had outpatient cataract surgery and lens implantation? a. Use of oral opioids for pain control b. Administration of corticosteroid eye drops c. Importance of coughing and deep breathing exercises d. Need for bed rest for the first 1 to 2 days after the surgery

ANS: B Antibiotic and corticosteroid eye drops are commonly prescribed after cataract surgery. The patient should be able to administer them using safe technique. Pain is not expected after cataract surgery and opioids will not be needed. Coughing and deep breathing exercises are not needed because a general anesthetic agent is not used. There is no bed rest restriction after cataract surgery

The nurse is caring for a patient who is receiving furosemide (Lasix) and an aminoglycoside antibiotic. The nurse will be most concerned if the patient reports which symptom? a. Dizziness b. Dysuria c. Nausea d. Tinnitus

ANS: D The interaction of furosemide and an aminoglycoside can produce ototoxicity in the patient. Tinnitus is a sign of ototoxicity. Dizziness can occur as a result of diuretic therapy but not necessarily as a result of this combination. Dysuria and nausea are not common signs of these drugs interacting

In reviewing a 55-year-old patient's medical record, the nurse notes that the last eye examination revealed an intraocular pressure of 28 mm Hg. The nurse will plan to assess a. visual acuity. b. pupil reaction. c. color perception. d. peripheral vision.

ANS: D The patient's increased intraocular pressure indicates glaucoma, which decreases peripheral vision. Because central visual acuity is unchanged by glaucoma, assessment of visual acuity could be normal even if the patient has worsening glaucoma. Color perception and pupil reaction to light are not affected by glaucoma

The nurse will anticipate teaching a patient with a possible seizure disorder about which test? a.Cerebral angiography b.Evoked potential studies c.Electromyography (EMG) d.Electroencephalography (EEG)

ANS: DSeizure disorders are usually assessed using EEG testing. Evoked potential is used to diagnose problems with the visual or auditory systems. Cerebral angiography is used to diagnose vascular problems. EMG is used to evaluate electrical innervation to skeletal muscle.DIF: Cognitive Level: Understand (comprehension) REF: 1311TOP: Nursing Process: Planning MSC: NCLEX: Physiological Integrity

The nurse is caring for a client with suspected disseminated intravascular coagulation​ (DIC). Which diagnostic test result supports the diagnosis of​ DIC? A. Decreased fibrin degradation products B. High fibrinogen levels C. Increased platelet count D. Increased​ D-dimer levels

D Plasma concentrations of D-dimer are significantly higher in patients with disseminated intravascular coagulation (DIC)

Which cranial nerves move the eyeballs? A. I, II, III B. III, IV, VI C. III, V, X D. III, IV

B. III, IV, VI - Oculomotor, Trochlear, and Abducens

Which of the following is NOT used in the treatment of sepsis? A. Crystalloid fluids B. Nitroglycerin C. Vasopressors D. Broad spectrum antibiotics

B. Nitroglycerin

Which of the following is not a symptom of sepsis? A. Short of breath B. Clammy and sweaty skin C. Bradycardia D. Confusion

C. Bradycardia Tachycardia is a side effect of sepsis because your body is trying to pump faster to get blood to the organs because of vasodilation and capillary leakage.

The nurse would question a prescription for steroids in a patient with which condition? A. Asthma B. Spinal cord injury C. Diabetes mellitus D. Rheumatoid arthritis

C. Diabetes mellitus

You are providing diet teaching to a patient with low iron levels. Which foods would you encourage the patient to eat regularly?* A. Herbal tea, apples, and watermelon B. Sweet potatoes, artichokes, and packaged meat C. Egg yolks, beef, and legumes D. Chocolate, cornbread, and cabbage

C. Egg yolks, beef, and legumes

Glucocorticoids are administered topically for: A. Control of steroid-responsive bronchospastic states B. Rhinitis and to prevent the recurrence of polyps after surgical removal C. Inflammations of the eye, ear, and skin D. Shock, status asthmaticus and spinal cord injury

C. Inflammations of the eye, ear, and skin

Which of the following signs and symptoms of increased ICP after head trauma would appear first? A. Bradycardia B. Large amounts of very dilute urine C. Restlessness and confusion D. Widened pulse pressure

C. Restlessness and confusion

Which of the following insulin cannot be mixed with any other type of insulin? A. Insulin glargine B. Insulin aspart C. Insulin isophane D. Insulin lispro

Correct Answer: A. Insulin glargine Insulin glargine when mixed with any other types of insulin changes its duration of action ( a combination of long-acting and short-acting insulin) so it is advised that it should not be mixed with any other type of insulin.

A patient admitted with a head injury has admission vital signs of temperature 98.6° F (37° C), blood pressure 128/68, pulse 110, and respirations 26. Which of these vital signs, if taken 1 hour after admission, will be of most concern to the nurse? a. Blood pressure 130/72, pulse 90, respirations 32 b. Blood pressure 148/78, pulse 112, respirations 28 c. Blood pressure 156/60, pulse 60, respirations 14 d. Blood pressure 110/70, pulse 120, respirations 30

Correct Answer: C Rationale: Systolic hypertension with widening pulse pressure, bradycardia, and respiratory changes represent Cushing's triad and indicate that the ICP has increased and brain herniation may be imminent unless immediate action is taken to reduce ICP. The other vital signs may indicate the need for changes in treatment, but they are not indicative of an immediately life-threatening process.

A patient has a systemic blood pressure (BP) of 120/60 mm Hg and an intracranial pressure of 24 mm Hg. The nurse determines that the cerebral perfusion pressure (CPP) of this patient indicates a. high blood flow to the brain. b. normal intracranial pressure (ICP). c. impaired brain blood flow. d. adequate cerebral perfusion.

Correct Answer: C Rationale: The patient's CPP is 56, below the normal of 70 to 100 mm Hg and approaching the level of ischemia and neuronal death. The patient has low cerebral blood flow/perfusion. Normal ICP is 0 to 15 mm Hg.

A diagnosis of Hodgkin's disease is suspected in a 12 year old child seen in a clinic. Several diagnostic studies are performed to determine the presence of this disease. Which diagnostic test results confirm the diagnosis of Hodgkin's disease? 1. Elevated vanillylmandelic acid urinary levels. 2. The presence of blast cells in the bone marrow. 3. The presence of Epstein-Barr virus in the blood. 4. The presence of Reed-Sternberg cells in the lymph nodes

Correct answer: #4 Hodgkin's disease is a malignancy of the lymph nodes. The presence of giant, multinucleated cells (Reed-sternberg cells) is the classic characteristic of this disease. The presence of blast cells in the bone marrow indicates leukemia. Epstein Barr virus is associated with infectious mononucleosis. Elevated levels of vanillylmandelic acid in the urine may be found in children with neuroblastoma."

Antiretroviral drugs are used to a. cure acute HIV infection. b. decrease viral RNA levels. c. treat opportunistic diseases. d. decrease pain and symptoms in terminal disease.

Correct answer: b Rationale: The goals of drug therapy in HIV infection are to (1) decrease the viral load, (2) maintain or raise CD4+ T cell counts, and (3) delay onset of HIV infection-related symptoms and opportunistic diseases.

The ambulance reports that they are transporting a patient to the ED who has experienced a full-thickness thermal burn from a grill. What manifestations should the nurse expect? A. Severe pain, blisters, and blanching with pressure B. Pain, minimal edema, and blanching with pressure C. Redness, evidence of inhalation injury, and charred skin D. No pain, waxy white skin, and no blanching with pressure

D. No pain, waxy white skin, and no blanching with pressure With full-thickness burns, the nerves and vasculature in the dermis are destroyed so there is no pain, the tissue is dry and waxy-looking or may be charred, and there is no blanching with pressure. Severe pain, blisters, and blanching occur with partial-thickness (deep, second-degree) burns. Pain, minimal edema, blanching, and redness occur with partial-thickness (superficial, first-degree) burns.

Which nerve does NOT play a role in swallowing?* A. Glossopharyngeal B. Hypoglossal C. Vagus D. Olfactory

D. Olfactory (Nerve I) which is responsible for hearing.

The nurse is caring for a client admitted with suspected myasthenia gravis. Which finding is usually associated with a diagnosis of myasthenia gravis? A. Increased saliva production B. Ascending paralysis and loss of motor function C. Cogwheel rigidity and loss of coordination D. Progressive weakness that is worse at the day's end

D. Progressive weakness that is worse at the days end The client with myasthenia develops progressive weakness that worsens during the day. Ascending paralysis and loss of motor function is incorrect because it refers to symptoms of Guillain Barre syndrome. Cogwheel rigidity and loss of coordination is incorrect because it refers to Parkinsons disease.

The function of cranial nerve I is?* A. hearing B. moving the eyeballs C. sight D. smell

D. Smell - Cranial nerve 1 is the olfactory nerve and is a sensory nerve responsible for smell.

The female patient has been brought to the ED with a sudden onset of a severe headache that is different from any other headache she has had previously. When considering the possibility of a stroke, which type of stroke should the nurse know is most likely occurring? A. TIA B. Embolic stroke C. Thrombotic stroke D. Subarachnoid hemorrhage

D. Subarachnoid hemorrhage Headache is common in a patient who has a subarachnoid hemorrhage or an intracerebral hemorrhage. A TIA is a transient loss of neurologic function usually without a headache. A headache may occur with an ischemic embolic stroke, but severe neurologic deficits are the initial symptoms. The ischemic thrombotic stroke manifestations progress in the first 72 hours as infarction and cerebral edema increase.

Cranial nerve IV is known as? A. Oculomotor B. Trigeminal C. Hypoglossal D. Trochlear

D. Trochlear Oculomotor nerve is Cranial nerve III Trigeminal nerve is Cranial nerve V Hypoglossal nerve is Cranial nerve XII

This is the largest cranial nerve that controls facial muscles, chewing, and facial sensations? A. X B. IV C. VIII D. V

D. V - Trigeminal

What is the most common cause of Guillain-Barré syndrome? A. Parasite infestation causing demyelinization. B. Brain neuron damage from plaques. C. Systemic sepsis from a bacterial infection. D. Viral infection or immunization.

D. Viral infection or immunization. The syndrome is often preceded by immune system stimulation by a viral infection, trauma, surgery, viral immunization, or human immunodeficiency virus (HIV). The other options are not related to Guillain-Barré syndrome.

Your patient has severe peripheral arterial disease. When the lower extremities are elevated you would expect them to appear _______________ and, when they are in the dependent position you would expect them to appear _________________. Fill in the blanks:* A. cyanotic; rubor B. rubor; pallor C. cyanotic, pallor D. pallor; rubor

D. pallor; rubor

During nursing report you learn that the patient you will be caring for has Guillain-Barré Syndrome. As the nurse you know that this disease tends to present with: A. signs and symptoms that are unilateral and descending that start in the lower extremities B. signs and symptoms that are symmetrical and ascending that start in the upper extremities C. signs and symptoms that are asymmetrical and ascending that start in the upper extremities D. signs and symptoms that are symmetrical and ascending that start in the lower extremities

D. signs and symptoms that are symmetrical and ascending that start in the lower extremities

Nurse Josefina is caring for a client who has been diagnosed with delirium. Which statement about delirium is true? A. It's characterized by an acute onset and lasts about 1 month. B. It's characterized by a slowly evolving onset and lasts about 1 week. C. It's characterized by a slowly evolving onset and lasts about 1 month. D. It's characterized by an acute onset and lasts hours to a number of days.

D: Delirium has an acute onset and typically can last from several hours to several days.

SONG BREAK! STAND UP AND SHAKE YOUR BODY. SING THE CHORUS OF YOUR FAVORITE SONG AND SMILE! YOU GOT THIS!

DID YOU REALLY LOOK TO SEE IF THERE WAS AN ANSWER. SEE YOU ARE FATIGUED. START SINING! TEXT ME WHAT SONG YOU CHOSE

https://www.youtube.com/watch?v=M11SvDtPBhA

Dance, wiggle, and sing it!

"The client asks the nurse, "They say I have cancer. How can they tell if I have Hodgkin'sdisease from a biopsy?" The nurse's answer is based on which scientific rationale? "A.Biopsies are nuclear medicine scans that can detect cancer. B.A biopsy is a laboratory test that detects cancer cells. C.It determines which kind of cancer the client has. D.A small piece is taken out of the tumor and is examined. If Reid Sternbburg cells are present, then Hodgkins lymphoma is confirmed.

D-Correct: A biopsy is the removal of cells from a massand examination of the tissue under amicroscope to determine if the cells arecancerous. Reed-Sternberg cells are diagnostic for Hodgkin's disease. If these cellsare not found in the biopsy, the HCP can rebiopsy to make sure the specimen pro- vided the needed sample or, depending on involvement of the tissue, diagnose a non-Hodgkin's lymphoma"

You're providing discharge teaching to a female patient on how to prevent urinary tract infections. Which statement is INCORRECT?* A. "Void immediately after sexual intercourse." B. "Avoid wearing tight fitting underwear." C. "Try to void every 2-3 hours." D. "Use scented sanitary napkins or tampons during menstruation."

D. "Use scented sanitary napkins or tampons during menstruation."

A patient admits with AKI resulting from heart failure. Which type of AKI is this? 1) Prerenal 2) Intrarenal 3) Postrenal 4) Interrenal

1) Prerenal Impaired cardiac efficiency = prerenal Others: MI, dysrhythmias, cardiogenic shock

A 14 year-old patient admits to the E.D. with profuse vomiting for the past 48 hours. Which of the following types of acute renal failurekidney injury is this patient at risk for? 1) Prerenal 2) Intrarenal 3) Postrenal 4) Interrenal

1)prerenal Volume depletion [dehydration/GI losses] = prerenalOthers: hemorrhage, renal losses (diuretics/osmotic diuresis), suctioning

The nurse knows that which of the following are side effects seen with immunosuppressive therapy? Select all that apply: 1) Nephrotoxicity 2) Hypertension 3) Cancer 4) Blurred vision 5) Hair loss

1, 2, 3 Other side effects include: hyperlipidemia, hirsutism, and tremors

A patient admits with AKI resulting from overuse of NSAIDs. Which type of AKI is this? 1) Prerenal 2) Intrarenal 3) Postrenal 4) Interrenal

2) Intrarenal Nephrotoxic agents = intrarenal Others: ACE inhibitors (-pril), chemotherapy drugs, corticosteriods, vancomyocin, gentomyocin

After a severe 3rd degree burn, a patient develops myoglobinuria. Which of the following types of acute kidney injury is this patient at risk for? 1) Prerenal 2) Intrarenal 3) Postrenal 4) Interrenal

2) Intrarenal Other things causing myoglobinuria: crashes, infections, severe blood transfusion reactions

A nurse is preparing to care for a client with potassium deficit. The nurse reviews the client's record and determines that the client was a risk for developing the potassium deficit because the client: 1) Sustained tissue damage 2) Requires nasogastric suctioning 3) Has a history of Addison's disease 4) Is taking a potassium-sparing diuretic

2) Requires nasogastric suctioning

What happens to urine sodium levels in a patient with acute kidney injury? 1) increased 2) decreased 3) about the same 4) none of the above

2) decreased The kidney's are having trouble concentrating the urine, so the amount of sodium is really low in the urine.

A 54 year-old male admits to the E.D. and is diagnosed with AKI secondary to BPH. Which type of AKI does this patient have? 1) Prerenal 2) Intrarenal 3) Postrenal 4) Interrenal

3) postrenal Obstruction = postrenal Others: calculi, tumors, strictures, blood clots

The nurse instructs a patient with thrombocytopenia about safety measures. Which statement made by the patient indicates the need for further instruction? 1 "I will avoid blowing my nose." 2 "At the gym, I will not lift weights." 3 "I will use an electric razor to shave." 4 "I will take aspirin if I have a headache."

4 "I will take aspirin if I have a headache."

A patient with acute kidney injury calls for the nurse and tells her that he is feeling weak and has developed loose stools over the past 2 hours. He also says he feels like his chest is fluttering. What does the nurse suspect may be happening? 1) Hypernatremia 2) Azotemia symptoms 3) Hypocalcemia 4) Hyperkalemia

4) Look for: diarrhea, weakness, dysrhythmias, and cardiac arrest.

The client with which condition is at the greatest risk of developing acute disseminated intravascular​ coagulation? A. Aortic aneurysm B. Gunshot wound to the distal arm C. Third-degree burns and septic shock D. Bacterial pneumonia treated with antibiotics

C

Which of the following laboratory values could indicate that a child has leukemia? "1. WBCs 32,000/mm3 2. Platelets 300,000/mm3 3. Hemoglobin 15g/dL

"Correct: 1. 1. YES! - A normal WBC count is approximately 4.5 mm3 - 11.0 mm3. In leukemia a high WBC count is diagnostic and is usually confirmed by a blood smear. 2-4. None of these indicate leukemia,"

A patient has an arterial ulcer on the lower extremity. What risk factors for peripheral arterial disease are in the patient's health history? Select all that apply:* A. Pregnancy B. Being Female C. High Cholesterol D. Diabetes Mellitus E. Uncontrolled hypertension F. Varicose veins G. Smoking

C, D, E, and G

"The female client recently diagnosed with Hodgkin's lymphoma asks the nurse about her prognosis. Which is the nurse's best response? "1.Survival for Hodgkin's disease is relatively good with standard therapy. 2.Survival depends on becoming involved in an investigational therapy program. 3.Survival is poor, with more than 50% of clients dying within six (6) months. 4.Survival is fine for primary Hodgkin's, but secondary cancers occur within a year."

"Correct Answer: 1. 1.Up to 90% of clients responds well to standard treatment with chemotherapy and radiation therapy, and those that relapse usually respond to a change of chemotherapy medications. Survival depends on the individual client and the stage of disease at diagnosis. 2.Investigational therapy regimens would not be recommended for clients initially diagnosed with Hodgkin's because of the expected prognosis with standard therapy. 3.Clients usually achieve a significantly longer survival rate than six (6) months. Many clients survive to develop long-term secondary complications. 4.Secondary cancers can occur as long as 20 years after a remission of the Hodgkin's disease has occurred."

Which of the following manifestations would be directly associated with Hodgkin's disease? "a. bone pain b. generalized edema c. petechiae and purpura d. painless, enlarged lymph nodes"

"Correct answer: D Rationale: Hodgkin's disease usually presents as painless enlarged lymph nodes. The diagnosis is made by lymph node biopsy."

The nurse would include which statement when teaching a patient about insulin glargine? "You should inject this insulin just before meals because it is very fast acting." "The duration of action for this insulin is approximately 8 to10 hours, so you will need to take it twice a day." "You can mix this insulin with NPH insulin to enhance its effects." "You cannot mix this insulin with regular insulin and thus will have to take two injections

"You cannot mix this insulin with regular insulin and thus will have to take two injections."

A nurse is assessing a client newly diagnosed with Stage I Hodgkin's lymphoma. Which area of the body would the nurse most likely find involved? 1. Back 2. Chest 3. Groin 4. Neck

(4. Neck is correct) At the time of diagnosis of stage I Hodgkin's lymphoma, a painless cervical lesion is often present. The back, chest, and groin areas may be involved in later stages.

Three year old Carlo has been admitted to the pediatric unit with a tentative diagnosis of nephrotic syndrome. The diagnosis of Idiopathic Nephrotic Syndrome has been confirmed. Which unexpected finding would the nurse report? a) proteinuria b) distended abdomen c) blood in the urine d) elevated serum lipid levels

) blood in the urine hematuria is rare in nephrotic syndrome but it is profuse is acute glomerulonephritis The manifestations of nephrotic syndrome are: Proteinuria - nephrosis is believed to be due to immunologic response that results in increased permeability of glomerular membrane to proteins resulting in massive protein losses in the urine -- proteinuria and albuminuria (+3 +4), the child losses 50-100 mg/kg weight/day from proteinuria. Hypoalbuminemia - loss of protein in blood results in hypoalbumenimia Edema - cardinal sign and appears first in the periorbital region followed by dependent edema and accompanied by pallor, fatigue and lethargy. Hypoalbuminemia leads to decreased oncotic pressure resulting in fluid shift from intravascular to interstitial causing generalized edema or anasarca.The child has lost appetite but gained weight -- puffiness of the eyes on awakening decreases during the day but appears on the legs and abdomen. Fluid shift causes decreased blood volume that leads to decreased blood supply to kidney. Decreased blood supply to kidney initiates release of aldosterone. Aldosterone causes sodium retention (in interstitial spaces so child will have hyponatremia) and water retention contributing to edema. Hypocholesteronemia and hyperlipidemia - triglycerides and fats are released by the liver in the blood to make up for the protein loss

A nurse is taking a blood sample with a syringe and large bore needle from a patient with chronic human immunodeficiency virus (HIV) who has a CD4+ T-cell count of 123/μL. If the nurse gets a needle injury, what factors may affect the transmission of HIV infection? Select all that apply. 1 Viral load 2 Age of the nurse 3 Volume of blood exposed to 4 Age of the patient 5 Immune status of nurse

1, 3, 5 Patients with a poor immune status are more susceptible to any kind of infection, including HIV. The concentration of the virus is an important variable. Other variables that influence the transmission are the volume of blood, virulence of the virus, and concentration of the organism in the blood. Large amounts of HIV can be found in the blood, and to a lesser extent in the semen, during the first 6 months of infection and again during the late stages. HIV positive patients can transmit the infection at any age to a person of any age when the route of transmission is established. Therefore, the age of the patient or nurse does not affect the transmission of HIV infection to the nurse.

Which client would be most at risk for developing disseminated intravascular coagulation (DIC)? 1. A 35-year-old pregnant client with placenta previa 2. A 42-year-old client with a pulmonary embolus 3. A 60-year-old client receiving hemodialysis 3 days a week 4. A 78-year-old client with septicemia

4. DIC is a clinical syndrome that develops as a complication of a wide variety of other disorders, with sepsis being the most common cause of DIC

Mike, a 43-year-old worker, has a history of hypertension. he smokes two packs a day, is nervous about the possibility of being unemployed, and has difficulty coping with stress. His current concern is calf pain during minimal exercise that decreased with rest. The nurse assesses Mike's symptoms as being associated with peripheral arterial occlusive disease. the nursing diagnosis is probably: 1. 1. Alteration in tissue perfusion related to compromised circulation 2. Dysfunctional use of extremities related to muscle spasms 3. Impaired mobility related to stress associated with pain 4. Impairment in muscle use associated with pain on exertion

1. 1. Alteration in tissue perfusion related to compromised circualtion

The client is admitted with full-thickness burns may be developing DIC. Which signs/symptoms would support the diagnosis of DIC? 1. Oozing blood from the IV catheter site 2. Sudden onset of chest pain and frothy sputum 3. Foul smelling, concentrated urine 4. A reddened, inflamed central line catheter site

1. Signs and symptoms of DIC result from clotting and bleeding, ranging from oozing blood to bleeding from every body orifice and into the tissues

A bedtime snack is provided for Albert. This is based on the knowledge that intermediate-acting insulins are effective for an approximate duration of: a. 6-8 hours b. 10-14 hours c. 12-18 hours d. 24-28 hour

12-18 hours

When caring for a client who is having clear drainage from his nares after transsphenoidal hypophysectomy, which action by the nurse is appropriate? 1. Lower the head of the bed. 2. Test the drainage for glucose. 3. Obtain a culture of the drainage. 4. Continue to observe the drainage.

2 After hypophysectomy, the client should be monitored for rhinorrhea, which could indicate a cerebrospinal fluid (CSF) leak. If this occurs, the drainage should be collected and tested for glucose, indicating the presence of CSF. The head of the bed should not be lowered to prevent increased intracranial pressure. Clear nasal drainage would not indicate the need for a culture. Continuing to observe the drainage without taking action could result in a serious complication.

The nurse understands that a patient with human immunodeficiency virus (HIV) starts to develop immune problems when their CD4 count: 1 Drops below 200 2 Drops below 500 3 Is greater than 500 4 Falls between 800 to 1200

2 Immune problems start to occur when the count drops below 500 CD4 T cells. When it drops below 200 CD4 T cells, severe immune problems will develop and the patient is diagnosed with acquired immunodeficiency syndrome (AIDS). The immune system generally remains healthy if there are more than 500 CD4 T cells. A count between 800 to 1200 CD4 T cells is normal for adults that do not have any immune dysfunction. Text Reference - p. 233

A patient with a platelet count of 52,000/mm3 is diagnosed with thrombocytopenia. The nurse expects what clinical manifestations? 1 Weakness and fatigue 2 Bruising and petechiae 3 Dizziness and vomiting 4 Lightheadedness and nausea

2 Bruising and petechiae - A low platelet count, known as thrombocytopenia, may be accompanied by signs of hemorrhage, such as bruising and petechiae. A normal platelet count is 150,000 to 400,000/mm3. The symptoms listed in the other answer options are not directly associated with thrombocytopenia.

A nurse is providing discharge instructions to the patient who just received a kidney transplant. The nurse instructs the patient to report which of the following? Select all that apply: 1) Large amounts of dilute urine 2) Fever 3) Weight gain 4) Swelling over the new kidney site 5) Chills

2,3,4,5 Large amounts of dilute urine are expected after a kidney transplantation, especially when the kidney is from a live donor.

A 24 year old man seeks medical attention for complaints of claudication in the arch of the lower leg. The nurse would next assess the client for: 1. Familial tendency toward peripheral vascular disease 2. Smoking history 3. Recent exposures to allergens 4. History of insect bites

2. Smoking history

A nurse caring for a client scheduled for a transsphenoidal hypophysectomy to remove a tumor in the pituitary gland assists to develop a plan of care for the client. The nurse suggests including which specific information in the preoperative teaching plan? 1. Hair will need to be shaved. 2. Deep breathing and coughing will be needed after surgery. 3. Toothbrushing will not be permitted for at least 2 weeks following surgery. 4. Spinal anesthesia is used.

3 Based on the location of the surgical procedure, spinal anesthesia would not be used. In addition, the hair would not be shaved. Although coughing and deep breathing are important, specific to this procedure is avoiding toothbrushing to prevent disruption of the surgical site. Also, coughing may disrupt the surgical site.

Following hypophysectomy, a client complains of being very thirsty and having to urinate frequently. The initial nursing action is to: 1. Document the complaints. 2. Increase fluid intake. 3. Check the urine specific gravity. 4. Check for urinary glucose.

3. Following hypophysectomy, diabetes insipidus can occur temporarily because of antidiuretic hormone deficiency. This deficiency is related to surgical manipulation. The nurse should check the urine for specific gravity and report the results if they are less than 1.005. Urinary glucose and diabetes mellitus is not a concern here. In this situation, increasing fluid intake would require a health care provider's prescription. The client's complaint would be documented but not as an initial action.

The patient asks about his lab test, which showed a high level of TSH and a low level of T4. You explain: 1. "It means that you have an inconsistency in your thyroid tests, and you will need more testing." 2. "I am sorry. You will have to ask your doctor about your lab results. We are not allowed to discuss them." 3. "The TSH is sending a message to your thyroid gland to increase production, but your thyroid isn't doing that." 4. "That means that you will have to go on hormone therapy for the rest of your life."

3. "The TSH is sending a message to your thyroid gland to increase production, but your thyroid isn't doing that." The test determines if the problem is in the pituitary or in the thyroid. In this case the high TSH is coming from the pituitary as it should but the thyroid is not responding.

Assuming the patient eats breakfast at 8:30 AM, lunch at noon, and dinner at 6:00 PM, he or she is at highest risk of hypoglycemia following an 8:00 AM dose of NPH insulin at what time? 10:00 AM 2:00 PM 5:00 PM 8:00 PM

5:00 PM

A nurse is providing self-care education to a patient who has been receiving treatment for acne vulgaris. What instruction should the nurse provide to the patient? A) "Wash your face with water and gentle soap each morning and evening." B) "Before bedtime, clean your face with rubbing alcohol on a cotton pad." C) "Gently burst new pimples before they form a visible 'head'." D) "Set aside time each day to squeeze blackheads and remove the plug."

A) "Wash your face with water and gentle soap each morning and evening."

The nurse expects which of the following assessment findings in the client in the diuretic phase of acute renal failure? a) Dehydration b) Crackles c) Hypertension d) Hyperkalemia

A) Dehydration The diuretic phase of acute renal failure is characterized by increased urine output, hypotension, and dehydration.

A patient who has type 2 diabetes has a glycated hemoglobin (HbA1c) result of 10%. A nurse should make which of these changes to the nursing care plan? A) Refer to a diabetic educator, there is poor glycemic control. B) Glycemic control is adequate, no changes are needed. C) Hypoglycemia is a risk, teach the patient the symptoms. D) Instruct the patient to limit activity and weekly exercise.

A) Refer to a diabetic educator, there is poor glycemic control.

A patient presents at the free clinic with a black, wart-like lesion on his face, stating, "I've done some research, and I'm pretty sure I have malignant melanoma." Subsequent diagnostic testing results in a diagnosis of seborrheic keratosis. The nurse should recognize what significance of this diagnosis? .A) The patient requires no treatment unless he finds the lesion to be cosmetically unacceptable. B) The patient's lesion will be closely observed for 6 months before a plan of treatment is chosen. C) The patient has one of the few dermatologic malignancies that respond to chemotherapy. D) The patient will likely require wide excision.

A) The patient requires no treatment unless he finds the lesion to be cosmetically unacceptable.

The nurse working in the dermatology clinic assesses a young adult female patient who is taking isotretinoin (Accutane) to treat severe cystic acne. Which assessment finding is most indicative of a need for further questioning of the patient? a. The patient recently had an intrauterine device removed. b. The patient already has some acne scarring on her forehead. c. The patient has also used topical antibiotics to treat the acne. d. The patient has a strong family history of rheumatoid arthritis.

ANS: A Because isotretinoin is teratogenic, contraception is required for women who are using this medication. The nurse will need to determine whether the patient is using other birth control methods. More information about the other patient data may also be needed, but the other data do not indicate contraindications to isotretinoin use

A patient with septic shock has a BP of 70/46 mm Hg, pulse 136, respirations 32, temperature 104° F, and blood glucose 246 mg/dL. Which intervention ordered by the health care provider should the nurse implement first? a. Give normal saline IV at 500 mL/hr. b. Give acetaminophen (Tylenol) 650 mg rectally. c. Start insulin drip to maintain BS at 110 to 150 mg/dL. d. Start norepinephrine (Levophed) to keep systolic BP >90 mm Hg.

ANS: A Because of the low systemic vascular resistance (SVR) associated with septic shock, fluid resuscitation is the initial therapy. The other actions also are appropriate, and should be initiated quickly as well.

The nurse is teaching a patient about taking hydrochlorothiazide. Which statement by the patient indicates a need for further teaching? a. I may need extra sodium and calcium while taking this drug. b. I should eat plenty of fruits and vegetables while taking this medication. c. I should take care when rising from a bed or chair when Im on this medication. d. I will take the medication in the morning to minimize certain side effects.

ANS: A Patients do not need extra sodium or calcium while taking thiazide diuretics. Thiazide diuretics can lead to hypokalemia, so patients should be counseled to eat fruits and vegetables that are high in potassium. Patients can develop orthostatic hypotension and should be counseled to rise from sitting or lying down slowly. Taking the medication in the morning helps to prevent nocturia-induced insomnia.

A patient has been taking spironolactone (Aldactone) to treat heart failure. The nurse will monitor for a. hyperkalemia. b. hypermagnesemia. c. hypocalcemia. d. hypoglycemia.

ANS: A Spironolactone is a potassium-sparing diuretic and can cause hyperkalemia.

To evaluate the effectiveness of antiretroviral therapy (ART), which laboratory test result will the nurse review? a. Viral load testing b. Enzyme immunoassay c. Rapid HIV antibody testing d. Immunofluorescence assay

ANS: A The effectiveness of ART is measured by the decrease in the amount of virus detectable in the blood. The other tests are used to detect HIV antibodies, which remain positive even with

The nurse is reviewing the laboratory tests for a patient who has recently been diagnosed with hypertension. Which result is most important to communicate to the health care provider? a. Serum creatinine of 2.6 mg/dL b. Serum potassium of 3.8 mEq/L c. Serum hemoglobin of 14.7 g/dL d. Blood glucose level of 98 mg/dL

ANS: A The elevated creatinine indicates renal damage caused by the hypertension. The other laboratory results are normal.

The client is being treated with furosemide and a steroid drug as well. As a result of the interaction of the drugs, the nurse should expect to see an increased loss of: a. potassium. b. calcium. c. magnesium. d. sodium.

ANS: A The interaction of furosemide and a steroid drug can result in an increased loss of potassium.

For a patient who had a right hemisphere stroke the nurse establishes a nursing diagnosis of a. risk for injury related to denial of deficits and impulsiveness. b. impaired physical mobility related to right-sided hemiplegia. c. impaired verbal communication related to speech-language deficits. d. ineffective coping related to depression and distress about disability.

ANS: A The patient with right-sided brain damage typically denies any deficits and has poor impulse control, leading to risk for injury when the patient attempts activities such as transferring from a bed to a chair. Right-sided brain damage causes left hemiplegia. Left-sided brain damage typically causes language deficits. Left-sided brain damage is associated with depression and distress about the disability. DIF: Cognitive Level: Apply (application) REF: 1407 TOP: Nursing Process: Diagnosis MSC: NCLEX: Physiological Integrity

A patient with atopic dermatitis has been using a high-potency topical corticosteroid ointment for several weeks. The nurse should assess for which adverse effect? a. Thinning of the affected skin b. Alopecia of the affected areas c. Reddish-brown discoloration of the skin d. Dryness and scaling in the areas of treatment

ANS: A Thinning of the skin indicates that atrophy, a possible adverse effect of topical corticosteroids, is occurring. The health care provider should be notified so that the medication can be changed or tapered. Alopecia, red-brown discoloration, and dryness/scaling of the skin are not adverse effects of topical corticosteroid use

A patient who has type 1 diabetes plans to swim laps for an hour daily at 1:00 PM. The clinic nurse will plan to teach the patient to a. check glucose levels before, during, and after swimming. b. delay eating the noon meal until after the swimming class. c. increase the morning dose of neutral protamine Hagedorn (NPH) insulin. d. time the morning insulin injection so that the peak occurs while swimming.

ANS: A The change in exercise will affect blood glucose, and the patient will need to monitor glucose carefully to determine the need for changes in diet and insulin administration. Because exercise tends to decrease blood glucose, patients are advised to eat before exercising. Increasing the morning NPH or timing the insulin to peak during exercise may lead to hypoglycemia, especially with the increased exercise.

The nurse is assessing a 55-yr-old female patient with type 2 diabetes who has a body mass index (BMI) of 31 kg/m2.Which goal in the plan of care is most important for this patient? a. The patient will reach a glycosylated hemoglobin level of less than 7%. b. The patient will follow a diet and exercise plan that results in weight loss. c. The patient will choose a diet that distributes calories throughout the day. d. The patient will state the reasons for eliminating simple sugars in the diet.

ANS: A The complications of diabetes are related to elevated blood glucose and the most important patient outcome is the reduction of glucose to near-normal levels. A BMI of 30?9?kg/m2 or above is considered obese, so the other outcomes are appropriate but are not as high in priority.

A patient receives aspart (NovoLog) insulin at 8:00 AM. At which time would the nurse anticipate the highest risk for hypoglycemia?a. 10:00 AM b. 12:00 AM c. 2:00 PM d. 4:00 PM

ANS: A The rapid-acting insulins peak in 30 min. to 3 hours. The patient is not at a high risk for hypoglycemia at the other listed times, although hypoglycemia may occur.

A nurse is teaching a patient with contact dermatitis of the arms and legs about ways to decrease pruritus. Which information should the nurse include in the teaching plan (select all that apply)? a. Cool, wet cloths or dressings can be used to reduce itching. b. Take cool or tepid baths several times daily to decrease itching. c. Add oil to your bath water to aid in moisturizing the affected skin. d. Rub yourself dry with a towel after bathing to prevent skin maceration. e. Use of an over-the-counter (OTC) antihistamine can reduce scratching.

ANS: A, B, E Cool or tepid baths, cool dressings, and OTC antihistamines all help reduce pruritus and scratching. Adding oil to bath water is not recommended because of the increased risk for falls. The patient should use the towel to pat (not rub) the skin dry

A patient who has been treated for status epilepticus in the emergency department will be transferred to the medical nursing unit. Which equipment should the nurse have available in the patient's assigned room (select all that apply)? a. Side-rail pads b. Tongue blade c. Oxygen mask d. Suction tubing e. Urinary catheter f. Nasogastric tube

ANS: A, C, D The patient is at risk for further seizures, and O2 and suctioning may be needed after any seizures to clear the airway and maximize oxygenation. The bed's side rails should be padded to minimize the risk for patient injury during a seizure. Use of tongue blades during a seizure is contraindicated. Insertion of a nasogastric (NG) tube is not indicated because the airway problem is not caused by vomiting or abdominal distention. A urinary catheter is not required unless there is urinary retention.

After change-of-shift report in the progressive care unit, who should the nurse care for first? a. Patient who had an inferior myocardial infarction 2 days ago and has crackles in the lung bases b. Patient with suspected urosepsis who has new orders for urine and blood cultures and antibiotics c. Patient who had a T5 spinal cord injury 1 week ago and currently has a heart rate of 54 beats/minute d. Patient admitted with anaphylaxis 3 hours ago who now has clear lung sounds and a blood pressure of 108/58 mm Hg

ANS: B Antibiotics should be administered within the first hour for patients who have sepsis or suspected sepsis in order to prevent progression to systemic inflammatory response syndrome (SIRS) and septic shock. The data on the other patients indicate that they are more stable. Crackles heard only at the lung bases do not require immediate intervention in a patient who has had a myocardial infarction. Mild bradycardia does not usually require atropine in patients who have a spinal cord injury. The findings for the patient admitted with anaphylaxis indicate resolution of bronchospasm and hypotension.

Which finding is the best indicator that the fluid resuscitation for a patient with hypovolemic shock has been effective? a. Hemoglobin is within normal limits. b. Urine output is 60 mL over the last hour. c. Central venous pressure (CVP) is normal. d. Mean arterial pressure (MAP) is 72 mm Hg.

ANS: B Assessment of end organ perfusion, such as an adequate urine output, is the best indicator that fluid resuscitation has been successful. The hemoglobin level, CVP, and MAP are useful in determining the effects of fluid administration, but they are not as useful as data indicating good organ perfusion.

The nurse is caring for a patient who has septic shock. Which assessment finding is most important for the nurse to report to the health care provider? a. Blood pressure (BP) 92/56 mm Hg b. Skin cool and clammy c. Oxygen saturation 92% d. Heart rate 118 beats/minute

ANS: B Because patients in the early stage of septic shock have warm and dry skin, the patient's cool and clammy skin indicates that shock is progressing. The other information will also be reported but does not indicate deterioration of the patient's status.

A patient in the clinic reports a recent episode of dysphasia and left-sided weakness at home that resolved after 2 hours. The nurse will anticipate teaching the patient about a. alteplase (tPA). b. aspirin (Ecotrin). c. warfarin (Coumadin). d. nimodipine (Nimotop).

ANS: B Following a transient ischemic attack (TIA), patients typically are started on medications such as aspirin to inhibit platelet function and decrease stroke risk. tPA is used for acute ischemic stroke. Coumadin is usually used for patients with atrial fibrillation. Nimodipine is used to prevent cerebral vasospasm after a subarachnoid hemorrhage. DIF: Cognitive Level: Apply (application) REF: 1396 TOP: Nursing Process: Planning MSC: NCLEX: Physiological Integrity

The nurse is caring for a patient who develops marked edema and a low urine output as a result of heart failure. Which medication will the nurse expect the provider to order for this patient? a. Digoxin (Lanoxin) b. Furosemide (Lasix) c. Hydrochlorothiazide d. Spironolactone

ANS: B Furosemide is a loop diuretic and is given when the patients condition warrants immediate removal of body fluid, as in heart failure. Digoxin improves cardiac function but does not remove fluid quickly. The other diuretics may be used when immediate fluid removal is not necessary.

A patient with glaucoma who has been using timolol (Timoptic) drops for several days tells the nurse that the eye drops cause eye burning and visual blurriness for a short time after administration. The best response to the patient's statement is a. "Those symptoms may indicate a need for an increased dosage of the eye drops." b. "The drops are uncomfortable, but it is important to use them to retain your vision." c. "These are normal side effects of the drug, which should be less noticeable with time." d. "Notify your health care provider so that different eye drops can be prescribed for you."

ANS: B Patients should be instructed that eye discomfort and visual blurring are expected side effects of the ophthalmic drops but that the drops must be used to prevent further visual-field loss. The temporary burning and visual blurriness might not lessen with ongoing use, are not relieved by avoiding systemic absorption, and are not symptoms of glaucoma

A teenaged male patient who wrestles in high school is examined by the nurse in the clinic. Which assessment finding would prompt the nurse to teach the patient about the importance of not sharing headgear to prevent the spread of pediculosis? a. Ringlike rashes with red, scaly borders over the entire scalp b. Papular, wheal-like lesions with white deposits on the hair shaft c. Patchy areas of alopecia with small vesicles and excoriated areas d. Red, hivelike papules and plaques with sharply circumscribed borders

ANS: B Pediculosis is characterized by wheal-like lesions with parasites that attach eggs to the base of the hair shaft. The other descriptions are more characteristic of other types of skin disorders

A patient who has severe Alzheimer's disease (AD) is being admitted to the hospital for surgery. Which intervention will the nurse include in the plan of care? a. Encourage the patient to discuss events from the past. b. Maintain a consistent daily routine for the patient's care. c. Reorient the patient to the date and time every 2 to 3 hours. d. Provide the patient with current newspapers and magazines.

ANS: B Providing a consistent routine will decrease anxiety and confusion for the patient.

Eight years after seroconversion, a human immunodeficiency virus (HIV)-infected patient has a CD4+ cell count of 800/µL and an undetectable viral load. What is the priority nursing intervention at this time? a. Teach about the effects of antiretroviral agents. b. Encourage adequate nutrition, exercise, and sleep. c. Discuss likelihood of increased opportunistic infections. d. Monitor for symptoms of acquired immunodeficiency syndrome (AIDS).

ANS: B The CD4+ level for this patient is in the normal range, indicating that the patient is the stage of asymptomatic chronic infection, when the body is able to produce enough CD4+ cells to maintain a normal CD4+ count. AIDS and increased incidence of opportunistic infections typically develop when the CD4+ count is much lower than normal. Although the initiation of ART is highly individual, it would not be likely that a patient with a normal CD4+ level would receive ART.

When a 74-yr-old patient is seen in the health clinic with new development of a stooped posture, shuffling gait, and pill rolling-type tremor, the nurse will anticipate teaching the patient about a. oral corticosteroids. b. antiparkinsonian drugs. c. magnetic resonance imaging (MRI). d. electroencephalogram (EEG) testing

ANS: B The clinical diagnosis of Parkinson's is made when tremor, rigidity, and akinesia, and postural instability are present. The confirmation of the diagnosis is made on the basis of improvement when antiparkinsonian drugs are administered. MRI and EEG are not useful in diagnosing Parkinson's disease, and corticosteroid therapy is not used to treat it.

The nurse obtains this information from a patient with hypertension. Which finding is most important to address with the patient? a. Low dietary fiber intake b. No regular aerobic exercise c. Weight 5 pounds above ideal weight d. Drinks wine with dinner once a week

ANS: B The recommendations for preventing hypertension include exercising aerobically for 30 minutes most days of the week. A weight that is 5 pounds over the ideal body weight is not a risk factor for hyperten- sion. The Dietary Approaches to Stop Hypertension (DASH) diet is high in fiber, but increasing fiber alone will not prevent hypertension from developing. The patient's alcohol intake will not increase the hypertension risk.

A client is taking a thiazide diuretic. The nurse assesses the client's serum glucose level—the fasting blood glucose level is 150 mg/dl. What is an appropriate response by the nurse? a. Instruct the client to discontinue taking hydrochlorothiazide. b. Inform the healthcare provider of the glucose level and the possible need for a different diuretic. c. Instruct the client to take hydrochlorothiazide every other day. d. Instruct the client to take an antidiabetic drug instead of the diuretic.

ANS: B Thiazide diuretics can lead to impaired insulin function and hyperglycemia, warranting changing diuretic agents.

When assessing a patient who spilled hot oil on the right leg and foot, the nurse notes that the skin is dry, pale, hard skin. The patient states that the burn is not painful. What term would the nurse use to document the burn depth? a. First-degree skin destruction b. Full-thickness skin destruction c. Deep partial-thickness skin destruction d. Superficial partial-thickness skin destruction

ANS: B With full-thickness skin destruction, the appearance is pale and dry or leathery and the area is painless because of the associated nerve destruction. Erythema, swelling, and blisters point to a deep partial-thickness burn. With superficial partial-thickness burns, the area is red, but no blisters are present. First-degree burns exhibit erythema, blanching, and pain

An active 32-yr-old male who has type 1 diabetes is being seen in the endocrine clinic. Which finding indicates a need for the nurse to discuss a possible a change in therapy with the health care provider? a. Hemoglobin A1C level of 6.2% b. Blood pressure of 140/88 mmHg c. Heart rate at rest of 58 beats/minute d. High density lipoprotein (HDL) level of 65 mg/dL

ANS: B To decrease the incidence of macrovascular and microvascular problems in patients with diabetes, the goal blood pressure is usually 130/80 mm Hg. An A1C less than 6.5%, a low resting heart rate (consistent with regular aerobic exercise in a young adult), and an HDL level of 65 mg/dL all indicate that the patient's diabetes and risk factors for vascular disease are well controlled.

Which action will the nurse include in the plan of care for a patient with impaired functioning of the left glossopharyngeal nerve (CN IX) and vagus nerve (CN X)? a.Assist to stand and ambulate. b.Withhold oral fluids and food. c.Insert an oropharyngeal airway. d.Apply artificial tears every hour.

ANS: BThe glossopharyngeal and vagus nerves innervate the pharynx and control the gag reflex. A patient with impaired function of these nerves is at risk for aspiration. An oral airway may be needed when a patient is unconscious and unable to maintain the airway, but it will not decrease aspiration risk. Taste and eye blink are controlled by the facial nerve. Balance and coordination are cerebellar functions.DIF: Cognitive Level: Apply (application) REF: 1305TOP: Nursing Process: Planning MSC: NCLEX: Physiological Integrity

A 68-year-old patient is being admitted with a possible stroke. Which information from the assessment indicates that the nurse should consult with the health care provider before giving the prescribed aspirin? a. The patient has dysphasia. b. The patient has atrial fibrillation. c. The patient reports that symptoms began with a severe headache. d. The patient has a history of brief episodes of right-sided hemiplegia.

ANS: C A sudden onset headache is typical of a subarachnoid hemorrhage, and aspirin is contraindicated. Atrial fibrillation, dysphasia, and transient ischemic attack (TIA) are not contraindications to aspirin use, so the nurse can administer the aspirin. DIF: Cognitive Level: Apply (application) REF: 1392-1393 TOP: Nursing Process: Assessment MSC: NCLEX: Physiological Integrity

A 72-year-old patient who has a history of a transient ischemic attack (TIA) has an order for aspirin 160 mg daily. When the nurse is administering medications, the patient says, "I don't need the aspirin today. I don't have a fever." Which action should the nurse take? a. Document that the aspirin was refused by the patient. b. Tell the patient that the aspirin is used to prevent a fever. c. Explain that the aspirin is ordered to decrease stroke risk. d. Call the health care provider to clarify the medication order.

ANS: C Aspirin is ordered to prevent stroke in patients who have experienced TIAs. Documentation of the patient's refusal to take the medication is an inadequate response by the nurse. There is no need to clarify the order with the health care provider. The aspirin is not ordered to prevent aches and pains. DIF: Cognitive Level: Apply (application) REF: 1396 TOP: Nursing Process: Implementation MSC: NCLEX: Physiological Integrity

While the nurse is transporting a patient on a stretcher to the radiology department, the patient begins having a tonic-clonic seizure. Which action should the nurse take? a. Insert an oral airway during the seizure to maintain a patent airway. b. Restrain the patient's arms and legs to prevent injury during the seizure. c. Time and observe and record the details of the seizure and postictal state. d. Avoid touching the patient to prevent further nervous system stimulation.

ANS: C Because the diagnosis and treatment of seizures frequently are based on the description of the seizure, recording the length and details of the seizure is important. Insertion of an oral airway and restraining the patient during the seizure are contraindicated. The nurse may need to move the patient to decrease the risk of injury during the seizure.

The nurse instructs a patient about application of corticosteroid cream to an area of contact dermatitis on the right leg. Which patient action indicates that further teaching is needed? a. The patient takes a tepid bath before applying the cream. b. The patient spreads the cream using a downward motion. c. The patient applies a thick layer of the cream to the affected skin. d. The patient covers the area with a dressing after applying the cream.

ANS: C Creams and ointments should be applied in a thin layer to avoid wasting the medication. The other actions by the patient indicate that the teaching has been successful

After the nurse teaches the patient with stage 1 hypertension about diet modifications that should be implemented, which diet choice indicates that the teaching has been most effective? a. The patient avoids eating nuts or nut butters. b. The patient restricts intake of chicken and fish. c. The patient drinks low-fat milk with each meal. d. The patient has two cups of coffee in the morning.

ANS: C For the prevention of hypertension, the Dietary Approaches to Stop Hypertension (DASH) recommendations include increasing the intake of calcium-rich foods. Caffeine restriction and decreased protein intake are not included in the recommendations. Nuts are high in beneficial nutrients and 4 to 5 servings weekly are recommended in the DASH diet.

Which stroke risk factor for a 48-year-old male patient in the clinic is most important for the nurse to address? a. The patient is 25 pounds above the ideal weight. b. The patient drinks a glass of red wine with dinner daily. c. The patient's usual blood pressure (BP) is 170/94 mm Hg. d. The patient works at a desk and relaxes by watching television.

ANS: C Hypertension is the single most important modifiable risk factor. People who drink more than 1 (for women) or 2 (for men) alcoholic beverages a day may increase risk for hypertension. Physical inactivity and obesity contribute to stroke risk but not as much as hypertension. DIF: Cognitive Level: Apply (application) REF: 1390 OBJ: Special Questions: Prioritization TOP: Nursing Process: Assessment MSC: NCLEX: Health Promotion and Maintenance

Which information should the nurse include when teaching a patient with newly diagnosed hypertension? a. Dietary sodium restriction will control BP for most patients. b. Most patients are able to control BP through lifestyle changes. c. Hypertension is usually asymptomatic until significant organ damage occurs. d. Annual BP checks are needed to monitor treatment effectiveness.

ANS: C Hypertension is usually asymptomatic until target organ damage has occurred. Lifestyle changes and sodium restriction are used to help manage blood pressure, but drugs are needed for most patients. BP should be checked by the health care provider every 3 to 6 months.

Thiazide diuretics are contraindicated if the client has: a. emphysema. b. arteriosclerotic cardiovascular disease. c. renal failure. d. viral infection.

ANS: C Renal failure decreases the excretion of the drug, leading to accumulation and electrolyte imbalance.

The nurse designs a program to decrease the incidence of human immunodeficiency virus (HIV) infection in the adolescent and young adult populations. Which information should the nurse assign as the highest priority? a. Methods to prevent perinatal HIV transmission b. Ways to sterilize needles used by injectable drug users c. Prevention of HIV transmission between sexual partners d. Means to prevent transmission through blood transfusions

ANS: C Sexual transmission is the most common way that HIV is transmitted. The nurse should also provide teaching about perinatal transmission, needle sterilization, and blood transfusion, but the rate of HIV infection associated with these situations is lower.

A hospitalized diabetic patient received 38 U of NPH insulin at 7:00 AM. At 1:00 PM, the patient has been away from the nursing unit for 2 hours, missing the lunch delivery while awaiting a chest x-ray. To prevent hypoglycemia, the best action by the nurse is to a. save the lunch tray for the patient's later return to the unit. b. ask that diagnostic testing area staff start a 5% dextrose IV. c. send a glass of milk or orange juice to the patient in the diagnostic testing area. d. request that if testing is further delayed, the patient is returned to the unit to eat.

ANS: D Consistency for mealtimes assists with regulation of blood glucose, so the best option is for the patient to have lunch at the usual time. Waiting to eat until after the procedure is likely to cause hypoglycemia. Administration of an IV solution is unnecessarily invasive for the patient. A glass of milk or juice will keep the patient from becoming hypoglycemic but will cause a rapid rise in blood glucose because of the rapid absorption of the simple carbohydrate in these items.

A nursing students needs to administer potassium chloride intravenously as prescribed to a client with hypokalemia. The nursing instructor determines that the student is unprepared for this procedure if the student states that which of the following is part of the plan for preparation and administration of the potassium? 1) Obtaining a controlled intravenous (IV) infusion pump 2) Monitoring urine output during administration 3) Preparing the medication for bolus administration 4) Diluting the medication in appropriate amount or normal saline

Answer: 3 Rational: Potassium chloride administered intravenously must always be diluted in IV fluid and infused via an infusion pump or controller. Potassium chloride is never given by bolus (IV push). Giving potassium chloride by IV push can result in cardiac arrest. Dilution in normal saline is recommended, and dextrose solution is avoided because this types of solution increases intracellular potassium shifting. The IV site is monitored closely because the potassium chloride is irritating to the veins and there is a risk of phlebitis. The nurse monitors urinary output during administration and contacts the physician if the urinary output is less than 30 mL/hr.

A nurse caring for a group of clients reviews the electrolytes laboratory results and notes a potassium level of 5.5 mEq/L on one client's laboratory report. The nurse understands that which client is a highest risk for developing a potassium value at this level? 1) The client with colitis 2) The client with Cushing's syndrome 3) The client who has been overusing laxatives 4) The client who has sustained a traumatic burn

Answer: 4Rational: A serum potassium level higher than 5 mEq/L indicates hyperkalemia. Clients who experience cellular shifting of potassium in the early stages of massive cell destruction, such as with trauma, burns, sepsis, or metabolic or respiratory acidosis are at risk for hyperkalemia. the client with Cushing's syndrome or colitis and the client who has been overusing laxatives are at risk for hypokalemia.

The nurse has instructed a patient who is receiving hemodialysis about dietary management. Which diet choices by the patient indicate that the teaching has been successful? a. Scrambled eggs, English muffin, and apple juice b. Cheese sandwich, tomato soup, and cranberry juice c. Split-pea soup, whole-wheat toast, and nonfat milk d. Oatmeal with cream, half a banana, and herbal tea

Answer: A Rationale: Scrambled eggs would provide high-quality protein, and apple juice is low in potassium. Cheese is high in salt and phosphate, and tomato soup would be high in potassium. Split-pea soup is high in potassium, and dairy products are high in phosphate. Bananas are high in potassium, and the cream would be high in phosphate.

A patient who is diagnosed with nephrotic syndrome has 3+ ankle and leg edema and ascites. Which nursing diagnosis is a priority for the patient? a. Fluid-volume excess related to low serum protein levels b. Altered nutrition: less than required related to protein restriction c. Activity intolerance related to increased weight and fatigue d. Disturbed body image related to peripheral edema and ascites

Answer: A Rationale: The patient has massive edema, so the priority problem at this time is the excess of fluid volume. The other nursing diagnoses are also appropriate, but the focus of nursing care should be resolution of the edema and ascites.

The nurse determines that instruction regarding prevention of future UTIs for a patient with cystitis has been effective when the patient states, a. "I will empty my bladder every 3 to 4 hours during the day." b. "I can use vaginal sprays to reduce bacteria." c. "I will wash with soap and water before sexual intercourse." d. "I will drink a quart of water or other fluids every day."

Answer: A Rationale: Voiding every 3 to 4 hours is recommended to prevent UTIs. Use of vaginal sprays is discouraged. The bladder should be emptied before and after intercourse, but cleaning with soap and water is not necessary. A quart of fluids is insufficient to provide adequate urine output to decrease risk for UTI.

A 34-year-old patient with diabetes mellitus is hospitalized with fever, anorexia, and confusion. The health care provider suspects acute pyelonephritis when the urinalysis reveals bacteriuria. An appropriate collaborative problem identified by the nurse for the patient is potential complication a. hydronephrosis. b. urosepsis. c. acute renal failure. d. chronic pyelonephritis.

Answer: B Rationale: Infection can easily spread from the kidney to the circulation, causing urosepsis. A patient with a urinary tract obstruction will be at risk for hydronephrosis. Acute renal failure is not a common complication of acute pyelonephritis unless urosepsis and septic shock develop. Chronic pyelonephritis may occur after recurrent upper UTIs.

A patient complains of leg cramps during hemodialysis. The nurse should a. give acetaminophen (Tylenol). b. infuse a bolus of normal saline. c. massage the patient's legs. d. reposition the patient.

Answer: B Rationale: Muscle cramps during dialysis are caused by rapid removal of sodium and water. Treatment includes infusion of normal saline. The other actions do not address the reason for the cramps.

When assessing the patient who has a lower urinary tract infection (UTI), the nurse will initially ask about a. flank pain. b. pain with urination. c. poor urine output. d. nausea.

Answer: B Rationale: Pain with urination is a common symptom of a lower UTI. Urine output does not decrease, but frequency may be experienced. Flank pain and nausea are associated with an upper UTI.

A patient with acute kidney injury (AKI) has an arterial blood pH of 7.30. The nurse will assess the patient for a. tachycardia. b. rapid respirations. c. poor skin turgor. d. vasodilation

Answer: B Rationale: Patients with metabolic acidosis caused by AKI may have Kussmaul respirations as the lungs try to regulate carbon dioxide. Tachycardia and vasodilation are not associated with metabolic acidosis. Because the patient is likely to have fluid retention, poor skin turgor would not be a finding in AKI.

A patient is admitted to the hospital with nephrotic syndrome after taking an OTC nonsteroidal antiinflammatory drug (NSAID) a week earlier. Which assessment data will the nurse expect to find related to this illness? a. Low blood pressure b. Recent weight gain c. Poor skin turgor d. High urine ketones

Answer: B Rationale: The patient with a rapid-onset nephrotic syndrome will have rapid weight gain associated with edema. Hypertension is a clinical manifestation of nephrotic syndrome. Skin turgor is normal because of the edema. Urine protein is high.

Which ocular or facial signs/ symptoms should the nurse expect to assess for the client diagnosed with myasthenia gravis? A) Weakness & Fatigue B) Ptosis & diplopia C) Breathlessness & dyspnea D) Weight loss & Dehydration

B

After teaching a client who is recovering from an transsphenoidal hypophysectomy, the nurse assesses the client's understanding. Which statement made by the client indicates a correct understanding of the teaching? a. I will wear dark glasses to prevent sun exposure. b. I'll keep food on upper shelves so I do not have to bend over. c. I must wash the incision with peroxide and redress it daily. d. I shall cough and deep breathe every 2 hours while I am awake.

B After this surgery, the client must take care to avoid activities that can increase intracranial pressure. The client should avoid bending from the waist and should not bear down, cough, or lie flat. With this approach, there is no incision to clean and dress. Protection from sun exposure is not necessary after this procedure.

A patient with massive trauma and possible spinal cord injury is admitted to the emergency department (ED). Which assessment finding by the nurse will help confirm a diagnosis of neurogenic shock? a. Inspiratory crackles b. Heart rate 45 beats/min c. Cool, clammy extremities d. Temperature 101.2°F (38.4°C)

B Neurogenic shock is characterized by hypotension and bradycardia. The other findings would be more consistent with other types of shock

A nurse cares for a client who possibly has syndrome of inappropriate antidiuretic hormone (SIADH). The client's serum sodium level is 114 mEq/L. Which action should the nurse take first? a. Consult with the dietitian about increased dietary sodium. b. Restrict the client's fluid intake to 600 mL/day. c. Handle the client gently by using turn sheets for re-positioning. d. Instruct unlicensed assistive personnel to measure intake and output.

B With SIADH, clients often have dilutional hyponatremia. The client needs a fluid restriction, sometimes to as little as 500 to 600 mL/24 hr. Adding sodium to the client's diet will not help if she is retaining fluid and diluting the sodium. The client is not at increased risk for fracture, so gentle handling is not an issue. The client should be on intake and output; however, this will monitor only the client's intake, so it is not the best answer. Reducing intake will help increase the client's sodium.

You're providing education to a patient about how to take their prescribed iron supplement. Which statement by the patient requires you to re-educate the patient on how to take this supplement?* A. "I will take this medication on an empty stomach." B. "I will avoid taking this medication with orange juice." C. "I will wait and take my calcium supplements 2 hours after I take my iron supplement." D. "This medication can cause constipation. So, I will drink plenty of fluids and take a stool softer as needed."

B - Vit C helps with the absorption of iron.

When teaching a patient about dietary management of stage 1 hypertension, which instruction is most appropriate? A Restrict all caffeine. B Restrict sodium intake. C Increase protein intake. D Use calcium supplements.

B Restrict sodium intake. The patient should decrease intake of sodium. This will help to control hypertension, which can be aggravated by excessive salt intake, which in turn leads to fluid retention. Caffeine and protein intake do not affect hypertension. Calcium supplements are not recommended to lower BP.

A nurse cares for a client who is recovering from a transsphenoidal hypophysectomy. Which action should the nurse take first? a. Keep the head of the bed flat and the client supine. b. Instruct the client to cough, turn, and deep breathe. c. Report clear or light yellow drainage from the nose. d. Apply petroleum jelly to lips to avoid dryness.

C A light yellow drainage or a halo effect on the dressing is indicative of a cerebrospinal fluid leak. The client should have the head of the bed elevated after surgery. Although deep breathing is important postoperatively, coughing should be avoided to prevent cerebrospinal fluid leakage. Although application of petroleum jelly to the lips will help with dryness, this instruction is not as important as reporting the yellowish drainage.

A patient with increased ICP has mannitol (Osmitrol) prescribed. Which option is the best indication that the drug is achieving the desired therapeutic effects? A. Urine output increases from 30 mL to 50 mL/hour. B. Blood pressure remains less than 150/90 mm Hg. C. The LOC improves. D. No crackles are auscultated in the lung fields.

C LOC is the most sensitive indicator of ICP. Mannitol is an osmotic diuretic that works to decrease the ICP by plasma expansion and an osmotic effect. Although the other options may indicate a therapeutic effect of a diuretic, they are not the main reason this drug is given.

A nurse assesses a client who is recovering from a transsphenoidal hypophysectomy. The nurse notes nuchal rigidity. Which action should the nurse take first? a. Encourage range-of-motion exercises. b. Document the finding and monitor the client. c. Take vital signs, including temperature. d. Assess pain and administer pain medication

C Nuchal rigidity is a major manifestation of meningitis, a potential postoperative complication associated with this surgery. Meningitis is an infection; usually the client will also have a fever and tachycardia. Range-of-motion exercises are inappropriate because meningitis is a possibility. Documentation should be done after all assessments are completed and should not be the only action. Although pain medication may be a palliative measure, it is not the most appropriate initial action.

When the nurse educator is evaluating the skills of a new registered nurse (RN) caring for patients experiencing shock, which action by the new RN indicates a need for more education? a. Placing the pulse oximeter on the ear for a patient with septic shock b. Keeping the head of the bed flat for a patient with hypovolemic shock c. Maintaining a cool room temperature for a patient with neurogenic shock d. Increasing the nitroprusside infusion rate for a patient with a very high SVR

C Patients with neurogenic shock have poikilothermia. The room temperature should be kept warm to avoid hypothermia. The other actions by the new RN are appropriate.

The nurse teaches a 28-year-old man newly diagnosed with hypertension about lifestyle modifications to reduce his blood pressure. Which statement by the patient requires an intervention by the nurse? A "I will avoid adding salt to my food during or after cooking." B "If I lose weight, I might not need to continue taking medications." C "I can lower my blood pressure by switching to smokeless tobacco." D "Diet changes can be as effective as taking blood pressure medications."

C "I can lower my blood pressure by switching to smokeless tobacco." Nicotine contained in tobacco products (smoking and chew) cause vasoconstriction and increase blood pressure. Persons with hypertension should restrict sodium to 1500 mg/day by avoiding foods high in sodium and not adding salt in preparation of food or at meals. Weight loss can decrease blood pressure between 5 to 20 mm Hg. Following dietary recommendations (such as the DASH diet) lowers blood pressure, and these decreases compare with those achieved with blood pressure-lowering medication.

The nurse is teaching a women's group about prevention of hypertension. What information should be included in the teaching for all the women (select all that apply)? A Lose weight. B Limit nuts and seeds. C Limit sodium and fat intake. D Increase fruits and vegetables. E Exercise 30 minutes most days.

C Limit sodium and fat intake. D Increase fruits and vegetables. E Exercise 30 minutes most days. Primary prevention of hypertension is to make lifestyle modifications that prevent or delay the increase in BP. Along with exercise for 30 minutes on most days, the DASH eating plan is a healthy way to lower BP by limiting sodium and fat intake, increasing fruits and vegetables, and increasing nutrients that are associated with lowering BP. Nuts and seeds and dried beans are used for protein intake. Weight loss may or may not be necessary for the individual.

The nurse is assessing their patient who was admitted for a UTI three days prior. Which of the following vital signs would cause the nurse to suspect that the patient may be going into sepsis? A) A respiratory rate of 14 and a heart rate of 82 bpm B) A temperature of 98.6 degrees F and a respiratory rate of 10 C) A blood pressure of 76/58 and a heart rate of 120 bpm D) An O2 saturation of 94% and a blood pressure of 140/96

C) A blood pressure of 76/58 and a heart rate of 120 bpm

A patient with squamous cell carcinoma has been scheduled for treatment of this malignancy. The nurse should anticipate that treatment for this type of cancer will primarily consist of what intervention? A) Chemotherapy B) Radiation therapy C) Surgical excision D) Biopsy of sample tissue

C) Surgical excision

A nurse is leading a health promotion workshop that is focusing on cancer prevention. What action is most likely to reduce participants' risks of basal cell carcinoma (BCC)? A) Teaching participants to improve their overall health through nutrition B) Encouraging participants to identify their family history of cancer C) Teaching participants to limit their sun exposure D) Teaching participants to control exposure to environmental and occupational radiation

C) Teaching participants to limit their sun exposure

In the administration of a drug such as levothyroxine (Synthroid), the nurse should teach the client: A) That therapy typically lasts about 6 months. B) That weekly laboratory tests for T4 levels will be required. C) To report weight loss, anxiety, insomnia, and palpitations. D) That the drug may be taken every other day if diarrhea occurs.

C) To report weight loss, anxiety, insomnia, and palpitations. As these are signs of hyperthyroidism, which is an adverse effect of levothyroxine.

A patient has been diagnosed with neutropenia. Which of the below precautions are correct? (SATA) A. Shared room B. Sideways precautions C. Reverse precautions D. No fresh fruits or vegetables E. No fresh flowers F. OK to use dirty supplies from another room.

C, D, E You did not really pick F, did you? If you did, you need to go to sleep. Studying too hard.. I know me too. OK, I am back to serious on the next question.

A client with a T1 spinal cord injury arrives at the emergency department with a BP of 82/40, pulse 34, 96.4 temp, and dry, warm extremities. Which of the following conditions would most likely be suspected? A. Autonomic dysreflexia. B. Hypervolemia. C. Neurogenic shock. D. Sepsis.

C. Neurogenic shock Loss of sympathetic control and unopposed vagal stimulation below the level of injury typically cause hypotension, bradycardia, pallor, and warm, dry skin in the extremities in the client in neurogenic shock. A: Autonomic dysreflexia occurs after neurogenic shock abates. B: Hypervolemia is indicated by rapid and bounding pulse and edema. D: Signs of sepsis would include elevated temperature, increased heart rate, and increased respiratory rate.

Which patient is at risk for hyperkalemia? A. Patient with Parathyroid cancer B. Patient with Cushing's Syndrome C. Patient with Addison's Disease D. Patient with breast cancer

C. Patient with Addison's Disease

A patient is presenting with an orthostatic blood pressure of 80/40 when she stands up, a thready and weak pulse of 58, and shallow respirations. In addition, the patient has been having frequent episodes of vomiting and nausea and is taking hydrochlorothiazide. Which of the following findings would explain the patient's condition? A. Potassium level of 7.0 B. Potassium level of 3.5 C. Potassium level of 2.4 D. None of the options are correct

C. Potassium level of 2.4

A patient is admitted with iron- deficiency anemia and has been receiving iron supplementation. The patient voices concern about how their stool is dark black. As the nurse, you would?* A. Notify the doctor B. Hold the next dose of iron C. Reassure the patient this is a normal side effect of iron supplementation D. None of the options are correct

C. Reassure the patient this is a normal side effect of iron supplementation

You are providing care for a patient who has been diagnosed with Guillain-Barré syndrome. Which assessment should you prioritize? A. Pain assessment B. Glasgow Coma Scale C. Respiratory assessment D. Musculoskeletal assessment

C. Respiratory assessment Although all of the assessments are necessary in the care of patients with Guillain-Barré syndrome, the acute risk of respiratory failure necessitates vigilant monitoring of the patient's respiratory status.

Which sensory-perceptual deficit is associated with left-sided stroke (right hemiplegia)? A. Overestimation of physical abilities. B. Difficulty judging position and distance. C. Slow and possibly fearful performance of tasks. D. Impulsivity and impatience at performing tasks.

C. Slow and possibly fearful performance of tasks. Patients with a left-sided stroke (right hemiplegia) commonly are slower in organization and performance of tasks and may have a fearful, anxious response to a stroke. Overconfidence, spatial disorientation, and impulsivity are more commonly associated with a right-sided stroke.

What is the primary goal of nursing care for the patient with Guillain-Barré syndrome? A. Assist the patient to adapt to their lifelong paralysis. B. Teach the patient to use a communication board. C. Support body systems until the patient recovers. D. Place the patient in contact isolation to prevent spread of the condition.

C. Support body systems until the patient recovers. The objective of therapy is to support body systems until the patient recovers. Respiratory failure and infection are serious threats. Most patients recover eventually. Depending on the progression of the disease, the patient may be incapable of communicating.

This cranial nerve arises from the cranial and spinal roots which controls swallowing movements and governs movement of the head and shoulders? A. IX B. X C. XI D. VII

C. X1 - Accessory Nerve

Glossopharyngeal nerve is responsible for? A. swallowing and speech B. hearing and equilibrium C. swallowing, taste, and secreting saliva D. none of the options are correct

C. swallowing, taste, and secreting saliva

The client was dx with iron-deficiency anemia is prescribed ferrous gluconate orally. Which should the nurse teach the client? A.Take Imodium, anti-diarrheal, and OTC for diarrhea B.Limit exercise for several weeks until tolerance is achieved C.Ferrous gluconate may cause constipation D.Eat only red meats and organ meats for protein

C.Ferrous gluconate may cause constipation.

Early this morning, a female client had a subtotal thyroidectomy. During evening rounds, nurse Tina assesses the client, who now has nausea, a temperature of 105° F (40.5° C), tachycardia, and extreme restlessness. What is the most likely cause of these signs? A. Diabetic ketoacidosis B. Thyroid crisis C. Hypoglycemia D. Tetany

Correct Correct Answer: B. Thyroid crisis Thyroid crisis usually occurs in the first 12 hours after thyroidectomy and causes exaggerated signs of hyperthyroidism, such as high fever, tachycardia, and extreme restlessness. Presentation of thyroid storm is an exaggerated manifestation of hyperthyroidism, with the presence of an acute precipitating factor. Fever, cardiovascular involvement (including tachycardia, heart failure, arrhythmia), central nervous system (CNS) manifestations, and gastrointestinal symptoms are common.

Signs of hypoglycemia include: A. Fruity breath, thirst, flushed skin B. Diarrhea, itching, hypertension C. Anxiety, weakness, pallor, sweating D. Muscle ache, fever, thirst

Correct Correct Answer: C. Anxiety, weakness, pallor, sweating These are signs of hypoglycemia, along with restlessness, chills, confusion, nausea, hunger, tachycardia, weakness, or headache. Neurogenic signs and symptoms can either be adrenergic (tremor, palpitations, anxiety) or cholinergic (hunger, diaphoresis, paresthesias). Neurogenic symptoms and signs arise from sympathoadrenal involvement (either norepinephrine or acetylcholine release) in response to perceived hypoglycemia.

A patient has a scald burn on the arm that is bright red, moist, and has several blisters. The nurse would classify this burn as which of the following? Select all that apply. 1. a superficial partial-thickness burn 2. a thermal burn 3. a superficial burn 4. a deep partial-thickness burn 5. a full-thickness burn

Correct Answer: 1,2 Rationale: Superficial partial-thickness burn if often bright red, has a moist, glistening appearance and blister formation. Thermal burns result from exposure to dry or moist heat. A superficial burn is reddened with possible slight edema over the area. A deep partial-thickness burn often appears waxy and pale and may be moist or dry. A full-thickness burn may appear pale, waxy, yellow, brown, mottled, charred, or non-blanching red with a dry, leathery, firm wound surface.

A patient is taking insulin glargine injection daily. The nurse instructed the client that the onset of action will likely happen? A. 2-4 hours after administration B. 4-12 hours after administration C. 6-12 hours after administration D. 18-24 hours after administration

Correct Answer: A. 2-4 hours after administration Insulin glargine is a long-acting insulin with an onset of 2-4 hours, no peak, and its duration of action is 24 hours.

Vasopressin is which of the following pituitary hormones? A. Antidiuretic hormone B. Desmopressin acetate C. Oxytocin D. ACTH

Correct Answer: A. Antidiuretic hormone Vasopressin is an antidiuretic hormone. Vasopressin or antidiuretic hormone (ADH) or arginine vasopressin (AVP) is a nonapeptide synthesized in the hypothalamus. Science has known it to play essential roles in the control of the body's osmotic balance, blood pressure regulation, sodium homeostasis, and kidney functioning. ADH primarily affects the ability of the kidney to reabsorb water; when present, ADH induces expression of water transport proteins in the late distal tubule and collecting duct to increase water reabsorption.

For the first 72 hours after thyroidectomy surgery, nurse Jamie would assess the female client for Chvostek's sign and Trousseau's sign because they indicate which of the following? A. Hypocalcemia B. Hypercalcemia C. Hypokalemia D. Hyperkalemia

Correct Answer: A. Hypocalcemia The client who has undergone a thyroidectomy is at risk for developing hypocalcemia from inadvertent removal or damage to the parathyroid gland. The client with hypocalcemia will exhibit a positive Chvostek's sign (facial muscle contraction when the facial nerve in front of the ear is tapped) and a positive Trousseau's sign (carpal spasm when a blood pressure cuff is inflated for a few minutes). These signs aren't present with hypercalcemia, hypokalemia, or hyperkalemia.

Which of the following statements involving Type II diabetes mellitus is correct? A. It involves inefficient insulin production. B. It involves cessation of Insulin production by the beta cells of the pancreas. C. It involves increased insulin receptor responsiveness. D. It involves the infant client.

Correct Answer: A. It involves inefficient insulin production. In type II diabetes mellitus, insulin is produced in insufficient amounts along with reduced insulin receptor responsiveness. T2DM involves a more insidious onset where an imbalance between insulin levels and insulin sensitivity causes a functional deficit of insulin. Insulin resistance is multifactorial but commonly develops from obesity and aging.

When caring for a patient who has had a head injury, which assessment information is of most concern to the nurse? a. The blood pressure increases from 120/54 to 136/62. b. The patient is more difficult to arouse. c. The patient complains of a headache at pain level 5 of a 10-point scale. d. The patient's apical pulse is slightly irregular.

Correct Answer: B Rationale: The change in level of consciousness (LOC) is an indicator of increased ICP and suggests that action by the nurse is needed to prevent complications. The change in BP should be monitored but is not an indicator of a need for immediate nursing action. Headache is not unusual in a patient after a head injury. A slightly irregular apical pulse is not unusual.

A male client has just been diagnosed with type 1 diabetes mellitus. When teaching the client and family how diet and exercise affect insulin requirements, Nurse Joy should include which guideline? A. "You'll need more insulin when you exercise or increase your food intake." B. "You'll need less insulin when you exercise or reduce your food intake." C. "You'll need less insulin when you increase your food intake." D. "You'll need more insulin when you exercise or decrease your food intake."

Correct Answer: B. "You'll need less insulin when you exercise or reduce your food intake." Exercise, reduced food intake, hypothyroidism, and certain medications decrease the insulin requirements. Growth, pregnancy, greater food intake, stress, surgery, infection, illness, increased insulin antibodies, and certain medications increase the insulin requirements. Exercise in patients with diabetes mellitus promotes cardiovascular benefits by reducing cardiovascular risk and mortality, assists with weight management, and it improves glycemic control. The increased tissue sensitivity to insulin produces a beneficial effect on glycemic control.

The nurse assesses a client who has bacterial pneumonia and finds​ tachycardia, hypotension,​ oliguria, and acrocyanosis of a foot. Schistocytes are found in a complete blood​ count, and the​ D-dimer is elevated. Which collaborative action should the nurse​ anticipate? A. Warfarin therapy B. Foot amputation C. Dialysis D. Heparin therapy

D

A client admitted to the hospital with a subarachnoid hemorrhage has complaints of severe headache, nuchal rigidity, and projectile vomiting. The nurse knows lumbar puncture (LP) would be contraindicated in this client in which of the following circumstances? A. Vomiting continues. B. Intracranial pressure (ICP) is increased. C. The client needs mechanical ventilation. D. Blood is anticipated in the cerebrospinal fluid (CSF).

Correct Answer: B. Intracranial pressure (ICP) is increased. Sudden removal of CSF results in pressures lower in the lumbar area than the brain and favors herniation of the brain; therefore, LP is contraindicated with increased ICP. A head computed tomogram (CT) should be obtained before performing a lumbar puncture if there is a concern for increased intracranial pressure. Signs and symptoms of possible increased intracranial pressure include altered mental status, focal neurological deficits, new-onset seizure, papilledema, immunocompromised state, malignancy, history of focal CNS disease (stroke, focal infection, tumor), concern for mass CNS lesion and age greater than 60 years old.

Nurse Casey is studying insulin administration. She should be knowledgeable that regular insulin: A. Is slow acting B. Is used IV C. Is a suspended insulin D. Peaks in 6 to 12 hours

Correct Answer: B. Is used IV Regular insulin is the only insulin preparation that can be administered IV. When administered intravenously, U-100 administration should be with close monitoring of serum potassium and blood glucose. Do not use if the solution is viscous or cloudy; administration should only take place if it is colorless and clear.

Drew is diagnosed with Type I diabetes mellitus. As a nurse taking care of the client, you should know that in his condition: A. Insulin is produced but is malformed. B. The beta cells of the pancreas stop producing insulin. C. The client cannot be treated. D. Diagnosis is made in clients over age 50.

Correct Answer: B. The beta cells of the pancreas stop producing insulin. In type I diabetes mellitus, the beta cells stop producing insulin completely. T1DM is characterized by the destruction of beta cells in the pancreas, typically secondary to the autoimmune destruction of beta cells. The result is the absolute destruction of beta cells, and consequentially, insulin is absent or extremely low.

When assessing a patient with a head injury, the nurse recognizes that the earliest indication of increased intracranial pressure (ICP) is a. vomiting. b. headache. c. change in level of consciousness (LOC). d. sluggish pupil response to light.

Correct Answer: C Rationale: LOC is the most sensitive indicator of the patient's neurologic status and possible changes in ICP. Vomiting and sluggish pupil response to light are later signs of increased ICP. A headache can be caused by compression of intracranial structures as the brain swells, but it is not unexpected after a head injury.

A patient with a head injury has an arterial blood pressure is 92/50 mm Hg and an intracranial pressure of 18 mm Hg. Which action by the nurse is appropriate? a. Document and continue to monitor the parameters. b. Elevate the head of the patient's bed. c. Notify the health care provider about the assessments. d. Check the patient's pupillary response to light.

Correct Answer: C Rationale: The patient's cerebral perfusion pressure is only 46 mm Hg, which will rapidly lead to cerebral ischemia and neuronal death unless rapid action is taken to reduce ICP and increase arterial BP. Documentation and monitoring are inadequate responses to the patient's problem. Elevating the head of the bed will lower the ICP but may also lower cerebral blood flow and further decrease CPP. Changes in pupil response to light are signs of increased ICP, so the nurse will only take more time doing this without adding any useful information

Nurse Ronn is assessing a client with possible Cushing's syndrome. In a client with Cushing's syndrome, the nurse would expect to find: A. Hypotension. B. Thick, coarse skin. C. Deposits of adipose tissue in the trunk and dorsocervical area. D. Weight gain in arms and legs.

Correct Answer: C. Deposits of adipose tissue in the trunk and dorsocervical area. Because of changes in fat distribution, adipose tissue accumulates in the trunk, face (moonface), and dorsocervical areas (buffalo hump). Physical examination of the patient will reveal increased fat deposits in the upper half of the body leading to "Buffalo torso," characteristic moon facies (earlobes are not visible when viewed from the front), thin arms and legs, acne, hirsutism, proximal muscle weakness of shoulder and hip girdle muscles, paper-thin skin, abdominal pain due to gut perforation in rare cases, and wide vertical purplish abdominal striae

A major side effect of insulin use that can be life threatening is: A. Hyperglycemia B. Stomach upset C. Hypoglycemia D. Tremors

Correct Answer: C. Hypoglycemia The action of insulin will lower glucose levels, which may prove fatal if levels drop too low. Hypoglycemia is, by far, the most common adverse effect of insulin therapy. The other adverse effects of insulin therapy include weight gain, and rarely electrolyte disturbances like hypokalemia, especially when used along with other drugs causing hypokalemia.

A client is admitted to the emergency room with a spinal cord injury. The client is complaining of lightheadedness, flushed skin above the level of the injury, and headache. The client's blood pressure is 160/90 mm Hg. Which of the following is a priority action for the nurse to take? A. Loosen tight clothing or accessories B. Assess for any bladder distention C. Raise the head of the bed D. Administer antihypertensive

Correct Answer: C. Raise the head of the bed The client is experiencing an autonomic dysreflexia, a life-threatening medical emergency that affects individuals with spinal injuries. Usually an individual with SCI has a blood pressure reading of 20 mm to 40 mm Hg above baseline. If this condition is suspected, the priority nursing action is to raise the head of bed or place the client in high Fowler's position. This promotes adequate ventilation and prevents the occurrence of hypertensive stroke.

A client with a subdural hematoma becomes restless and confused, with dilation of the ipsilateral pupil. The physician orders mannitol for which of the following reasons? A. To reduce intraocular pressure. B. To prevent acute tubular necrosis. C. To promote osmotic diuresis to decrease ICP. D. To draw water into the vascular system to increase blood pressure.

Correct Answer: C. To promote osmotic diuresis to decrease ICP. Mannitol promotes osmotic diuresis by increasing the pressure gradient, drawing fluid from intracellular to intravascular spaces. Although mannitol is used for all the reasons described, the reduction of ICP in this client is a concern. The mannitol causes the cells in the brain to dehydrate mildly. The water inside the brain cells (intracellular water) leaves the cells and enters the bloodstream as the mannitol draws it out of the cells and into the bloodstream. Once in the bloodstream, the extra water is whisked out of the skull. When the mannitol gets to the kidneys, the kidneys filter the mannitol into the urine.

Opportunistic diseases in HIV infection a. are usually benign. b. are generally slow to develop and progress. c. occur in the presence of immunosuppression. d. are curable with appropriate drug interventions.

Correct answer: b Rationale: The goals of drug therapy in HIV infection are to (1) decrease the viral load, (2) maintain or raise CD4+ T cell counts, and (3) delay onset of HIV infection-related symptoms and opportunistic diseases. Opportunistic diseases in HIV infection a. are usually benign. b. are generally slow to develop and progress. c. occur in the presence of immunosuppression. d. are curable with appropriate drug interventions. Correct answer: c Rationale: Management of HIV infection is complicated by the many opportunistic diseases that can develop as the immune system deteriorates (see Table 15-10).

A patient is recovering from second- and third-degree burns over 30% of his body and is now ready for discharge. The first action the nurse should take when meeting with the patient would be to: a. arrange a return-to-clinic appointment and prescription for pain medications b. teach the patient and caregiver proper wound care to be performed at home c. review the patient's current health care status and readiness for discharge to home d. give the patient written discharge information and websites for additional information for burn survivors.

Correct answer: c Rationale: Recovery from a burn injury to 30% of total body surface area (TBSA) takes time and is exhausting, both physically and emotionally, for the patient. The health care team may think that a patient is ready for discharge, but the patient may not have any idea that discharge is being contemplated in the near future. Patients are often very fearful about how they will manage at home. The patient would benefit from the nurse's careful review of his or her progress and readiness for discharge; then the nurse should outline the plans for support and follow-up after discharge.

Rest pain is a manifestation of PAD that occurs due to a chronic a. vasospasm of small cutaneous arteries in the feet. b. increase in retrograde venous blood flow in the legs. c. decrease in arterial blood flow to the nerves of the feet. d. decrease in arterial blood flow to the leg muscles during exercise.

Correct answer: c Rationale: Rest pain most often occurs in the forefoot or toes and is aggravated by limb elevation. Rest pain occurs when blood flow is insufficient to meet basic metabolic requirements of the distal tissues. Rest pain occurs more often at night because cardiac output tends to drop during sleep and the limbs are at the level of the heart. Patients often try to achieve partial pain relief by dangling the leg over the side of the bed or sleeping in a chair to allow gravity to maximize blood flow.

A 50-year-old woman weighs 95 kg and has a history of tobacco use, high blood pressure, high sodium intake, and sedentary lifestyle. When developing an individualized care plan for her, the nurse determines that the most important risk factors for peripheral artery disease (PAD) that need to be modified are: a. weight and diet. b. activity level and diet. c. tobacco use and high blood pressure. d. sedentary lifestyle and high blood pressure.

Correct answer: cRationale: Significant risk factors for peripheral artery disease include tobacco use, hyperlipidemia, elevated levels of high-sensitivity C-reactive protein, diabetes mellitus, and uncontrolled hypertension; the most important is tobacco use. Other risk factors include family history, hypertriglyceridemia, hyperuricemia, increasing age, obesity, sedentary lifestyle, and stress.

When assessing a patient with a partial-thickness burn, the nurse would expect to find (SATA): a. blisters b. exposed fascia c. exposed muscles d. intact nerve endings e. red, shiny, wet appearance

Correct answers: a, d, e Rationale: The appearance of partial-thickness (deep) burns may include fluid-filled vesicles (blisters) that are red, shiny, or wet (if vesicles have ruptured). Patients may have severe pain caused by exposure of nerve endings and may have mild to moderate edema.

Which clinical manifestations are seen in patients with either Buerger's disease or Raynaud's phenomenon (select all that apply)? a. Intermittent fevers b. Sensitivity to cold temperatures c. Gangrenous ulcers on fingertips d. Color changes of fingers and toes e. Episodes of superficial vein thrombosis

Correct answers: b, c, d Rationale: Both Buerger's disease and Raynaud's phenomenon have the following clinical manifestations in common: cold sensitivity, ischemic and gangrenous ulcers on fingertips, and color changes of the distal extremity (fingers or toes).

The client asks the nurse to explain what it means that his Hodgkin's disease is diagnosed at stage 1A. Which of the following describes the involvement of the disease? "1. Involvement of a single lymph node. 2. Involvement of two or more lymph nodes on the same side of the diaphragm. 3. Involvement of lymph node regions on both sides of the diaphragm. 4. Diffuse disease of one or more extralymphatic organs."

Correct: 1. In the staging process, the designations A and B signify, respectively, that symptoms were or were not present when Hodgkin's disease was found. The Roman numerals I through IV indicate the extent and location of involvement of the disease. Stage I indicates involvement of a single lymph node; stage II, two or more lymph nodes on the same side of the diaphragm; stage III, lymph node regions on both sides of the diaphragm; and stage IV, diffuse disease of one or more extralymphatic organs.

The mother of a 5-year-old child asks the nurse questions regarding the importance of vigilant use of sunscreen. Which information is most important for the nurse to convey to the mother? "A. Appropriate use of sunscreen decreases the risk of skin cancer. B. Repeated exposure to the sun causes premature aging of the skin. C. A child's skin is delicate, and burns easily. D. In addition to causing skin cancer, repeated sun exposure predisposes the child to other forms of cancer

Correct: A. Appropriate use of sunscreen decreases the risk of skin cancer. While all of the answer choices are correct, recommending the use of sunscreen to decrease the incidence of skin cancer is the best response. Nursing Process: Implementation Category of Client Need: Health Promotion and Maintenance Cognitive Level: Application

A nurse assesses a client shortly after living donor kidney transplant surgery. Which postoperative finding must the nurse report to the physician immediately? a) Serum sodium level of 135 mEq/L b) Serum potassium level of 4.9 mEq/L c) Temperature of 99.2° F (37.3° C) d) Urine output of 20 ml/hour

D) Urine output of 20 ml/hour Explanation: Because kidney transplantation carries the risk of transplant rejection, infection, and other serious complications, the nurse should monitor the client's urinary function closely. A decrease from the normal urine output of 30 ml/hour is significant and warrants immediate physician notification. A serum potassium level of 4.9 mEq/L, a serum sodium level of 135 mEq/L, and a temperature of 99.2° F are normal assessment findings.

Of the following patients, the nurse recognizes that the one with the highest risk for stroke is a(n): A. obese 45-year old Native American. B. 35-year-old Asian American woman who smokes. C. 32-year-old white woman taking oral contraceptives. D. 65-year-old African American man with hypertension.

D. 65-year-old African American man with hypertension. Nonmodifiable risk factors for stroke include age (older than 65 years), male gender, ethnicity or race (incidence is highest in African Americans; next highest in Hispanics, Native Americans/Alaska Natives, and Asian Americans; and next highest in white people), and family history of stroke or personal history of a transient ischemic attack or stroke. Modifiable risk factors for stroke include hypertension (most important), heart disease (especially atrial fibrillation), smoking, excessive alcohol consumption (causes hypertension), abdominal obesity, sleep apnea, metabolic syndrome, lack of physical exercise, poor diet (high in saturated fat and low in fruits and vegetables), and drug abuse (especially cocaine). Other risk factors for stroke include a diagnosis of diabetes mellitus, increased serum levels of cholesterol, birth control pills (high levels of progestin and estrogen), history of migraine headaches, inflammatory conditions, hyperhomocystinemia, and sickle cell disease.

You're assessing your patients during morning rounding. Which patient below is at MOST risk for developing a urinary tract infection? A. A 25-year-old patient who finished a regime of antibiotics for strep throat 10 weeks ago. B. A 55-year-old female who is post-opt day 7 from hip surgery. C. A 68-year-old male is experiencing nausea and vomiting. D. A 87-year-old female with Alzheimer's disease who is experiencing bowel incontinence.

D. A 87 year old female with Alzheimer's disease who is experiencing bowel inconti

What is the indicatio for potassium sparing diuretic?

Heart Failure. Hypertension. Edema Associated with Hepatic Cirrhosis or Nephrotic Syndrome. Primary Hyperaldosteronism. Cushing's (hypokalemia)

The nurse is caring for a patient with hypovo- lemia secondary to severe diarrhea and vomit- ing. In evaluating the respiratory system for this patient, what does the nurse expect to assess? a. No changes, because the respiratory sys- tem is not involved b. Hypoventilation, because the respiratory system is trying to compensate for low pH c. Increased respiratory rate, because the body perceives hypovolemia as hypoxia d. Normal respiratory rate, but a decreased oxygen saturation

Increased respiratory rate, because the body perceives hypovolemia as hypoxia

Patients taking levothyroxine (Synthroid) and warfarin (Coumadin) concurrently would be monitored for which adverse effect?

Increased risk of bleeding.

The spouse of a client diagnosed with delirium tells the​ nurse, "I'm not sure what caused​ this, as my spouse has never had any mental issues​ before." Which potential cause should the nurse include in the response to the​ client's spouse?​ (Select all that​ apply.) Infection Thyroid disease Sleep deprivation Drug or alcohol use Irritable bowel disease

Infection, Sleep deprivation, and Drug or alcohol use

A 68-year-old patient who is hospitalized with pneumonia is disoriented and confused 3 days after admission. Which information indicates that the patient is experiencing delirium rather than dementia? a. The patient was oriented and alert when admitted. b. The patient's speech is fragmented and incoherent. c. The patient is oriented to person but disoriented to place and time. d. The patient has a history of increasing confusion over several years.

NS: A-The patient was oriented and alert when admitted The onset of delirium occurs acutely. The degree of disorientation does not differentiate between delirium and dementia. Increasing confusion for several years is consistent with dementia. Fragmented and incoherent speech may occur with either delirium or dementia.

As the nurse you know that there is a risk of a transfusion reaction during the administration of red blood cells. Which patient below it is at most RISK for a febrile (non-hemolytic) transfusion reaction? A. A 38 year old male who has received multiple blood transfusions in the past year. B. A 42 year old female who is immunocompromised. C. A 78 year old male who is B+ that just received AB+ blood during a transfusion. D. A 25 year old female who is AB+ and just received B+ blood.

The answer is A. A febrile transfusion reaction is where the recipient's WBCs are reacting with the donor's WBCs. This causes the body to build antibodies. It is most COMMON in patients who have received blood transfusion in the past. Option B is at risk for GvHD (graft versus host disease). Option C is wrong because this places the patient at risk for a hemolytic transfusion reaction (not febrile). The patient is receiving incompatible blood. However, option D is not the patient at MOST risk compared to option A. Note the patient is receiving compatible blood. Note the patient is receiving compatible blood in this option.

Which of the following signs and symptoms causes concern and requires nursing intervention for a patient who recently had a thyroidectomy? A. Heart rate of 120, blood pressure 220/102, temperature 103.2 'F B. Heart rate of 35, blood pressure 60/43, temperature 95.3 'F C. Soft hair, irritable, diarrhea D. Constipation, drowsiness, goiter

The answer is A. A patient is at risk for experiencing thyroid storm after a thyroidectomy because of manipulation of the thryroid gland that could cause excessive T3 and T4 to enter into the bloodstream during removal of the gland. Therefore, heart rate of 120, blood pressure 220/102, temperature 103.2 'F are classic signs of thyroid storm and this requires nursing intervention.

A 6 year old male is diagnosed with nephrotic syndrome. In your nursing care plan you will include which of the following as a nursing diagnosis for this patient? A. Risk for infection B. Deficient fluid volume C. Constipation D. Overflow urinary incontinence

The answer is A. A patient with nephrotic syndrome is at risk for infection due to the potential loss of proteins (immunoglobulins) in the urine that help fight infection. In addition, medication treatment for nephrotic syndrome may include corticosteroids or immune suppressors, which will further suppress the immune system. Option B is wrong because the patient will be experiencing fluid volume overload (not deficient). Option C and D are wrong because constipation and overflow urinary incontinence are not common findings with nephrotic syndrome.

You're assessing a patient who recently experienced a focal type seizure (partial seizure). As the nurse, you know that which statement by the patient indicates the patient may have experienced a focal impaired awareness (complex partial) seizure? A. "My friend reported that during the seizure I was staring off and rubbing my hands together, but I don't remember doing this." B. "I remember having vision changes, but it didn't last long." C. "I woke up on the floor with my mouth bleeding." D. "After the seizure I was very sleepy, and I had a headache for several hours."

The answer is A. The patient will experience an alternation in consciousness (hence the name focal IMPAIRED awareness) AND will perform an action without knowing they are doing it called automatism like lip-smacking, rubbing the hands together etc. With a focal onset AWARE seizure (also called partial simple seizure) the patient is aware and will remember what happens (like vision changes etc.).

Myasthenia gravis occurs when antibodies attack the __________ receptors at the neuromuscular junction leading to ____________.* A. metabotropic; muscle weakness B. nicotinic acetylcholine; muscle weakness C. dopaminergic adrenergic; muscle contraction D. nicotinic adrenergic; muscle contraction

The answer is B. In myasthenia gravis, either the nicotinic acetylcholine receptors are attacked by antibodies created by the immune system (hence why this disease is considered autoimmune) or antibodies are inhibiting the function of muscle-specific kinase (which is a receptor tyrosine kinase that helps with maintaining and building the neuromuscular junction). Either way this leads to the neurotransmitter acetylcholine from being able to communicate with the muscle fiber to make it contract.

Which patient below is at MOST risk for developing a condition called autonomic dysreflexia?* A. A 24-year-old male patient with a traumatic brain injury. B. A 15-year-old female patient with a spinal cord injury at C7. C. A 35-year-old male patient with a spinal cord injury at L6. D. A 42-year-old male patient recovering from a hemorrhagic stroke.

The answer is B. Patients who are at MOST risk for developing autonomic dysreflexia are patients who've experienced a spinal cord injury at T6 or higher...this includes C7. L6 is below T6, and traumatic brain injury and hemorrhagic stroke does not increase a patient risk of AD.

You're a home health nurse providing care to a patient with myasthenia gravis. Today you plan on helping the patient with bathing and exercising. When would be the best time to visit the patient to help these tasks?* A. Mid-afternoon B. Morning C. Evening D. Before bedtime

The answer is B. Patients with MG tend to have the best muscle strength in the morning after sleeping or resting rather than at the end of the day....the muscles are tired from being used and the muscle become weaker as the day progresses etc. Therefore any rigorous activities are best performed in the morning or after the patient has rested.

You're providing care to a 6 year old male patient who is receiving treatment for nephrotic syndrome. Which assessment finding below requires you to notify the physician immediately? A. Frothy, dark urine B. Redden area on the patient's left leg that is swollen and warm C. Elevated lipid level on morning labs D. Urine test results that shows proteinuria

The answer is B. Patients with nephrotic syndrome are at risk for hypercoagulability (blood clot formation) due to the loss of proteins in the urine that prevent blood clot formation. Option B represents a possible deep vein thrombosis, which will appear as a redden, warm, and swollen area on the extremity. Options A, C, and D are common findings with nephrotic syndrome, which are expected.

As the nurse, you know that it is important to implement a low sodium diet for a patient with nephrotic syndrome. However, it is important to implement what other type of diet due to another complication associated with this syndrome? A. Low-phosphate B. Low-fat C. High-carbohydrate D. Low-potassium

The answer is B. Patients with nephrotic syndrome can experience hyperlipidemia. Why? Remember that in this condition there will be low amounts of albumin in the blood. This decrease of albumin in the blood causes the liver to make more albumin, BUT while it does this it also makes more cholesterol and triglycerides...hence increasing lipid levels. Therefore, the patient should follow a low-sodium and low-fat diet as well.

The Human Immunodeficiency Virus (HIV) mainly attacks what type of cells in the human body?(Required) A. Red Blood Cells B. CD4 positive cells C. Stem Cells D. Platelets

The answer is B. The HIV virus attacks the human body's immune system, specifically the CD4 positive cells...mainly the helper t cells. These cells are white blood cells that help the immune system fight infection.

While assessing a patient with Parkinson's Disease, you note the patient's arms slightly jerk as you passively move them toward the patient's body. This is known as:* A. Lead Pipe Rigidity B. Cogwheel Rigidity C. Pronate Rigidity D. Flexor Rigidity

The answer is B. This is known as cogwheel rigidity, and occurs when the arms are passively moved, which will cause them to jerk slightly.

Your patient is having a transfusion reaction. You immediately stop the transfusion. Next you will:* A. Notify the physician. B. Disconnect the blood tubing from the IV site and replace it with a new IV tubing set-up and keep the vein open with normal saline 0.9%. C. Collect urine sample. D. Send the blood tubing and bag to the blood bank.

The answer is B. This question wants to know your NEXT nursing action. AFTER stopping the transfusion, the nurse will DISCONNECT the blood tubing from the IV site and replace it with a new IV tubing set-up and keep the vein open with normal saline 0.9%. This will limit any more blood from entering the patient's system. THEN the nurse will notify the MD and blood bank.

You're developing discharge instructions to the parents of a child who experiences atonic seizures. What information below is important to include in the teaching? A. "This type of seizure is hard to detect because the child may appear like he or she is daydreaming." B. "Be sure your child wears a helmet daily." C. "It is common for the child to feel extremely tired after experiencing this type of seizure." D. "Avoid high fat and low carbohydrate diets."

The answer is B. This type of seizure leads to a sudden loss of muscle tone. The patient will go limp and fall, which when this happens the head is usually the first part of the body to hit the floor or an object nearby. It is important the child wears a helmet daily to protect their head from injury. Option A is a characteristic of an absence seizure. Option C is a characteristic of a tonic-clonic seizure during the post ictus stage. And option D is wrong because some patients benefit from this type of diet known as the ketogenic diet.

A 7-year-old male patient is being evaluated for seizures. While in the child's room talking with the child's parents, you notice that the child appears to be daydreaming. You time this event to be 10 seconds. After 10 seconds, the child appropriately responds and doesn't recall the event. This is known as what type of seizure? A. Focal Impaired Awareness (complex partial) B. Atonic C. Tonic-clonic D. Absence

The answer is D. This is an absence seizure and is most common in children. The hallmark of it is staring that appears to be like a daydreaming state. It is very short and the post ictus stage of this type of seizure is immediate.

A patient has an extremely high T3 and T4 level. Which of the following signs and symptoms DO NOT present with this condition?* A. Weight loss B. Intolerance to heat C. Smooth skin D. Hair loss

The answer is D: Hair loss

You're performing a head-to-toe assessment on a patient with a spinal cord injury at T6. The patient is restless, sweaty, and extremely flushed. You assess the patient's blood pressure and heart rate. The patient's blood pressure is 140/98 and heart rate is 52. You look at the patient's chart and find that their baseline blood pressure is 106/76 and heart rate is 72. What action should the nurse take FIRST?* A. Reassess the patient's blood pressure. B. Check the patient's blood glucose. C. Position the patient at 90 degrees and lower the legs. D. Provide cooling blankets for the patient.

The answer is C. Based on the patient findings and how the patient has a spinal cord injury at T6, they are experiencing autonomic dysreflexia. Patients with this condition may have a blood pressure that is 20-40 mmHg higher than their baseline and may experience bradycardia (heart rate less than 60). The FIRST action the nurse should take when AD is suspected is to position the patient at 90 degree (high Fowler's) and lower the legs. This will allow gravity to cause the blood to pool in the lower extremities and help decrease the blood pressure. Then the nurse should try to find the cause of the autonomic dysreflexia, which could be a full bladder, impacted bowel, or skin break down.

A patient is in anaphylactic shock. The patient has a severe allergy to peanuts and mistakenly consumed an egg roll containing peanut ingredients during his lunch break. The patient is given Epinephrine intramuscularly. As the nurse, you know this medication will have what effect on the body? A. It will prevent a recurrent attack. B. It will cause vasoconstriction and increase blood pressure. C. It will help dilate the airways. D. It will help block the effects of histamine in the body.

The answer is C. Epinephrine acts as a vasopressor and will actually dilate the airway. Epinephrine performs vasoconstriction which will INCREASE the blood pressure. It does not prevent a recurrent attack (corticosteroids may help with this), and it does not block the effects of histamine (antihistamine helps with this).

Your patient is having a sudden and severe anaphylactic reaction to a medication. You immediately stop the medication and call a rapid response. The patient's blood pressure is 80/52, heart rate 120, and oxygen saturation 87%. Audible wheezing is noted along with facial redness and swelling. As the nurse you know that the first initial treatment for this patient's condition is? A. IV Diphenhydramine B. IV Normal Saline Bolus C. IM Epinephrine D. Nebulized Albuterol

The answer is C. IM or subq Epinephrine is the first-line treatment for anaphylaxis. Epinephrine will cause vasoconstriction (this will increase the blood pressure and decrease swelling) and bronchodilation (this will dilate the airways). This patient's cardiovascular and respiratory system is compromised. Therefore, epinephrine will provide fast relief with anaphylaxis.

Your patient has a history of epilepsy. While helping the patient to the restroom, the patient reports having this feeling of déjà vu and seeing spots in their visual field. Your next nursing action is to? A. Continue assisting the patient to the restroom and let them sit down. B. Initiate the emergency response system. C. Lay the patient down on their side with a pillow underneath the head. D. Assess the patient's medication history.

The answer is C. The patient is reporting signs and symptoms of an aura (this is a warning sign before a seizure event). Lay the patient down on their side with a pillow underneath the head and remove any restrictive clothing. Also, time the seizure. If the seizure lasts more than 5 minutes or if the patient starts to have seizures back-to-back activate the emergency response system.

You're collecting a urine sample on a patient who is experiencing proteinuria due to nephrotic syndrome. As the nurse, you know the urine will appear: A. Tea-colored B. Orange and frothy C. Dark and foamy D. Straw-colored

The answer is C. The urine will appear dark and foamy due to the high amount of proteins present in the urine. Remember in nephrotic syndrome the patient is losing a massive amount of protein per day (3 grams per day) and this will cause the urine to foam.

A patient with Parkinson's Disease has slow movements that affects their swallowing, facial expressions, and ability to coordinate movements. As the nurse you will document the patient has:* A. Akinesia B. "Freeze up" tremors C. Bradykinesia D. Pill-rolling

The answer is C. This is known as bradykinesia.

Your patient, who has a spinal cord injury at T3, states they are experiencing a throbbing headache. What is your NEXT nursing action?* A. Perform a bladder scan B. Perform a rectal digital examination C. Assess the patient's blood pressure D. Administer a PRN medication to alleviate pain and provide a dark, calm environment.

The answer is C. This is the nurse's NEXT action. The patient is at risk for developing autonomic dysreflexia because of their spinal cord injury at T3 (remember patients who have a SCI at T6 or higher are at MOST risk). If a patient with this type of injury states they have a headache, the nurse should NEXT assess the patient's blood pressure. If it is elevated, the nurse would take measures to check the bladder (a bladder issue is the most common cause of AD), bowel, and skin for breakdown.

Keeping the previous question in mind, the patient is now experiencing characteristics of a tonic-clonic seizure. The seizure started at 1402 and it is now 1408, and the patient is still experiencing a seizure. The nurse should? A. Continue to monitor the patient B. Suction the patient C. Initiate the emergency response system D. Restrain the patient to prevent further injury

The answer is C. Tonic-clonic seizures should last about 1-3 minutes. If the seizure lasts MORE than 5 minutes, the patient needs medical treatment FAST to stop the seizure....this is known as status epilepticus.

A patient is being discharged home for treatment of hypothyroidism. Which medication is most commonly prescribed for this condition? A. Tapazole B. PTU (Propylthiouracil) C. Synthroid (levothyroxine) D. Inderal

The answer is C: Synthroid (levothyroxine) is the only medication listed that treats hypothyroidism. All the other medications are used for hyperthyroidism.

A patient who needs a unit of packed red blood cells is ordered by the physician to be premeditated with oral diphenhydramine and acetaminophen. You will administer these medications?* A. 15 minutes before starting the transfusion B. Immediately after starting the transfusion C. Right before starting the transfusion D. 30 minutes before starting the transfusion

The answer is D. For ORAL medications you will administer the medications 30 minutes before starting the transfusion.

You're providing education to a patient, who has a severe peanut allergy, on how to recognize the signs and symptoms of anaphylactic shock. Select all the signs and symptoms associated with anaphylactic shock: A. Hyperglycemia B. Difficulty speaking C. Angioedema D. Hypertension E. Dyspnea F. Itchy G. Vomiting and Nausea H. Fever I. Slow heart rate

The answers are B, C, E, F, and G. Patients who are in anaphylactic shock will have signs and symptoms associated with the effects of histamine. Remember histamine affects the respiratory, cardiac, GI and skin. The patient can have the following: Respiratory: dyspnea and wheezing (bronchoconstriction), swelling of upper airways due to edema "tightness"...can't speak, coughing, stuffy nose, watery eyes, Cardiac: tachycardia, hypotension (vasodilation)...loss of consciousness, dizzy, GI: vomiting, nausea, pain, Skin: vasodilation...red, swollen, itchy, hives

Nmeumonic Delerium Causes DELIRIUM:

The main causes of delirium are outlined by the acronym DELIRIUM: Drugs, including any new medications, increased dosages, drug interactions, over-the-counter drugs, alcohol, etc. Electrolyte disturbances, especially dehydration, and thyroid problems. Lack of drugs, such as when long-term sedatives (including alcohol and sleeping pills) are stopped, or when pain drugs are not being given adequately. Infection, commonly urinary or respiratory tract infection. Reduced sensory input, which happens when vision or hearing are poor. Intracranial (referring to processes within the skull) such as a brain infection, hemorrhage, stroke, or tumor (rare). Urinary problems or intestinal problems, such as constipation or inability to urinate. Myocardial (heart) and lungs, such as heart attack, problems with heart rhythm (arrhythmia), worsening of heart failure, or chronic obstructive lung disease.

In the administration of a drug such as levothyroxine (Synthroid), the nurse must teach the client: (Select all that apply.) a.) Therapy could take three weeks or longer. b.) Periodic lab tests for T4 levels are required. c.) Report weight loss, anxiety, insomnia, and palpitations. d.) Jaundice

Therapy could take three weeks or longer.Periodic lab tests for T4 levels are required.Report weight loss, anxiety, insomnia, and palpitations.A,B,C

The nurse is assessing a client suspected of having acute disseminated intravascular coagulation​ (DIC). Which assessment finding supports the​ diagnosis? (Select all that​ apply.) A. Multiple bruises on various skin surfaces B. A history of a malignant tumor C. Bleeding at the IV insertion site D. A history of thyroid disease E. Pale, cool extremities

a, c, e

Which of the following may be included in the diet of the client with chronic renal failure? a) orange slices b) watermelon slices c) cantaloupe slices d) apple slices

Which of the following may be included in the diet of the client with chronic renal failure? a) orange slices b) watermelon slices c) cantaloupe slices d) apple slices

A patient with type 2 diabetes is scheduled for a follow-up visit in the clinic several months from now. Which test will the nurse schedule to evaluate the effectiveness of treatment for the patient? a. Fasting blood glucose b. Oral glucose tolerance c Glycosylated hemoglobin d. Urine dipstick for glucose

_ANS: CThe glycosylated hemoglobin (A1C) test shows the overall control of glucose over 90 to 120 days. A fasting blood level indicates only the glucose level at one time. Urine glucose testing is not an accurate reflection of blood glucose level and does not reflect the glucose over a prolonged time. Oral glucose tolerance testing is done to diagnose diabetes but is not used for monitoring glucose control after diabetes has been diagnosed.

Which of the following is an expected finding in the client with chronic renal failure? a) anemia b) polyuria c) increased creatinine clearance d) increased serum calcium levels

a) anemia

The nurse instructs a client with renal failure who is receiving hemodialysis about dietary modifications. The nurse determines that the client understands these dietary modifications if the client selects which items from the dietary menu? a) cream of wheat, blueberries, coffee b) sausage and eggs, banana, orange juice c) bacon, cantaloupe melon, tomato juice d) cured pork, strawberries, orange juice

a) cream of wheat, blueberries, coffee - the diet for a client with renal failure who is receiving hemodialysis should include controlled amounts of sodium, phosphorus, calcium, potassium, and fluids. Option B, C, and D are high in sodium, phosphorus, and potassium.

The client hemodialyzed suddenly becomes short of breath and complains of chest pain. The client is tachycardic, pale and anxious. The nurse suspects air embolism. The priority action for the nurse is to: a) discontinue dialysis and notify the physician b) monitor vital signs every 15 minutes for the next hour c) continue dialysis at a slower rate after checking the lines for air d) bolus the client with 500 ml of normal saline to break up the air embolus

a) discontinue dialysis and notify the physician

A physician prescribes levothyroxine sodium (Synthroid), 0.15 mg orally daily, for a client with hypothyroidism. The nurse will prepare to administer this medication: a) in the morning to prevent insomnia b) only when the client complains of fatigue and cold intolerance c) at various times during the day to prevent tolerance from occurring d) three times daily in equal doses of 0.5 mg each to ensure consistent serum drug levels

a) in the morning to prevent insomnia Levothyroxine (Synthroid) is a synthetic thyroid hormone that increases cellular metabolism. Levothyroxine should be given in the morning in a single dose to prevent insomnia and should be given at the same time each day to maintain an adequate drug level. Therefore, options B, C, and D are incorrect

Which of the following should the nurse include in the nursing care plan of the client who is diagnosed to have renal failure, whose BUN is 32 mg/dl, serum creatinine is 4 mg/dl, hematocrit is 38%. He is complaining of fatigue and edema. a) low protein diet and fluid restriction b) high protein diet and fluid restriction c) low protein diet and increase in fiber d) high protein diet and potassium restriction

a) low protein diet and fluid restriction

The client has end-stage renal disease. He had undergone kidney transplant 5 days ago. Which of the following is the most important intervention for the client to prevent infection? a) observe asepsis b) increase fluid intake c) avoid clients with flu d) avoid crowded places

a) observe asepsis It is important for the client to avoid crowds when possible once discharged but after just 5 days, observe asepsis is the best choice.

The client with chronic renal failure is on chronic hemodialysis. Which of the following indicate improvement of the client's condition due to hemodialysis? Select all that apply a) the client's BP is 130/90 b) the client's serum potassium is 4.8 mEq/L c) the client's hemoglobin level is 10 g/dL d) the client's serum calcium is 7.7 mg/dL e) the client's serum sodium is 140 mEg/L f) the client's serum magnesium is 4 mEq/L g) the client's weight has increased from 60 kg to 63 kg

a) the client's BP is 130/90 b) the client's serum potassium is 4.8 mEq/L e) the client's serum sodium is 140 mEg/L

A patient is admitted to the emergency department with first- and second-degree burns after being involved in a house fire. Which assessment findings would alert you to the presence of an inhalation injury? (select all that apply)? a. singed nasal hair b. generalized pallor c. painful swallowing d. burns on the upper extremities e. history of being involved in a large fire

a, b, c, e Reliable clues to the occurrence of inhalation injury is the presence of facial burns, singed nasal hair, hoarseness, painful swallowing, darkened oral and nasal membranes, carbonaceous sputum, history of being burned in an enclosed space, altered mental status, and "cherry red" skin color.

The nurse is planning care for a patient with partial- and full-thickness skin destruction related to burn injury of the lower extremities. Which interventions should the nurse expect to include in this patient's care ()? (select all that apply)? a. escharotomy b. administration of diuretics c. IV and oral pain medications d. daily cleansing and debridement e. application of topical antimicrobial agent

a, c, d, e An escharotomy (a scalpel incision through full-thickness eschar) is frequently required to restore circulation to compromised extremities. Daily cleansing and debridement as well as application of an antimicrobial ointment are expected interventions used to minimize infection and enhance wound healing. Pain control is essential in the care of a patient with a burn injury. With full-thickness burns, myoglobin and hemoglobin released into the bloodstream can occlude renal tubules. Adequate fluid replacement is used to prevent this occlusion.

The patient with cancer is having chemotherapy treatments and has now developed neutropenia. What care should the nurse expect to provide and teach the patient about (select all that apply)? a. Strict hand washing b. Daily nasal swabs for culture c. Monitor temperature every hour. d. Daily skin care and oral hygiene e. Encourage eating all foods to increase nutrients. f. Private room with a high-efficiency particulate air (HEPA) filter

a, d, f: Strict hand washing and daily skin and oral hygiene must be done with neutropenia, because the patient is predisposed to infection from the normal body flora, other people, and uncooked meats, seafood, eggs, unwashed fruits and vegetables, and fresh flowers or plants. The private room with HEPA filtration reduces the aerosolized pathogens in the patient's room. Blood cultures and antibiotic treatment are used when the patient has a temperature of 100.4° F or more, but temperature is not monitored every hour.

Which statement accurately describes Graves' disease? a. Exophthalmos occurs in Graves' disease. b. It is an uncommon form of hyperthyroidism. c. Manifestations of hyperthyroidism occur from tissue desensitization to the sympathetic nervous system. d. Diagnostic testing in the patient with Graves' disease will reveal an increased thyroid-stimulating hormone (TSH) level.

a. Exophthalmos or protrusion of the eyeballs may occur in Graves' disease from increased fat deposits and fluid in the orbital tissues and ocular muscles, forcing the eyeballs outward. Graves' disease is the most common form of hyperthyroidism. Increased metabolic rate and sensitivity of the sympathetic nervous system lead to the clinical manifestations. Thyroid-stimulating hormone (TSH) level is decreased in Graves' disease.

The nurse is caring for a teenage client with thrombocytopenia. Which statement indicates that the teenager understands the nurse's teaching about thrombocytopenic precautions? a. I can play golf but not football when I return to school. b. I can use a disposable razor when I shave. c. I will take ibprophen if I have a headache. d. I will use a hard bristled toothbush to brush my teeth.

a. I can play golf but not football when I return to school.

The patient with type 1 diabetes mellitus is having a seizure. Which medication should the nurse anticipate will be administered first? a. IV dextrose solution b. IV diazepam (Valium) c. IV phenytoin (Dilantin) d. Oral carbamazepine (Tegretol)

a. IV dextrose solution This patient's seizure could be caused by low blood glucose, so IV dextrose solution would be given first to stop the seizure. IV diazepam, IV phenytoin, and oral carbamazepine would be used first to treat seizures from other causes such as head trauma, drugs, and infections. These drugs will be tried if the IV dextrose is ineffective.

When caring for a patient with thrombocytopenia, the nurse instructs the patient to a. dab his or her nose instead of blowing. b. be careful when shaving with a safety razor. c. continue with physical activities to stimulate thrombopoiesis. d. avoid aspirin because it may mask the fever that occurs with thrombocytopenia.

a. dab his or her nose instead of blowing.

A major goal of treatment for the patient with AD is to a. maintain patient safety. b. maintain or increase body weight. c. return to a higher level of self-care. d. enhance functional ability over time.

a. maintain patient safety. Rationale: The overall management goals are that the patient with AD will (1) maintain functional ability for as long as possible, (2) be maintained in a safe environment with a minimum of injuries, (3) have personal care needs met, and (4) have dignity maintained. The nurse should place emphasis on patient safety while planning and providing nursing care.

Antidote for heparin overdose a. protamine sulfate b. vitamin K c. vitamin E d. cyanocobolamine

a. protamine sulfate

In the administration of hydrocortisone, it is vital that the nurse recognize that this drug might mask which symptoms? a.) Signs and symptoms of infection b.) Signs and symptoms of heart failure c.) Hearing loss d.) Skin infections

a.) Signs and symptoms of infection

During the care of the patient with a burn in the acute phase, which new interventions should the nurse expect to do after the patient progressed from the emergent phase? a. begin IV fluid replacement b. monitor for signs of complications c. access and manage pain and anxiety d. discuss possible reconstructive surgery

b Monitoring for complications (e.g., wound infection, pneumonia, contractures) is needed in the acute phase. Fluid replacement occurs in the emergent phase. Assessing and managing pain and anxiety occurs in the emergent and the acute phases. Discussing possible reconstructive surgeries is done in the rehabilitation phase.

For a patient who is suspected of having a stroke, one of the most important pieces of information that the nurse can obtain is a. time of the patient's last meal. b. time at which stroke symptoms first appeared. c. patient's hypertension history and management. d. family history of stroke and other cardiovascular diseases.

b Rationale: During initial evaluation, the most important point in the patient's history is the time since onset of stroke symptoms. If the stroke is ischemic, recombinant tissue plasminogen activator (tPA) must be administered within 3 to 4.5 hours of the onset of clinical signs of ischemic stroke; tPA reestablishes blood flow through a blocked artery and prevents brain cell death in patients with acute onset of ischemic stroke.

A client has been admitted to the hospital for urinary tract infection and dehydration. The nurse determines that the client has received adequate volume replacement if the blood urea nitrogen level drops to: a) 3 mg/dL b) 15 mg/dL c) 29 mg/dL d) 35 mg/dL

b) 15 mg/dL

You're assisting a physician with sickle cell anemia screening. As the nurse you know that which patient population listed below is at risk for sickle cell disease? a) Native Americans b) African-Americans c) Pacific Islanders d) Latino

b) African-Americans - The answer is B. Sickle cell anemia is most common in African-Americans along with Middle Eastern, Asian, Caribbean, and Eastern Mediterranean. WHY? According to the CDC, 1 in 12 African-Americans have the sickle cell trait, so it can easily be passed to their offspring. Remember if both parents have sickle cell trait there is a 25% chance they will pass it to their child.

The patient in the emergent phase of a burn injury is being treated for pain. What medication should the nurse anticipate using for this patient? a. SQ tetanus toxoid b. IV morphine sulfate c. IM hydromorphone d. PO oxycodone and acetaminophen

b. IV medications are used for burn injuries in the emergent phase to rapidly deliver relief and prevent unpredictable absorption as would occur with the IM route. The PO route is not used because GI function is slowed or impaired due to shock or paralytic ileus, although oxycodone and acetaminophen may be used later in the patient's recovery. Tetanus toxoid may be administered but not for pain.

A patient with a tremor is being evaluated for Parkinson's disease. The nurse explains to the patient that Parkinson's disease can be confirmed by a. CT and MRI scans. b. relief of symptoms with administration of dopaminergic agents. c. the presence of tremors that increase during voluntary movement. d. cerebral angiogram that reveals the presence of cerebral atherosclerosis.

b. Although clinical manifestations are characteristic in PD, no laboratory or diagnostic tests are specific for the condition. A diagnosis is made when at least two of the three signs of the classic triad are present and it is confirmed with a positive response to antiparkinsonian medication. Research regarding the role of genetic testing and MRI to diagnose PD is ongoing. Essential tremors increase during voluntary movement whereas the tremors of PD are more prominent at rest.

A patient with hypothyroidism is treated with levothyroxine (Synthroid). What should the nurse include when teaching the patient about this therapy? a. Explain that alternate-day dosage may be used if side effects occur. b. Provide written instruction for all information related to the drug therapy. c. Assure the patient that a return to normal function will occur with replacement therapy. d. Inform the patient that the drug must be taken until the hormone balance is reestablished.

b. Because of the mental sluggishness, inattentiveness, and memory loss that occur with hypothyroidism, it is important to provide written instructions and repeat information when teaching the patient. Replacement therapy must be taken for life and alternate-day dosing is not therapeutic. Although most patients return to a normal state with treatment, cardiovascular conditions and psychoses may persist.

When providing care for a patient with ALS, the nurse recognizes what as one of the most distressing problems experienced by the patient? a. Painful spasticity of the face and extremities b. Retention of cognitive function with total degeneration of motor function c. Uncontrollable writhing and twisting movements of the face, limbs, and body d. Knowledge that there is a 50% chance the disease has been passed to any offspring

b. In ALS there is gradual degeneration of motor neurons with extreme muscle wasting from lack of stimulation and use. However, cognitive function is not impaired and patients feel trapped in a dying body. Chorea manifested by writhing, involuntary movements is characteristic of HD. As an autosomal dominant genetic disease, HD also has a 50% chance of being passed to each offspring.

Which of these laboratory values noted by the nurse when reviewing the chart of a diabetic patient indicates the need for further assessment of the patient? a. Fasting blood glucose of 130 mg/dl b. Noon blood glucose of 52 mg/dl c. Glycosylated hemoglobin of 6.9% d. Hemoglobin A1C of 5.8%

b. Noon blood glucose of 52 mg/dl. The nurse should assess the patient with a blood glucose level of 52 mg/dl for symptoms of hypoglycemia, and give the patient some carbohydrate-containing beverage such as orange juice. The other values are within an acceptable range for a diabetic patient.

The factor related to cerebral blood flow that most often determines the extent of cerebral damage from a stroke is the a. amount of cardiac output. b. oxygen content of the blood. c. degree of collateral circulation. d. level of carbon dioxide in the blood.

c Rationale: The extent of the stroke depends on the rapidity of onset, size of the lesion, and presence of collateral circulation.

A patient with increased ICP is being monitored in the intensive care unit (ICU) with a fiberoptic catheter. Which order is a priority for you? A. Perform hourly neurologic checks. B. Take a complete set of vital signs. C. Administer the prescribed mannitol (Osmitrol). D. Give an H2-receptor blocker.

c The priority is to treat the known existing problem, and mannitol is the only thing that can do that. Because the patient is having the current pressure measured with objective numbers, treating the known problem is a priority over additional assessments. H2-blockers are given when corticosteroids are administered to help prevent gastrointestinal bleeding, but they are not a priority compared with the treatment of ICP.

A adult client has had laboratory work done as part of a routine physical examination. The nurse interprets that the client may have a mild degree of renal insufficiency if which of the following serum creatinine levels is noted? a) 0.2 mg/dlL b) 0.5 mg/dL c) 1.9 mg/dL d) 3.5 mg/dL

c) 1.9 mg/dLthe normal serum creatinine level foadults is 0.6 to 1.3 mg/dL.The client with a mild degree of renal insufficiency would have a slight elevated level. A creatinie level of 0.2 mg/dL is low, and a level of 0.5 mg/dL is just below normal. A creeatinie level of 3.5 mg/dL may be associated with acute or chronic renal failure.

The client in end-stage of renal failure had undergone kidney transplant. Which of the following assessment findings indicate kidney transplant rejection? a) increased urinary output, BUN = 15 mg/dL b) HCT = 50%, Hgb = 17 g/dl c) decreased urinary output, sudden weight gain d) decreased urinary output, sudden weight loss

c) decreased urinary output, sudden weight gain

Three year old Carlo has been admitted to the pediatric unit with a tentative diagnosis of nephrotic syndrome. Carlo's potential for impairment of skin integrity is related to: a) joint inflammation b) drug therapy c) edema d) generalized body rash

c) edema - management: reduce protein excretion Prevention of Skin Breakdown from Edema frequent turning keep nails short to prevent scratching meticulous skin care to dependent and edematous areas - sacrum, scrotum, labia, abdomen, legs loose clothing Monitor Edema

The home care nurse is making follow-up visits to a client following renal transplant. The nurse assesses the client for which signs of acute graft rejection? a) hypotension, graft tenderness, and anemia b) hypertension, oliguria, thirst, and hypothermia c) fever, hypertension, graft tenderness, and malaise d) fever, vomiting, hypotension, and copious amounts of dilute urine

c) fever, hypertension, graft tenderness, and malaise

Which of the following is not a sign or symptom of sepsis? a) fever b) tachycardia c) hypertension d) shortness of breath

c) hypertension

The patient in the acute phase of burn care has electrical burns on the left side of her body, type 2 diabetes mellitus, and a serum glucose level of 485 mg/dL. What should be the nurse's priority intervention to prevent a life-threatening complication of hyperglycemia for this burned patient? a. replace the blood lost b. maintain a neutral pH c. maintain fluid balance d. replace serum potassium

c. This patient is most likely experiencing hyperosmolar hyperglycemic syndrome (HHS). HHS dehydrates a patient rapidly. Thus HHS combined with the massive fluid losses of a burn tremendously increase this patient's risk for hypovolemic shock and serious hypotension. This is clearly the nurse's priority because the nurse must keep up with the patient's fluid requirements to prevent circulatory collapse caused by low intravascular volume. There is no mention of blood loss. Fluid resuscitation will help to correct the pH and serum potassium abnormalities.

A patient who has a positive test for human immunodeficiency virus (HIV) antibodies is admitted to the hospital with Pneumocystis jiroveci pneumonia (PCP) and a CD4+ T-cell count of less than 200 cells/uL. Based on diagnostic criteria established by the Centers for Disease Control and Prevention (CDC), which statement by the nurse is correct? a. "The patient meets the criteria for a diagnosis of an acute HIV infection." b. "The patient will be diagnosed with asymptomatic chronic HIV infection." c. "The patient has developed acquired immunodeficiency syndrome (AIDS)." d. "The patient will develop symptomatic chronic HIV infection in less than a year."

c. "The patient has developed acquired immunodeficiency syndrome (AIDS)."

In caring for a patient with burns to the back, the nurse knows that the patient is moving out of the emergent phase of burn injury when what happens? a. Serum sodium and potassium increase b. Serum sodium and potassium decrease. c. Edema and arterial blood gases improve. d. Diuresis occurs and hematocrit decreases.

d. In the emergent phase, the immediate, life-threatening problems from the burn, hypovolemic shock and edema, are treated and resolved. Toward the end of the emergent phase, fluid loss and edema formation end. Interstitial fluid returns to the vascular space and diuresis occurs. Urinary output is the most commonly used parameter to assess the adequacy of fluid resuscitation. The hemolysis of RBCs and thrombosis of burned capillaries also decreases circulating RBCs. When the fluid balance has been restored, dilution causes the hematocrit levels to drop. Initially sodium moves to the interstitial spaces and remains there until edema formation ceases, so sodium levels increase at the end of the emergent phase as the sodium moves back to the vasculature. Initially potassium level increases as it is released from injured cells and hemolyzed RBCs, so potassium levels decrease at the end of the emergent phase when fluid levels normalize.

A patient who is diagnosed with acquired immunodeficiency syndrome (AIDS) tells the nurse, "I feel obsessed with thoughts about dying. Do you think I am just being morbid?" Which response by the nurse is best? a. "Thinking about dying will not improve the course of AIDS." b. "It is important to focus on the good things about your life now." c. "Do you think that taking an antidepressant might be helpful to you?" d. "Can you tell me more about the kind of thoughts that you are having?"

d. "Can you tell me more about the kind of thoughts that you are having?"

What is a cause of primary hypothyroidism in adults? a. Malignant or benign thyroid nodules b. Surgical removal or failure of the pituitary gland c. Surgical removal or radiation of the thyroid gland d. Autoimmune-induced atrophy of the thyroid gland

d. Both Graves' disease and Hashimoto's thyroiditis are autoimmune disorders that eventually destroy the thyroid gland, leading to primary hypothyroidism. Thyroid tumors most often result in hyperthyroidism. Secondary hypothyroidism occurs as a result of pituitary failure and iatrogenic hypothyroidism results from thyroidectomy or radiation of the thyroid gland.

A patient with Graves' disease asks the nurse what caused the disorder. What is the best response by the nurse? a. "The cause of Graves' disease is not known, although it is thought to be genetic." b. "It is usually associated with goiter formation from an iodine deficiency over a long period of time." c. "Antibodies develop against thyroid tissue and destroy it, causing a deficiency of thyroid hormones." d. "In genetically susceptible persons, antibodies are formed that cause excessive thyroid hormone secretion."

d. In Graves' disease, antibodies to the TSH receptor are formed, attach to the receptors, and stimulate the thyroid gland to release triiodothyronine (T3), thyroxine (T4), or both, creating hyperthyroidism. The disease is not directly genetic but individuals appear to have a genetic susceptibility to develop autoimmune antibodies. Goiter formation from insufficient iodine intake is usually associated with hypothyroidism.

When providing discharge instructions to a patient who had a subtotal thyroidectomy for hyperthyroidism, what should the nurse teach the patient? a. Never miss a daily dose of thyroid replacement therapy. b. Avoid regular exercise until thyroid function is normalized. c. Use warm saltwater gargles several times a day to relieve throat pain. d. Substantially reduce caloric intake compared to what was eaten before surgery.

d. With the decrease in thyroid hormone postoperatively, calories need to be reduced substantially to prevent weight gain. When a patient has had a subtotal thyroidectomy, thyroid replacement therapy is not given because exogenous hormone inhibits pituitary production of TSH and delays or prevents the restoration of thyroid tissue regeneration. Regular exercise stimulates the thyroid gland and is encouraged. Saltwater gargles are used for dryness and irritation of the mouth and throat following radioactive iodine therapy.

In teaching a patient with hypertension about controlling the condition, the nurse recognizes that: a. all patients with elevated BP require medication b. obese persons must achieve a normal weight to lower BP c. It is not necessary to limit salt in the diet if taking a diuretic d. lifestyle modifications are indicated for persons with elevated BP

d. lifestyle modifications are indicated for persons with elevated BP

Which is a rapid-acting insulin with an onset of action of less than 15 minutes? insulin glargine (Lantus) insulin aspart (NovoLog) regular insulin (Humulin R) insulin detemir (Levemir)

insulin aspart (NovoLog)

Which long-acting insulin mimics natural, basal insulin with no peak action and a duration of 24 hours? insulin glargine (Lantus) insulin glulisine (Apidra) regular insulin (Humulin R) NPH insulin

insulin glargine (Lantus)

Which insulin can be administered by continuous intravenous infusion? insulin glargine (Lantus) insulin glulisine (Apidra) regular insulin (Humulin R) NPH insulin

regular insulin (Humulin R)


Conjuntos de estudio relacionados

Accounting Chapter 1: A Framework for Financial Accounting

View Set

Lectures 1, 2, 3, 6, 7, and 8 ( 1/2 of Final Exam )

View Set

Chapter 3: managing in a Global Environment

View Set

Nature of Insurance, Risk, Perils and Hazards

View Set

AP GOV: PLESSY V. FERGUSON (1896)

View Set

Common Regional Names and Anatomical Terms

View Set